Get Ahead EMQs Flashcards

1
Q

A 45-year-old woman collapses during a church service in summer. Her husband states that she turned pale and collapsed shortly after standing up. She did not jerk or lose urinary continence. She denies chest pain, shortness of breath and palpitations. She recovered after 30 seconds and now feels back to her usual self. All observations – including blood tests, ECG, and lying and standing blood pressure – are normal

A.	 Acute myocardial infarction
B.	 Cerebrovascular accident
C.	 Dissecting aortic aneurysm
D.	 Drug allergy
E.	 First-dose hypotension
F.	 Left ventricular failure
G.	 Ruptured abdominal aortic aneurysm
H.	 Stable angina
I.	 Stokes–Adams attack
J.	 Supraventricular tachycardia
K.	 Vasovagal syncope
L.	 Ventricular rupture
M.	Wolff–Parkinson–White syndrome
A

K – Vasovagal syncope

Vasovagal syncope occurs when there is excessive activation of the parasympathetic nervous system in response to certain stimuli, such as heat, fear and stress. The parasympathetic activity causes systemic vasodilatation and bradycardia, which triggers profound hypotension and cerebral hypoperfusion. Collapse is often preceded by a feeling of faintness, nausea, sweating and ringing in the ears. Occasionally, witnesses may describe the patient twitching and a loss of urinary incontinence, which may be confused with seizure activity. Following collapse, cerebral perfusion is restored and recovery is rapid. Situational syncope describes the scenario when vasovagal episodes are triggered by specific actions, such as coughing, urinating or having blood taken.

In all cases of collapse, the patient should be questioned about the presence of chest pain, shortness of breath, urinary incontinence, tongue biting, palpitations, weakness and paraesthesia. A pre-/peri-/post-collapse history is essential, and should be supplemented by a collateral history from a witness when available. Routine investigation should always include ECG, capillary blood glucose, and lying and standing blood pressure.

How well did you know this?
1
Not at all
2
3
4
5
Perfectly
2
Q

A 72-year-old man attends the emergency department following a fall. On examination, there is bruising on the right upper arm and the man is unable to extend his metacarpals on the same side. There is loss of sensation over the lateral dorsal aspect of the hand.

A.	 Accessory nerve
B.	 Anterior interosseous nerve
C.	 Axillary nerve
D.	 Distal median nerve
E.	 Distal ulnar nerve
F.	 Long thoracic nerve
G.	 Lower brachial plexus
H.	 Posterior interosseous nerve
I.	 Proximal median nerve
J.	 Proximal ulnar nerve
K.	 Radial nerve
L.	 Upper brachial plexus
A

K – Radial nerve

The radial nerve runs in close proximity to the shaft of the humerus in the spiral groove. Common causes of radial nerve palsies include humeral shaft fractures (as in this scenario, in which case patients also suffer bruising to the upper arm), compression of the nerve in the arm with prolonged use of ill-fitting crutches, and elbow dislocations or Monteggia fractures. Damage to the radial nerve is also seen in people who fall asleep with their arm hanging over the back of a chair (‘Saturday night palsy’).

The radial nerve supplies the extensors to the forearm and wrist, and radial nerve palsy results in an inability to extend the wrist or metacarpophalangeal joints (wrist drop), forearm extensor wasting and a loss of sensation in the anatomical snuffbox. The anatomical snuffbox is the name given to the triangular region on the radial dorsal aspect of the hand at the level of the carpal bones. It is so called as this was the surface used since the 16th century for placing and snorting ‘snuff’ (powdered tobacco).

How well did you know this?
1
Not at all
2
3
4
5
Perfectly
3
Q

A 28-year-old man is involved in a motorcycle accident. He has fractured both his femurs and complains of pain in his neck. On examination, his right arm is hanging by his side, is fully extended at the elbow and is rotated inwards. There is loss of sensation on the outer edge of the right arm and forearm.

A.	 Accessory nerve
B.	 Anterior interosseous nerve
C.	 Axillary nerve
D.	 Distal median nerve
E.	 Distal ulnar nerve
F.	 Long thoracic nerve
G.	 Lower brachial plexus
H.	 Posterior interosseous nerve
I.	 Proximal median nerve
J.	 Proximal ulnar nerve
K.	 Radial nerve
L.	 Upper brachial plexus
A

L – Upper brachial plexus

Upper brachial plexus injuries, also known as Erb’s palsy, involve the C5 and C6 nerve roots (the brachial plexus is made of the roots C5–T1). They are commonly caused by traction injuries, such as motorcycle accidents or birth injuries (due to pulling on the baby’s arm). There is flaccid paralysis of the arm abductors, lateral rotators of the shoulder and supinators, so the affected arm hangs limp and is medially rotated, extended at the elbow and pronated, with the hand pointing backwards – the ‘waiter’s tip’ position. Paralysis is accompanied by loss of sensation over the lateral arm and forearm.

How well did you know this?
1
Not at all
2
3
4
5
Perfectly
4
Q

A 46-year-old type 1 diabetic man attends a routine follow-up clinic. A urine dipstick analysis demonstrates 3+ protein, 1+ blood, no nitrates and no leucocytes. His annual blood tests show a sodium of 135 mmol/L, a potassium of 4.9 mmol/L, a creatinine of 130 µmol/L and a urea of 8.9 mmol/L.

A.	 Atherosclerosis
B.	 Autonomic neuropathy
C.	 Charcot’s joint
D.	 Connective tissue disease
E.	 Diabetes dermopathy
F.	 Lipoatrophy
G.	 Lipohypertrophy
H.	 Peripheral vascular disease
I.	 Neuropathic ulcer
J.	 Nephropathy
K.	 Necrobiosis lipoidica
L.	 Osteoarthritis
M.	Venous ulce
A

J – Nephropathy

Diabetic nephropathy is an important cause of morbidity and mortality in the diabetic population, with up to 30% developing chronic renal failure during their lifetime. Diabetic nephropathy has a complex pathophysiology, which can be simplified into three important processes: glomerular involvement, ischaemia and infection. Important contributors to the development of diabetic nephropathy are poor glycaemic control, hypertension and renovascular disease (e.g. renal artery stenosis). The earliest detectable sign of nephropathy is microalbuminuria, i.e. the presence of trace amounts of albumin in the urine that are not detectable using standard dipstick analysis. If untreated, microalbuminuria can progress to intermittent albuminuria and then persistent albuminuria. It is thought that patients with persistent albuminuria are 5–10 years away from end-stage renal failure, even if their creatinine is normal. To reduce the risk of developing end-stage renal failure, all diabetic patients should have their kidney function and urine tested on a regular basis. Good glycaemic and blood pressure control are essential and must be communicated to the patient. Angiotensin-converting enzyme (ACE) inhibitors and angiotensin II receptor antagonists have been shown to reduce the progression of microalbuminuria, and also contribute to blood pressure control.

How well did you know this?
1
Not at all
2
3
4
5
Perfectly
5
Q

A 37-year-old type 1 diabetic man presents to his GP complaining of a painless discoloured lesion over the lateral aspect of his right lower leg. It began as a well-circumscribed red nodule, but enlarged and flattened over time. The surface is brownish-yellow and scaly and has a number of telangiectatic vessels.

A.	 Atherosclerosis
B.	 Autonomic neuropathy
C.	 Charcot’s joint
D.	 Connective tissue disease
E.	 Diabetes dermopathy
F.	 Lipoatrophy
G.	 Lipohypertrophy
H.	 Peripheral vascular disease
I.	 Neuropathic ulcer
J.	 Nephropathy
K.	 Necrobiosis lipoidica
L.	 Osteoarthritis
M.	Venous ulce
A

K – Necrobiosis lipoidica

Necrobiosis lipoidica diabeticorum is a rare dermatological complication of diabetes mellitus seen in approximately 1% of patients. It is most often occurs on the anterolateral aspect of the lower limb. The lesion usually begins as a small red nodule that increases in size and flattens over time. Eventually, the lesion becomes depressed, irregular, scaly or waxy, and develops a brownish-yellow discoloration. If the lesion is not ulcerated, topical steroids in association with appropriate dressings can be applied. If ulceration is present, immunosuppressive agents such as ciclosporin can be beneficial. Treatment is often unsuccessful.

Lipoatrophy describes a localized loss of fat tissue, which may occur in people with diabetes at the site of repeated subcutaneous insulin injections. The opposite condition is lipohypertrophy, the accumulation of fat at the point of repeated subcutaneous injections. Both of these conditions can be avoided by regularly rotating injection sites.

How well did you know this?
1
Not at all
2
3
4
5
Perfectly
6
Q

A 12-year-old boy is brought to the GP by his mother. Over the last couple of years, he has suffered from repeated chest infections, and his mother is worried that he is not putting on any weight. On examination, there are coarse crepitations throughout both lung lobes.

A.	 Asbestos-related lung disease
B.	 Asthma
C.	 Bronchiectasis
D.	 Chronic obstructive pulmonary disease
E.	 Cryptogenic fibrosing alveolitis
F.	 Cystic fibrosis
G.	 Extrinsic allergic alveolitis
H.	 Mesothelioma
I.	 Non-small cell lung tumour
J.	 Open pneumothorax
K.	 Sarcoidosis
L.	 Secondary lung metastases
M.	 Sleep apnoea
N.	 Small cell lung tumour
O.	 Tuberculosis
A

F – Cystic fibrosis

Cystic fibrosis (CF) is the commonest autosomal recessive condition in white populations in the UK, affecting 1 in 2500 live births. CF is caused by an abnormal gene coding for the cystic fibrosis transmembrane regulator protein (CFTR), located on chromosome 7. CFTR is a cAMP-dependent chloride channel blocker. The most common mutation in CF is the ∆F508 mutation. The poor transport of chloride ions and water across epithelial cells of the respiratory and pancreatic exocrine glands in CF results in an increased viscosity of secretions. The range of presentations is varied, including recurrent chest infections, failure to thrive due to malabsorption and liver disease. In the neonatal period, infants may present with prolonged neonatal jaundice, bowel obstruction (meconium ileus) or rectal prolapse.

The boy in this case has bilateral coarse crepitations. Given the history, these are a sign of bronchiectasis. Bronchiectasis is an abnormal dilatation of the bronchioles. Causes of bronchiectasis can be congenital (ciliary dyskinesia or CF) or acquired (whooping cough, measles or tuberculosis). Pus accumulates in the dilated bronchi, resulting in a productive cough that is worse in the morning and brought about by changes in posture. Bronchiectasis is associated with clubbing of the fingers. The best investigation to confirm bronchiectasis is CT. Management is by physiotherapy (postural drainage and chest percussion) and prompt antibiotics for respiratory tract infections.

How well did you know this?
1
Not at all
2
3
4
5
Perfectly
7
Q

A 25-year-old man presents with facial and ankle swelling. He is found to have 3+ of protein in his urine on dipstick analysis. Blood tests reveal an albumin of 16 g/L.

A.	 Drug-induced glomerulonephritis
B.	 Focal segmental glomerulosclerosis
C.	 Goodpasture’s syndrome
D.	 IgA nephropathy
E.	 Membranous glomerulonephritis
F.	 Mesangiocapillary glomerulonephritis
G.	 Minimal-change glomerulonephritis
H.	 Post-streptococcal glomerulonephritis
I.	 Rapidly progressive glomerulonephritis
J.	 Rhabdomyolysis
A

E – Membranous glomerulonephritis

Membranous glomerulonephritis is the most common cause of nephrotic syndrome in adults. In this condition, immune complexes are formed when circulating antibodies bind to basement membrane antigens or antigens deposited in the glomerulus from the circulation. The immune complexes activate an immunological response that involves the complement cascade. This response damages the glomerular basement membrane, making it more permeable. Membranous glomerulonephritis is usually idiopathic (90%), but may be secondary to a number of disease processes, including systemic lupus erythematosus, hepatitis B, systemic infection and drugs.

Treatment is largely the same as for nephrotic syndrome in children (see above). The prognosis is variable. Approximately one-third of patients recover spontaneously, one-third will respond to immunosuppression and one-third will develop end-stage renal failure. Up to 5% of patients will develop renal vein thrombosis.

How well did you know this?
1
Not at all
2
3
4
5
Perfectly
8
Q

A 15-year-old girl presents to the GP 2 weeks after being diagnosed with tonsillitis. She is generally unwell and puffy and complaining of dark urine. Urine dipstick analysis shows 3+ of protein and 3+ of blood. She is found to have high titres of antistreptolysin O antibody.

A.	 Drug-induced glomerulonephritis
B.	 Focal segmental glomerulosclerosis
C.	 Goodpasture’s syndrome
D.	 IgA nephropathy
E.	 Membranous glomerulonephritis
F.	 Mesangiocapillary glomerulonephritis
G.	 Minimal-change glomerulonephritis
H.	 Post-streptococcal glomerulonephritis
I.	 Rapidly progressive glomerulonephritis
J.	 Rhabdomyolysis
A

H – Post-streptococcal glomerulonephritis

This form of glomerulonephritis is usually seen 1–2 weeks after an infection caused by a group A β-haemolytic streptococcus, e.g. tonsillitis or cellulitis. The pathophysiology involves immune complex deposition within the glomerular basement membrane. Patients present with acute nephritic syndrome, i.e. fluid retention, peripheral oedema, hypertension, proteinuria and haematuria. The urine is classically dark in colour and is sometimes referred to as ‘Pepsi urine’. Investigation usually reveals proteinuria, haematuria, elevated anti-streptolysin O titres and reduced complement (C3). In cases with a clear history and consistent initial investigation results, renal biopsy is often not required. Management is largely supportive with fluid restriction and antihypertensive medications.

How well did you know this?
1
Not at all
2
3
4
5
Perfectly
9
Q

A 45-year-old man is admitted with headache and neck stiffness. A lumbar puncture is performed and reveals the following results: clear fluid, lymphocytes 2/mm3, neutrophils 1/mm3, low glucose, high protein, positive India ink stain.

A.	 Cryptococcus neoformans
B.	 Haemophilus influenzae
C.	 Malignancy
D.	 Multiple sclerosis
E.	 Mumps
F.	 Mycobacterium tuberculosis
G.	 Neisseria meningitidis
H.	 Streptococcus pneumoniae
A

A – Cryptococcus neoformans (fungal)

How well did you know this?
1
Not at all
2
3
4
5
Perfectly
10
Q

A 43-year-old man with a long history of bipolar disorder presents to his GP complaining of weight gain, lethargy and feeling cold all the time. Blood tests show a low T 4 level and a high TSH level.

A.	 De Quervain’s thyroiditis
B.	 Graves’ disease
C.	 Hashimoto’s thyroiditis
D.	 Hypopituitarism
E.	 Hypothyroidism secondary to amiodarone therapy
F.	 Hypothyroidism secondary to lithium therapy
G.	 Sick euthyroid syndrome
H.	 Single toxic nodule goitre
I.	 Thyroid cancer
J.	 Toxic multinodular goitre
A

F – Hypothyroidism secondary to lithium therapy

Lithium, a drug used to treat bipolar disorder (manic depression), is one of a number of medications known to affect the thyroid gland. Lithium can cause goitre formation and inhibit T4 and triiodothyronine (T3) secretion from the thyroid gland. Therefore, prior to starting lithium, every patient should receive a neck examination and have their thyroid function checked. The majority of patients taking lithium become subclinically hypothyroid (i.e. are asymptomatic, with a normal T 4 level and a high TSH). Unless the patient is acutely unwell, it is usual practice to continue the lithium and start the patient on T4 replacement. After commencing therapy, thyroid function should be tested every 6 months.

How well did you know this?
1
Not at all
2
3
4
5
Perfectly
11
Q

A 38-year-old man who is currently in hospital being treated for a severe chest infection is shown to have low T4 and TSH levels.

A.	 De Quervain’s thyroiditis
B.	 Graves’ disease
C.	 Hashimoto’s thyroiditis
D.	 Hypopituitarism
E.	 Hypothyroidism secondary to amiodarone therapy
F.	 Hypothyroidism secondary to lithium therapy
G.	 Sick euthyroid syndrome
H.	 Single toxic nodule goitre
I.	 Thyroid cancer
J.	 Toxic multinodular goitre
A

G – Sick euthyroid syndrome

Sick euthyroid syndrome describes abnormal thyroid function in the presence of systemic disease. There are many different subtypes of sick euthyroidism, but, as a general rule, the T3 and T4 levels are low, with TSH levels falling in more severe disease (i.e. everything is low). As the patient recovers from the underlying disease, the thyroid function usually returns to normal.

How well did you know this?
1
Not at all
2
3
4
5
Perfectly
12
Q

A 70-year-old man presents with a 6-month history of worsening forgetfulness and intellectual decline. He has, however, had frequent episodes of relative lucidness during this period. He tries to chase soldiers whom he thinks he sees in the garden, but his gait has markedly slowed down. His sleeping pattern is now irregular.

A.	 Alzheimer’s disease
B.	 Creutzfeldt–Jakob disease
C.	 Depression
D.	 HIV dementia
E.	 Huntington’s disease
F.	 Lewy body dementia
G.	 Normal-pressure hydrocephalus
H.	 Parkinson’s disease
I.	 Pick’s disease
J.	 Vascular dementia
K.	 Wernicke–Korsakoff syndrome
A

F – Lewy body dementia

Lewy body dementia is the second most common dementia after Alzheimer’s disease. Characteristic features of Lewy body dementia include day-today fluctuating levels of cognitive functioning, visual hallucinations, sleep disturbance, transient loss of consciousness, recurrent falls and parkinsonian features (tremor, hypokinesia, rigidity and postural instability). Although people with Lewy body dementia are prone to hallucination, antipsychotics should be avoided, as they precipitate severe parkinsonism in 60%. Lewy bodies are abnormalities of the cytoplasm found within neurons, containing various proteins and granular material. They are found in the cerebral cortex in patients with Lewy body dementia, and are also found in patients with Parkinson’s disease

How well did you know this?
1
Not at all
2
3
4
5
Perfectly
13
Q

A 52-year-old woman presents to the emergency department following her first-ever seizure. Her husband said that she had to leave her job because, over a period of 2 months, she rapidly lost the ability to cope with its intellectual demands. A previous cryptic crossword fanatic, she cannot even fill out the simplest tabloid grid. On further questioning, she admits to muscle weakness and difficulty walking.

A.	 Alzheimer’s disease
B.	 Creutzfeldt–Jakob disease
C.	 Depression
D.	 HIV dementia
E.	 Huntington’s disease
F.	 Lewy body dementia
G.	 Normal-pressure hydrocephalus
H.	 Parkinson’s disease
I.	 Pick’s disease
J.	 Vascular dementia
K.	 Wernicke–Korsakoff syndrome
A

B – Creutzfeldt–Jakob disease

Creutzfeldt–Jakob disease (CJD) is a rapidly progressive dementia caused by prions (infectious agents composed only of protein). The prion proteins can be transmitted by neurosurgical instruments and human-derived pituitary hormones. Features of CJD include rapid cognitive impairment, which may be preceded by anxiety and depression. Eventually, physical features become prominent, including muscle disturbance (rigidity, tremor, wasting, spasticity, fasciculations, cyclonic jerks and choreoathetoid movements). Convulsions may also occur. The EEG is characteristic (showing stereotypical sharp wave complexes). Death occurs within 6–8 months.

New variant CJD (nvCJD) occurs secondary to ingestion of bovine spongiform encephalopathy (BSE)-infected beef. It is more common in younger adults. The features are as for CJD, but decline is slower, with death occurring within 18 months. There are no typical EEG changes in nvCJD, although there is a characteristic feature on MRI (symmetrical hyperintensity in the posterior nucleus of the thalamus – the pulvinar sign).

How well did you know this?
1
Not at all
2
3
4
5
Perfectly
14
Q

A 69-year-old woman presents after a fall, having been unsteady on her feet for some time. She has severe memory loss, but is relatively well orientated. She has recently developed urinary incontinence. On examination, she has a wide-based gait.

A.	 Alzheimer’s disease
B.	 Creutzfeldt–Jakob disease
C.	 Depression
D.	 HIV dementia
E.	 Huntington’s disease
F.	 Lewy body dementia
G.	 Normal-pressure hydrocephalus
H.	 Parkinson’s disease
I.	 Pick’s disease
J.	 Vascular dementia
K.	 Wernicke–Korsakoff syndrome
A

G – Normal-pressure hydrocephalus

Normal-pressure hydrocephalus is characterized by the triad of dementia (mainly memory problems), gait disturbance and urinary incontinence. It is caused by an increased volume of CSF, but with only a slightly raised pressure (as the ventricles dilate to compensate). There is an underlying obstruction in the subarachnoid space that prevents CSF from being reabsorbed but allows it to flow from the ventricular system into the subarachnoid space. Diagnosis is by lumbar puncture (to demonstrate a normal CSF opening pressure) followed by head CT/MRI (showing enlarged ventricles). Treatment is with ventriculoperitoneal shunting.

How well did you know this?
1
Not at all
2
3
4
5
Perfectly
15
Q

A 26-year-old woman presents with a 3-month history of general malaise, fever and weight loss. Over the last week, she has developed an intermittent cramping pain in her right arm. On examination, the upper limbs appear normal, but the radial pulses are not palpable.

A.	 Behçet’s disease
B.	 Churg–Strauss syndrome
C.	 Giant cell arteritis
D.	 Kawasaki’s disease
E.	 Microscopic polyangiitis
F.	 Polyarteritis nodosa
G.	 Polymyalgia rheumatica
H.	 Takayasu’s arteritis
I.	 Wegener’s disease
A

H – Takayasu’s arteritis

Vasculitis is an inflammation of the blood vessels. Takayasu’s arteritis (also known as ‘pulseless disease’) is a rare granulomatous inflammation of the aorta and its major branches. This results in poor blood flow to the peripheries and a lack of distal pulses. It is commonest in Japanese women. Patients present with systemic illness (malaise, fever and weight loss) and arm claudication. A third of patients suffer visual disturbance. Examination reveals absent pulses and arterial bruits. Diagnosis is by angiography, which demonstrates the inflamed constricted major vessels. Treatment is with steroids, but surgery may be required to bypass significantly stenosed or obliterated vessels.

How well did you know this?
1
Not at all
2
3
4
5
Perfectly
16
Q

A 32-year-old man presents with shortness of breath. He feels that it is similar to the asthma that he had as a child. He has also developed hayfever recently, which he has never had before. On examination, a palpable, purple rash is seen on the legs.

A.	 Behçet’s disease
B.	 Churg–Strauss syndrome
C.	 Giant cell arteritis
D.	 Kawasaki’s disease
E.	 Microscopic polyangiitis
F.	 Polyarteritis nodosa
G.	 Polymyalgia rheumatica
H.	 Takayasu’s arteritis
I.	 Wegener’s disease
A

B – Churg–Strauss syndrome

Churg–Strauss syndrome is a rare systemic vasculitis that is associated with eosinophilia and asthma. The disease may be triphasic, with a prodromal period (rhinitis and allergies), eosinophilia (asthma or eosinophilic gastroenteritis) and finally a systemic vasculitis. Churg–Strauss syndrome is associated with perinuclear antineutrophil cytoplasmic antibodies (pANCAs). Treatment is with steroids.

How well did you know this?
1
Not at all
2
3
4
5
Perfectly
17
Q

A 29-year-old man complains of recurrent painful ulceration in his mouth and on his genitals. The ulceration is clearly seen on examination. Swab cultures are taken and found to be negative for herpes simplex virus.

A.	 Behçet’s disease
B.	 Churg–Strauss syndrome
C.	 Giant cell arteritis
D.	 Kawasaki’s disease
E.	 Microscopic polyangiitis
F.	 Polyarteritis nodosa
G.	 Polymyalgia rheumatica
H.	 Takayasu’s arteritis
I.	 Wegener’s disease
A

A – Behçet’s disease

Behçet’s disease (pronounced ‘beh-chet’) is a chronic vasculitis most common in Turkey and the eastern Mediterranean. There is a strong association with HLA-B5 and disease is more severe in males. Behçet’s disease is characterized by an occlusive vasculitis and venulitis. The diagnosis is based on the clinical features, which include recurrent oral and genital ulceration, recurrent iritis, skin lesions, and thrombophlebitis. The pathergy reaction (where red papules >2 mm develop after 48 hours at sites of needle pricks) is pathognomonic of Behçet’s disease.

How well did you know this?
1
Not at all
2
3
4
5
Perfectly
18
Q

A 65-year-old woman presents with a pain in her shoulders. The pain came on suddenly this morning, and now she is barely able to move her arms. She is otherwise well. On examination, shoulder movement is limited by stiffness bilaterally

A.	 Behçet’s disease
B.	 Churg–Strauss syndrome
C.	 Giant cell arteritis
D.	 Kawasaki’s disease
E.	 Microscopic polyangiitis
F.	 Polyarteritis nodosa
G.	 Polymyalgia rheumatica
H.	 Takayasu’s arteritis
I.	 Wegener’s disease
A

G – Polymyalgia rheumatica

Polymyalgia rheumatica is not a vasculitis but is found in 50% of people with temporal arteritis (below). It is commonest in females over 50 years. Patients present with abrupt-onset proximal muscle pain (shoulder and hips) and stiffness without weakness. (The lack of weakness helps distinguish polymyalgia rheumatica from polymyositis.) Symptoms are worse in the morning. Treatment is with corticosteroids, and the response is prompt and dramatic.

Giant cell arteritis (temporal arteritis) is an inflammatory vasculitis of the cranial branches arising from the aorta. It is most common in the over-50s and is twice as frequent in females. Clinical features include general malaise, temporal headache, scalp tenderness and jaw claudication. Eventually, visual disturbance or visual loss can occur due to ischaemic optic neuritis caused by arteritis of the posterior ciliary artery and branches of the ophthalmic arteries. On examination, an enlarged, tender, non-pulsatile temporal artery is seen on the affected side. Temporal artery biopsy is the definitive investigation (showing patchy granulomatous inflammation), but skip lesions may be present and investigation should not delay treatment. Management is with prednisolone.

How well did you know this?
1
Not at all
2
3
4
5
Perfectly
19
Q

A 42-year-old man presents with a 2-month history of recurrent nosebleeds. More recently, he has started coughing up blood as well. On examination, there is a deformity in the bridge of his nose.

A.	 Behçet’s disease
B.	 Churg–Strauss syndrome
C.	 Giant cell arteritis
D.	 Kawasaki’s disease
E.	 Microscopic polyangiitis
F.	 Polyarteritis nodosa
G.	 Polymyalgia rheumatica
H.	 Takayasu’s arteritis
I.	 Wegener’s disease
A

I – Wegener’s disease

Wegener’s disease is a granulomatous necrotizing vasculitis characterized by a classic triad of involvement: upper airway pathology (epistaxis, saddle nose deformity, rhinitis, deafness and proptosis), respiratory disease (pulmonary nodules and pulmonary haemorrhage) and renal disease (glomerulonephritis). It is associated with cytoplasmic antineutrophil cytoplasmic antibodies (cANCAs) in 90% of cases. Treatment is with steroids and immunosuppressants

How well did you know this?
1
Not at all
2
3
4
5
Perfectly
20
Q

A 54-year-old woman with end-stage liver failure secondary to primary biliary cirrhosis is noticed to have a rising serum creatinine and urea, and is passing only small volumes of urine each day.

A.	 Clotting factor deficiency
B.	 Hepatic encephalopathy
C.	 Hepatorenal syndrome
D.	 Hypoalbuminaemia
E.	 Hypoglycaemia
F.	 Lower gastrointestinal bleed
G.	 Ruptured oesophagus
H.	 Seizure
I.	 Spontaneous bacterial peritonitis
J.	 Thrombocytopenia
K.	 Upper gastrointestinal bleed
A

C – Hepatorenal syndrome

Hepatorenal syndrome is seen in up to 20% of patients with cirrhosis and portal hypertension. In hepatorenal syndrome, the patient develops acute renal failure despite having histologically normal kidneys. It is thought to arise secondary to the release of vasoactive substances that cause dilatation of the splanchnic vasculature and constriction of the renal cortical vasculature. This combination of events reduces the glomerular filtration rate. The treatment of hepatorenal syndrome involves restoring the intravascular volume with human albumin solution and reversing the splanchnic dilatation with potent arterial vasoconstrictors such as terlipressin. Severe and refractory disease may require liver transplantation for cure.

How well did you know this?
1
Not at all
2
3
4
5
Perfectly
21
Q

A 34-year-old woman presents with multiple painful lesions over her body. On examination, each of these lesions is soft and mobile and 2cm in size. She has no other symptoms.

A.	 Cavernous haemangioma
B.	 Deep capillary naevus
C.	 Dercum’s disease
D.	 Ganglion
E.	 Granuloma annulare
F.	 Kaposi’s sarcoma
G.	 Neurofibroma
H.	 Pyogenic granuloma
I.	 Sebaceous cyst
J.	 Seborrhoeic keratosis
K.	 Superficial capillary naevus
A

C – Dercum’s disease

These woman’s lesions are lipomas – soft, mobile lesions composed of fatty tissue that are usually painless. However, the presence of multiple painful lipomas is known as Dercum’s disease (or adiposis dolora). This occurs most commonly in obese middle-aged women, and may be accompanied by headaches, amenorrhoea and reduced sweating. Simple lipomas can be removed by excision for cosmetic reasons.

How well did you know this?
1
Not at all
2
3
4
5
Perfectly
22
Q

A 5-year-old girl was admitted to the emergency department with a seizure. On examination, she has a well-demarcated, purple, flat lesion over her right cheek and forehead. Her parents say that this has been present since birth.

A.	 Cavernous haemangioma
B.	 Deep capillary naevus
C.	 Dercum’s disease
D.	 Ganglion
E.	 Granuloma annulare
F.	 Kaposi’s sarcoma
G.	 Neurofibroma
H.	 Pyogenic granuloma
I.	 Sebaceous cyst
J.	 Seborrhoeic keratosis
K.	 Superficial capillary naevus
A

B – Deep capillary naevus

A deep capillary naevus (or port-wine stain) is a malformation of the capillaries in the deep and superficial dermis. These are congenital malformations that can occur anywhere in the body but are most often found unilaterally on the face. Occasionally, a port-wine stain is associated with seizures, learning difficulties and eye abnormalities (glaucoma and optic atrophy) due to underlying cranial malformations. This is known as Sturge–Weber syndrome and is usually associated with a port-wine stain in the distribution of the ophthalmic or maxillary division of the trigeminal nerve.

Superficial capillary naevi (also known as salmon patches) are small, flat, pink patches of skin with poorly defined borders. They are commonly found on the forehead (‘angel’s kiss’) or on the nape of the neck (‘stork mark’). Most superficial naevi disappear in the first year of life

How well did you know this?
1
Not at all
2
3
4
5
Perfectly
23
Q

An 18-month-old boy is brought to the GP by his parents with a red lesion on his forehead. This was not present at birth, but has been growing for 3 months. On examination, the lesion is 3 cm, bright red and well defined. There are no other symptoms.

A.	 Cavernous haemangioma
B.	 Deep capillary naevus
C.	 Dercum’s disease
D.	 Ganglion
E.	 Granuloma annulare
F.	 Kaposi’s sarcoma
G.	 Neurofibroma
H.	 Pyogenic granuloma
I.	 Sebaceous cyst
J.	 Seborrhoeic keratosis
K.	 Superficial capillary naevus
A

A – Cavernous haemangioma

A cavernous haemangioma (or strawberry naevus) is a condition that appears in the first months of life as a bright-red lesion on the face or trunk that grows rapidly. Occasionally, these lesions bleed or ulcerate. Cavernous haemangiomas eventually regress and disappear spontaneously, so intervention is required only if lesions persist beyond a few years of age. Cavernous haemangiomas may rarely be associated with thrombocytopenia and haemolytic anaemia secondary to trapping and destruction of platelets and erythrocytes within the lesions. This is known as Kasabach–Merritt syndrome.

How well did you know this?
1
Not at all
2
3
4
5
Perfectly
24
Q

A 15-month-old boy is brought to the emergency department with a grossly swollen right knee. His parents claim that he has had several episodes of bleeding into his joints and muscles over the previous 6 months. A clotting screen was requested and showed a significantly prolonged APTT with a normal PT and bleeding time. The full blood count showed a mild microcytic anaemia.

A.	 Acquired haemophilia
B.	 Antibiotic side-effect
C.	 Complication of warfarin therapy
D.	 Congenital haemophilia
E.	 Disseminated intravascular coagulation
F.	 Global reduction in clotting factor synthesis
G.	 Heparin overdose
H.	 Hereditary haemorrhagic telangiectasia
I.	 Hypofibrinogenaemia
J.	 Pancytopenia
K.	 Thrombocytopenia
L.	 Von Willebrand’s disease
A

D – Congenital haemophilia

Haemophilia A is an X-linked recessive disorder of coagulation in which the patient cannot synthesize clotting factor VIII due to a gene mutation. Haemophilia B (also known as Christmas disease) is caused by an inability to synthesize factor IX and is clinically indistinguishable from the much more common haemophilia A. Although usually familial, a significant proportion of cases are caused by sporadic mutations. Factors VIII and IX are essential in the extrinsic clotting cascade, meaning that patients with haemophilia have a prolonged APTT. The intrinsic pathway does not require factors VIII or IX, and is therefore unaffected by haemophilia – shown by a normal PT. The bleeding time is also normal. Symptoms usually begin when the patient becomes mobile, i.e. when they begin to crawl or walk. Patients with haemophilia typically suffer painful recurrent bleeds into the joints and soft tissues (haemarthrosis), which may eventually lead to crippling arthropathy and neuropathy.

How well did you know this?
1
Not at all
2
3
4
5
Perfectly
25
Q

Six months after her original diagnosis, Mrs X (80 y/o, diagnosed with chronic heart failure) visits her GP because she still gets very short of breath, especially when she lies down. Her current medications include enalapril, atenolol, zopiclone, aspirin and furosemide. What additional treatment should the doctor prescribe her?

A.	 Angiotensin II receptor antagonist
B.	 Angiotensin-converting enzyme inhibitors
C.	 Amlodipine
D.	 β-Blockers
E.	 Brain-type natriuretic peptide
F.	 Digoxin
G.	 Lipid profile
H.	 Loop diuretics
I.	 Myoglobin
J.	 Potassium-sparing diuretics
K.	 Spironolactone
L.	 Thiazide diuretics
M.	Thyroid function tests
N.	 Transoesophageal echocardiography
O.	 Transthoracic Doppler echocardiography
P.	 Troponin I
A

K – Spironolactone

Spironolactone is an aldosterone antagonist; therefore it causes increased sodium secretion and reduced potassium excretion. Because it does not cause hypokalaemia (unlike the thiazide and loop diuretics), spironolactone is known as a potassium-sparing diuretic. Amiloride is another example of such a drug. Spironolactone is indicated in patients with severe symptomatic disease that is not controlled by optimal medical therapy to improve symptoms and mortality. Since this medication reduces potassium excretion by the kidneys, it should not be given to patients with a potassium level over 5.0 mmol/L or to patients taking potassium supplements. Patients should have their serum potassium measured prior to prescription and at regular intervals thereafter in order to detect hyperkalaemia. Other well-known side-effects of spironolactone include hyponatraemia, gynaecomastia and menstrual disturbance.

How well did you know this?
1
Not at all
2
3
4
5
Perfectly
26
Q

A 45-year-old woman presents to the GP complaining of a new rash. There are multiple, light-brown, depressed lesions on her shins, each around 2 cm in size. She has diabetes mellitus, but is otherwise well.

A.	 Acanthosis nigricans
B.	 Alopecia
C.	 Diabetic dermopathy
D.	 Eruptive xanthomas
E.	 Erythema ab igne
F.	 Hyperhidrosis
G.	 Necrobiosis lipoidica diabeticorum
H.	 Paget’s disease
I.	 Pretibial myxoedema
J.	 Thrombophlebitis migrans
K.	 Tylosis
L.	 Xanthelasma
A

C – Diabetic dermopathy

Diabetic dermopathy describes the presence of depressed pigmented scars in the shin. It is associated with diabetic microangiopathy. Other skin features of diabetes include necrobiosis lipoidica diabeticorum (shiny, atrophic, yellowish-red plaques on the shins), cheiroarthropathy (a sclerodermalike thickening of the skin of the hands), granuloma annulare (small, papular lesions arranged in a ring and found on the back of the hands or feet) and acanthosis nigricans.

How well did you know this?
1
Not at all
2
3
4
5
Perfectly
27
Q

A 45-year-old woman presents to the GP complaining of a new rash. There is a dark-brown, lacy rash over the front of both of her shins. She has recently started treatment for an underactive thyroid, but is otherwise well.

A.	 Acanthosis nigricans
B.	 Alopecia
C.	 Diabetic dermopathy
D.	 Eruptive xanthomas
E.	 Erythema ab igne
F.	 Hyperhidrosis
G.	 Necrobiosis lipoidica diabeticorum
H.	 Paget’s disease
I.	 Pretibial myxoedema
J.	 Thrombophlebitis migrans
K.	 Tylosis
L.	 Xanthelasma
A

E – Erythema ab igne

Erythema ab igne is a brown lacy rash seen on skin that has been exposed to heat for long periods of time. It classically develops in hypothyroid patients who are cold and spend a lot of time in front of the fire. Excessive hot water bottle use can also result in the rash.

Other cutaneous features of hypothyroidism include alopecia (especially loss of the outer third of the eyebrows), dry coarse hair, puffy yellow skin, periorbital oedema, xanthomas and a malar flush on an otherwise pale face (‘strawberries and cream’ appearance).

How well did you know this?
1
Not at all
2
3
4
5
Perfectly
28
Q

A 34-year-old woman has been suffering from profuse diarrhoea and vomiting for 4 days. She is now complaining of extreme muscle weakness and muscle cramps.

A.	 3rd-degree heart block
B.	 Atrial fibrillation
C.	 Atrial flutter
D.	 Bifid P-waves
E.	 Delta wave
F.	 J-waves	
G.	 Sinus arrhythmia
H.	 Sinus tachycardia
I.	 Tented T-waves	
J.	 U-waves
A

J – U-waves

The 4-day history of diarrhoea and vomiting followed by the development of severe muscle weakness and cramps suggests hypokalaemia secondary to excessive gastrointestinal potassium loss. Other causes of reduced total body potassium include diuretic use, Conn’s syndrome, excessive sweating and burns. Potassium can also be redistributed from the extracellular compartment into the intracellular compartment, resulting in a reduction in bioavailable potassium despite there being normal total body potassium levels. Causes of potassium redistribution include β2-antagonist use (e.g. salbutamol), excess insulin administration and alkalosis. The clinical features of hypokalaemia become evident with serum potassium levels less than 2.5 mmol/L. These include lethargy, polyuria, profound muscle weakness, muscle cramps, palpitations and arrhythmia. ECG changes in hypokalaemia include U-wave formation (upward deflections following the T-waves), flattened T-waves, ST-segment depression, and atrial and ventricular arrhythmias. Treatment of hypokalaemia involves correcting the underlying cause and replacing the lost potassium. Patients with potassium levels above 2.5 mmol/L can usually be managed with oral potassium supplements.

How well did you know this?
1
Not at all
2
3
4
5
Perfectly
29
Q

An 85-year-old man with dementia is brought to the emergency department by the ambulance after escaping from his care home during the night. He was found wandering on the local moors by a farmer in the early hours of the morning. He is shivering and confused. His core temperature is 31°C.

A.	 3rd-degree heart block
B.	 Atrial fibrillation
C.	 Atrial flutter
D.	 Bifid P-waves
E.	 Delta wave
F.	 J-waves	
G.	 Sinus arrhythmia
H.	 Sinus tachycardia
I.	 Tented T-waves	
J.	 U-waves
A

F – J-waves

This patient is hypothermic secondary to prolonged environmental exposure. Severe hypothermia can lead to coagulopathy, bradycardia, heart failure, arrhythmia and death. The ECG in severely affected patients may show an upward deflection following the R-wave of the QRS complex (J-wave). Patients with hypothermia should be re-warmed slowly at a rate no greater than 0.5°C/h, as rapid re-warming can cause vasodilatation, hypotension and circulatory collapse. Methods of re-warming include removal of wet clothing, supplying warmed humidified oxygen, applying a bear hugger device and infusing warm saline intravenously. More invasive methods of re-warming include peritoneal, pleural and bladder lavage with warmed fluid. Due to the risk of arrhythmia, hypothermic patients should be managed on a cardiac and blood pressure monitor.

How well did you know this?
1
Not at all
2
3
4
5
Perfectly
30
Q

This serological marker would indicate chronic hepatitis B infection if detected 6 months after the original infection.

A.	 α-Fetoprotein
B.	 Alanine aminotransferase
C.	 Hepatitis B core antibody (HBcAb)
D.	 Hepatitis B core antigen (HBcAg)
E.	 Hepatitis B e antigen (HBeAg)
F.	 Hepatitis B surface antibody (HBsAb)
G.	 Hepatitis B surface antigen (HBsAg)
H.	 Hepatitis B RNA titre
A

G – Hepatitis B surface antigen (HBsAg)

If HBsAg is still detected in the serum 6 months after an acute HBV infection, the patient has become a chronic carrier of the virus. The risk of developing chronic HBV infection is related to age at the time of infection. The majority of infected neonates develop chronic infection, whereas only 5–10% of adults do so. Patients with chronic hepatitis B are at risk of developing cirrhosis, liver failure and hepatocellular carcinoma. The treatment of chronic hepatitis B can involve interferon-α, peginterferon-α2a, lamivudine and (in some patients) eventual liver transplantation.

How well did you know this?
1
Not at all
2
3
4
5
Perfectly
31
Q

This serological marker indicates high infectivity in a chronic hepatitis B carrier.

A.	 α-Fetoprotein
B.	 Alanine aminotransferase
C.	 Hepatitis B core antibody (HBcAb)
D.	 Hepatitis B core antigen (HBcAg)
E.	 Hepatitis B e antigen (HBeAg)
F.	 Hepatitis B surface antibody (HBsAb)
G.	 Hepatitis B surface antigen (HBsAg)
H.	 Hepatitis B RNA titre
A

E – Hepatitis B e antigen (HBeAg)

HBeAg can be present in both acute and chronic hepatitis B and indicates a high level of infectivity. The presence of HBeAg in chronic hepatitis B suggests that the patient is infective and has an aggressive disease requiring treatment. In addition to establishing infectivity, measurement of HBeAg can be used to assess the efficacy of treatment, with falling levels indicating success. It should be noted that there are some mutant strains of HBV found in Asia and the Middle East that do not produce HBeAg. HBV DNA levels can also be used to assess disease activity, with high levels indicating active disease and infectivity.

How well did you know this?
1
Not at all
2
3
4
5
Perfectly
32
Q

This serological marker, if high, would indicate low infectivity in a chronic hepatitis B carrier.

A.	 α-Fetoprotein
B.	 Alanine aminotransferase
C.	 Hepatitis B core antibody (HBcAb)
D.	 Hepatitis B core antigen (HBcAg)
E.	 Hepatitis B e antigen (HBeAg)
F.	 Hepatitis B surface antibody (HBsAb)
G.	 Hepatitis B surface antigen (HBsAg)
H.	 Hepatitis B RNA titre
A

C – Hepatitis B core antibody (HBcAb)

HBcAb can be present in acute and chronic infection. They are formed against the HBcAg, which is found only within the liver and therefore cannot be detected using serology. In acute infection, it is the second marker to be detected, after HBsAb, and is usually seen after 4 weeks of infection. In chronic disease, the presence of high titres of HBcAb, in the absence of HBeAg, indicates low infectivity and disease activity. This serological picture is associated with a better prognosis, with fewer individuals progressing to cirrhosis and hepatocellular carcinoma.

How well did you know this?
1
Not at all
2
3
4
5
Perfectly
33
Q

A 62-year-old woman presents with intermittent scalp pain. She says that the pain is specifically on the left side at the front and is worse when touching the area. She has no medical history. She is very tender over the affected area. Examination is otherwise unremarkable.

A.	 Cluster headache
B.	 Coitus-induced headache
C.	 Giant cell arteritis
D.	 Ice-cream headache
E.	 Idiopathic intracranial hypertension
F.	 Meningitis
G.	 Migraine
H.	 Sagittal sinus thrombosis
I.	 Sinusitis
J.	 Space-occupying lesion
K.	 Subarachnoid haemorrhage
L.	 Tension headache
M.	Trigeminal neuralgia
A

M – Trigeminal neuralgia

Trigeminal neuralgia describes sharp, stabbing pains in the second and third divisions of the trigeminal nerve. The pain is severe, brief and repetitive, often causing the patient to flinch. (This feature gives rise to the French name for the condition ‘tic douloureux’ – painful twitch). The pain may often have triggers, such as touching or eating. Trigeminal neuralgia is most common in older women, and the condition has a tendency to improve and relapse. The underlying cause may be compression of the trigeminal nerve rootlets at their entry into the brain stem by aberrant loops of cerebellar arteries. Management options include carbamazepine, phenytoin or gabapentin, which can improve the neuropathic pain.

How well did you know this?
1
Not at all
2
3
4
5
Perfectly
34
Q

A 56-year-old woman complains of a hoarse voice and dry cough following a thyroidectomy. She is otherwise well

A.	 Agranulocytosis
B.	 Angina
C.	 Bilateral recurrent laryngeal nerve injury
D.	 Bronchoconstriction
E.	 Hyperthyroid storm
F.	 Hypocalcaemia
G.	 Hypothyroidism
H.	 Laryngeal oedema
I.	 Malignancy
J.	 Unilateral recurrent laryngeal nerve injury
A

J – Unilateral recurrent laryngeal nerve palsy

The recurrent laryngeal nerve can be damaged during thyroidectomy via a number of mechanisms, including transection, infarction, diathermy burns and accidental ligation. Unilateral recurrent laryngeal nerve injury usually presents in the days following surgery with a hoarse voice and dyspnoea. In some cases, the symptoms will disappear after a few months. For those who do not recover, a number of surgical interventions are available. In total thyroidectomy, both laryngeal nerves can be damaged. This is a potentially life-threatening complication, since both vocal cords are paralysed, resulting in upper airway obstruction at extubation. If this is the case, the patient should be re-intubated and have their neck explored to assess the extent of any damage. If extubation continues to fail, the patient will require a tracheostomy. In the long term, surgical correction may allow the airway to be maintained without tracheostomy, but the voice will not recover.

How well did you know this?
1
Not at all
2
3
4
5
Perfectly
35
Q

A 45-year-old woman presents to her GP with lower abdominal pain, frequency of urination and pain on passing urine. She describes her urine as dark and offensive. The urine dipstick shows leucocytes, nitrites and blood.

A.	 Acute bacterial prostatitis
B.	 Acute pyelonephritis
C.	 Bacterial epididymo-orchitis
D.	 Balanitis
E.	 Chronic prostatitis
F.	 Chronic pyelonephritis
G.	 Escherichia coli cystitis
H.	 Mumps epididymo-orchitis
I.	 Proteus mirabilis urinary tract infection
J.	 Renal tuberculosis
K.	 Testicular torsion
L.	 Testicular trauma
M.	Urinary tract obstruction
A

G – Escherichia coli cystitis

Cystitis usually presents with dysuria, frequency of urine, suprapubic discomfort and fever. Cystitis is much more common in women than in men due to the shorter urethra in women. Escherichia coli, usually found in the bowel, is the most common pathogen responsible for uncomplicated cystitis in the UK. Cystitis can be confirmed by the presence of leucocytes, nitrites and blood in the urine. A midstream urine (MSU) sample should be sent to microbiology for microscopy, culture and sensitivity. A positive culture is when more than 105 of a single organism are grown per millilitre of urine. The growth of mixed bacterial species usually indicates contamination of the sample. Antibiotic treatment should be guided by local patterns of bacterial resistance and MSU sensitivity results. Uncomplicated cystitis is usually treated with a 3- to 5-day course of trimethoprim. Other antibiotics that are used to treat cystitis include amoxicillin, ciprofloxacin, cefradine and nitrofurantoin. Patients who are susceptible to recurrent urinary tract infection can be prescribed prophylactic antibiotics (e.g. 100 mg of trimethoprim once nightly). Patients should be advised to drink lots of fluids and avoid dehydration. Urinary tract infections in men and children require further investigation

How well did you know this?
1
Not at all
2
3
4
5
Perfectly
36
Q

A 34-year-old woman has had frequent episodes of lower abdominal pain, urinary frequency and nocturia over the past few years. On all these occasions, the urine dipstick showed nitrites, blood and leucocytes. Today, she developed sudden-onset, severe right-sided loin pain and macroscopic haematuria. An X-ray on admission to hospital confirmed the presence of a ureteric calculus.

A.	 Acute bacterial prostatitis
B.	 Acute pyelonephritis
C.	 Bacterial epididymo-orchitis
D.	 Balanitis
E.	 Chronic prostatitis
F.	 Chronic pyelonephritis
G.	 Escherichia coli cystitis
H.	 Mumps epididymo-orchitis
I.	 Proteus mirabilis urinary tract infection
J.	 Renal tuberculosis
K.	 Testicular torsion
L.	 Testicular trauma
M.	Urinary tract obstruction
A

I – Proteus mirabilis urinary tract infection

Proteus mirabilis is a Gram-negative anaerobic pathogen that can cause urinary tract infection. In addition, P. mirabilis infection causes the urine to become more alkaline, which predisposes to the development of urinary calculi. P. mirabilis infection is more common in patients with structural abnormalities of their urinary tract and in patients with long-term catheters.

How well did you know this?
1
Not at all
2
3
4
5
Perfectly
37
Q

A 68-year-old man is admitted to the emergency department with reduced consciousness. He has recently complained to his wife about passing large amounts of urine and excessive thirst. Investigation show a plasma blood glucose of 31.0 mmol/L, a potassium of 3.8 mmol/L, a sodium of 150 mmol/L, a urea of 16 mmol/L and a creatinine of 68 µmol/L.

A. Decrease insulin dose temporarily
B. Increase insulin dose temporarily
C. Intravenous bicarbonate
D. Intravenous glucose
E. Intravenous glucagon
F. Intravenous insulin with fluid resuscitation
G. Oral glucose
H. Rectal diazepam
I. Subcutaneous insulin with fluid resuscitation
J. Urgent referral to an endocrinologist

A

F – Intravenous insulin with fluid resuscitation

This patient has entered a hyperosmolar non-ketotic state (HONK). HONK is seen in people with type 2 diabetes, including previously undiagnosed patients. The event is usually triggered by an underlying illness, the ingestion of a high sugar load or the use of medications that cause a rise in plasma glucose (e.g. steroids). Patients usually present in a confused state, with a history of polyuria and polydipsia. The patient has a high plasma glucose, which can be in excess of 30 mmol/L. In HONK, the circulating insulin is sufficient to prevent ketogenesis but insufficient to allow the peripheral uptake and metabolism of glucose. Therefore the patient does not become acidotic. The hyperglycaemia causes an osmotic diuresis that rapidly dehydrates the patient and produces an extremely high plasma osmolality.

HONK is treated with a combination of intravenous fluid resuscitation and intravenous insulin. Care must be taken not to reduce the glucose levels or sodium concentration too rapidly, as the resultant osmotic shifts can result in fatal cerebral oedema. The insulin should therefore be started at a low rate (e.g. 3 units/h) and only isotonic (0.9%) saline should be prescribed. HONK also places the patient at high risk of thromboembolic events such as deep vein thrombosis, pulmonary embolism and stroke; therefore all patients should receive prophylactic heparin. The mortality rate of HONK approaches 30%, which is likely to be a reflection of the advanced age of many of the patients and the presence of underlying illness.

How well did you know this?
1
Not at all
2
3
4
5
Perfectly
38
Q

The mother of a 14-year-old type 1 diabetic boy phones the GP’s surgery asking for advice. Her son has a chest infection and is not eating. She is worried about his blood sugar levels.

A. Decrease insulin dose temporarily
B. Increase insulin dose temporarily
C. Intravenous bicarbonate
D. Intravenous glucose
E. Intravenous glucagon
F. Intravenous insulin with fluid resuscitation
G. Oral glucose
H. Rectal diazepam
I. Subcutaneous insulin with fluid resuscitation
J. Urgent referral to an endocrinologist

A

B – Increase insulin dose temporarily

Although this case does not represent a true diabetic emergency, the patient is at risk of deteriorating. As discussed above in Case 1, in the presence of illness, the patient’s insulin requirements increase even if they are not eating. If the insulin dose is lowered or remains the same, the patient may become ketoacidotic. The patient in this scenario should therefore be advised to increase his insulin dose temporarily and seek medical attention. Regular capillary blood glucose measurement is essential until an improvement is seen.

How well did you know this?
1
Not at all
2
3
4
5
Perfectly
39
Q

This dermatome supplies the back of the head

A.	 C1
B.	 C2
C.	 C4
D.	 C6
E.	 C7
F.	 C8
G.	 T4
H.	 T6
I.	 T10
J.	 L1
K.	 L3
L.	 L5
M.	 S1
N.	 S3
O.	 S5
P.	 Trigeminal nerve
A

B – C2

How well did you know this?
1
Not at all
2
3
4
5
Perfectly
40
Q

This dermatome supplies the perianal area.

A.	 C1
B.	 C2
C.	 C4
D.	 C6
E.	 C7
F.	 C8
G.	 T4
H.	 T6
I.	 T10
J.	 L1
K.	 L3
L.	 L5
M.	 S1
N.	 S3
O.	 S5
P.	 Trigeminal nerve
A

O – S5

How well did you know this?
1
Not at all
2
3
4
5
Perfectly
41
Q

A 56-year-old man presents to the GP with a 6-month history of worsening shortness of breath. Examination reveals bilateral endinspiratory crackles and clubbing. A chest X-ray shows reticulonodular shadowing, especially in the lower zones.

A.	 Acute coryza
B.	 Asbestosis
C.	 Aspergillosis
D.	 Bronchiectasis
E.	 Chronic obstructive pulmonary disease
F.	 Cryptogenic fibrosing alveolitis
G.	 Extrinsic allergic alveolitis
H.	 Pneumoconiosis
I.	 Sarcoidosis
J.	 Tuberculosis
A

F – Cryptogenic fibrosing alveolitis

Cryptogenic fibrosing alveolitis (CFA) is a fibrosing alveolitis that is not associated with another disease. It is most common in older male smokers. CFA presents with exertional dyspnoea and a dry cough. Examination may reveal clubbing and bilateral end-inspiratory crepitations, especially at the lower zones. As disease progresses, central cyanosis and cor pulmonale develop. Chest X-ray demonstrates diffuse pulmonary opacities, or honeycombing in advanced disease. Pulmonary function tests show a restrictive defect (with a reduced vital capacity and reduced forced expiratory volume). The diagnosis is confirmed with high-resolution CT. Treatment is with prednisolone and azathioprine, although CFA tends to respond poorly. Death usually occurs within 5 years of diagnosis.

How well did you know this?
1
Not at all
2
3
4
5
Perfectly
42
Q

A 47-year-old man presents to the GP with a 4-month history of worsening shortness of breath. Examination reveals bilateral endinspiratory crackles. A chest X-ray shows reticulonodular shadowing, especially in the upper zones.

A.	 Acute coryza
B.	 Asbestosis
C.	 Aspergillosis
D.	 Bronchiectasis
E.	 Chronic obstructive pulmonary disease
F.	 Cryptogenic fibrosing alveolitis
G.	 Extrinsic allergic alveolitis
H.	 Pneumoconiosis
I.	 Sarcoidosis
J.	 Tuberculosis
A

G – Extrinsic allergic alveolitis

Extrinsic allergic alveolitis is caused by the inhalation of organic dusts, resulting in an immune complex-mediated reaction with the formation of granulomas within the lung (and eventual lung fibrosis). Patients present with fever, muscle aches, cough, dyspnoea (no wheeze) and headaches. Examination reveals widespread end-inspiratory crepitations. Chest X-ray demonstrates micronodular shadowing, especially in the upper zones. Lung function tests confirm a restrictive defect. The diagnosis of extrinsic allergic alveolitis is made by demonstrating typical clinical and radiological features along with identifying the underlying allergen. Treatment is with prednisolone and requires avoidance of the precipitating allergen. Examples of allergens that cause extrinsic allergic alveolitis include:

  • farmer’s lung: Aspergillus fumigatus in mouldy hay
  • bird fancier’s lung: avium serum proteins found in bird droppings and feathers
  • malt worker’s lung: A. clavatus in mouldy maltings.
How well did you know this?
1
Not at all
2
3
4
5
Perfectly
43
Q

A 32-year-old man has developed multiple painful ulcers on his penis, with an associated phimosis. On examination, there is inguinal lymph node enlargement and a discharging sinus. A culture demonstrates Haemophilus ducreyi.

A.	 Bacterial vaginosis
B.	 Cervical cancer
C.	 Chancroid
D.	 Chlamydia infection
E.	 Epstein–Barr virus
F.	 Genital candidiasis
G.	 Genital herpes
H.	 Genital warts
I.	 Gonorrhoea
J.	 Granuloma inguinale (donovanosis)
K.	 HIV
L.	 Lymphogranuloma venereum
M.	Molluscum contagiosum
N.	 Phthiriasis
O.	 Reiter’s syndrome
P.	 Scabies
Q.	 Syphilis
R.	 Trichomoniasis
A

C – Chancroid

Chancroid is caused by the Gram-negative bacterium Haemophilus ducreyi, and is found mostly in tropical countries. It is an ulcerative condition of the genitalia that develops within a week of exposure. Lesions begin as a small papule, which eventually ulcerates to form single or multiple, painful, superficial ulcers. Inflammation may lead to phimosis. Enlargement and suppuration of inguinal lymph nodes may occur, leading to a unilocular abscess (bubo) that can rupture to form a discharging sinus. Treatment is with appropriate antibiotics (e.g. erythromycin).

How well did you know this?
1
Not at all
2
3
4
5
Perfectly
44
Q

A 21-year-old female student presents with abdominal pain in the right upper quadrant. She has had multiple casual sexual partners with whom she used no barrier protection. When questioned further, she admits to having a burning sensation on passing urine.

A.	 Bacterial vaginosis
B.	 Cervical cancer
C.	 Chancroid
D.	 Chlamydia infection
E.	 Epstein–Barr virus
F.	 Genital candidiasis
G.	 Genital herpes
H.	 Genital warts
I.	 Gonorrhoea
J.	 Granuloma inguinale (donovanosis)
K.	 HIV
L.	 Lymphogranuloma venereum
M.	Molluscum contagiosum
N.	 Phthiriasis
O.	 Reiter’s syndrome
P.	 Scabies
Q.	 Syphilis
R.	 Trichomoniasis
A

D – Chlamydia infection

The right upper quadrant pain with non-specific urethritis in a girl with this history suggests chlamydial infection. Chlamydia infection is caused by the oculogenital serovars D–K of Chlamydia trachomatis. Infection tends to be asymptomatic, although there can be increased vaginal discharge, dysuria and urinary frequency. Ascending infection can cause salpingitis and, if it enters the abdominal cavity, perihepatitis (Fitz-Hugh–Curtis syndrome), which leads to right upper quadrant pain and tenderness. Chlamydia infection is a major cause of infertility and increases the possibility of a future ectopic pregnancy. In males, symptoms include mucopurulent discharge and dysuria, although it is asymptomatic in 25%. Epididymo-orchitis is a complication. Diagnosis of chlamydial infection is by urine antigen detection or vaginal swab culture. Treatment is with doxycycline.

Acute pelvic inflammatory disease is characterized by pelvic pain, pyrexia, cervical excitation, adnexal tenderness and a raised white cell count. It is most often caused by chlamydia infection and gonorrhoea.

How well did you know this?
1
Not at all
2
3
4
5
Perfectly
45
Q

A 75-year-old woman presents with sudden-onset pain in her right knee. On examination, the knee is swollen, red and painful. She has a temperature of 36.8°C. An X-ray of the knee shows a bright line on the surface of the meniscus

A.	 Ankylosing spondylitis
B.	 Enteropathic arthritis
C.	 Golfer’s elbow
D.	 Gout
E.	 Osteoarthritis
F.	 Pseudogout
G.	 Psoriatic arthritis
H.	 Reiter’s syndrome
I.	 Rheumatoid arthritis
J.	 Septic arthritis
K.	 Still’s disease
L.	 Tennis elbow
A

F – Pseudogout

Calcium pyrophosphate deposition disease is caused by the deposition of calcium pyrophosphate dihydrate crystals in the joints. The crystals deposit along the cartilages, producing a linear chondrocalcinosis that is best seen on the meniscus on X-ray. Shedding of crystals into the joint precipitates an acute synovitis known as pseudogout. Patients present with acute-onset pain, swelling and redness in the affected joint. Pseudogout is most common in elderly women and usually affects the knee or wrist. The diagnosis is made by joint aspiration, demonstrating the weakly positively birefringent brick-shaped crystals of calcium pyrophosphate. The joint fluid should always be sent for culture to exclude a septic arthritis. The treatment of pseudogout is with nonsteroidal anti-inflammatory drugs (NSAIDs). The chondrocalcinosis itself cannot be treated.

How well did you know this?
1
Not at all
2
3
4
5
Perfectly
46
Q

A 30-year-old man presents with a 2-day history of left knee pain and a burning sensation on passing urine. On examination, his temperature is 37.2°C and you notice that his eyes are red.

A.	 Ankylosing spondylitis
B.	 Enteropathic arthritis
C.	 Golfer’s elbow
D.	 Gout
E.	 Osteoarthritis
F.	 Pseudogout
G.	 Psoriatic arthritis
H.	 Reiter’s syndrome
I.	 Rheumatoid arthritis
J.	 Septic arthritis
K.	 Still’s disease
L.	 Tennis elbow
A

H – Reiter’s syndrome

The triad of arthritis, urethritis and iritis is indicative of Reiter’s syndrome.

How well did you know this?
1
Not at all
2
3
4
5
Perfectly
47
Q

A 6-month-old baby boy is brought to see the doctor by his mother. She is worried that he is becoming increasing floppy and can no longer lift his head. He is also finding it difficult to bottle-feed. On examination, although the baby lies flat and does not move much, he is obviously alert.

A.	 Ataxia telangiectasia
B.	 Charcot–Marie–Tooth disease
C.	 Guillain–Barré syndrome
D.	 Hypokalaemia
E.	 Lambert–Eaton syndrome
F.	 Mononeuritis multiplex
G.	 Motor neuron disease
H.	 Multiple sclerosis
I.	 Muscular dystrophy
J.	 Myasthenia gravis
K.	 Poliomyelitis
L.	 Spinal cord compression
M.	 Spinal muscular atrophy
A

M – Spinal muscular atrophy

Spinal muscular atrophy describes a group of genetic disorders affecting spinal and cranial motor neurons. There is degeneration of anterior horn cells, resulting in progressive proximal and distal weakness, wasting, and fasciculation (usually symmetrical). An example is Werdnig–Hoffmann disease: autosomal recessive severe muscle weakness, with death occurring by 12 months. Spinal muscular atrophy is the second most common cause of neuromuscular disease in the UK

How well did you know this?
1
Not at all
2
3
4
5
Perfectly
48
Q

A 6-year-old boy is brought in by his mother as he has had progressive difficulty walking over the last few months. He especially finds it difficult to stand up after he has been playing on the floor. On examination, his calves appear large. There is no sensory deficit.

A.	 Ataxia telangiectasia
B.	 Charcot–Marie–Tooth disease
C.	 Guillain–Barré syndrome
D.	 Hypokalaemia
E.	 Lambert–Eaton syndrome
F.	 Mononeuritis multiplex
G.	 Motor neuron disease
H.	 Multiple sclerosis
I.	 Muscular dystrophy
J.	 Myasthenia gravis
K.	 Poliomyelitis
L.	 Spinal cord compression
M.	 Spinal muscular atrophy
A

I - Muscular Dystrophy

Muscular dystrophy describes a group of inherited disorders with progressive degeneration of groups of muscles without involvement of the nervous system. It is characterized by symmetrical wasting and weakness, with no fasciculations or sensory loss. The most common muscular dystrophy is the Duchenne type, an X-linked recessive deficiency of the protein dystrophin, which is required to maintain the integrity of muscle cell walls. Duchenne muscular dystrophy begins in childhood and affects the proximal arms and legs. There is pseudohypertrophy of the calves (due to replacement of muscle by fibrosis and fat), a waddling gait and difficultly standing (children may ‘climb’ up their legs to help stand up – Gower’s sign). Death by respiratory failure or cardiomyopathy occurs in the early 20s. A diagnosis of muscular dystrophy is confirmed by electromyography and biopsy. Creatine kinase may also be elevated. Management of muscular dystrophy is supportive.

Other forms of muscular dystrophy include Becker muscular dystrophy (Xlinked recessive, with features similar to Duchenne muscular dystrophy but less severe, and with death occurring by the early 40s), myotonic dystrophy (autosomal dominant, weakness of temporal, facial, sternomastoid and distal limb muscles), fascioscapulohumeral dystrophy (autosomal dominant, affecting facial and shoulder girdle muscles) and limb girdle dystrophy (autosomal recessive, affecting pelvic and shoulder girdles).

How well did you know this?
1
Not at all
2
3
4
5
Perfectly
49
Q

A 47-year-old woman presents with difficulty writing and walking. Over the last week, she has had numbness and weakness of the first three digits of the right hand. She is on insulin for type 2 diabetes mellitus. On examination, there is a left-sided foot drop.

A.	 Ataxia telangiectasia
B.	 Charcot–Marie–Tooth disease
C.	 Guillain–Barré syndrome
D.	 Hypokalaemia
E.	 Lambert–Eaton syndrome
F.	 Mononeuritis multiplex
G.	 Motor neuron disease
H.	 Multiple sclerosis
I.	 Muscular dystrophy
J.	 Myasthenia gravis
K.	 Poliomyelitis
L.	 Spinal cord compression
M.	 Spinal muscular atrophy
A

F – Mononeuritis multiplex

Mononeuritis multiplex is a form of peripheral neuropathy where there is damage to at least two anatomically distinct peripheral or spinal nerves. There may be a variety of underlying pathologies, such as ischaemia or inflammation of the nerves. Common causes of mononeuritis multiplex include diabetes mellitus, polyarteritis nodosa, rheumatoid arthritis and systemic lupus erythematosus.

How well did you know this?
1
Not at all
2
3
4
5
Perfectly
50
Q

A 50-year-old man with type 2 diabetes is found to have retinal microaneurysms, haemorrhages and hard exudates on fundoscopy. He claims to have no visual impairment

A.	 Background retinopathy
B.	 Cataract
C.	 Glaucoma
D.	 Hypertensive retinopathy
E.	 Macular degeneration
F.	 Macular oedema
G.	 Pre-proliferative retinopathy
H.	 Proliferative retinopathy
I.	 Retinal detachment
J.	 Vitreous haemorrhage
A

A – Background retinopathy

Diabetic eye disease is a serious microvascular complication of diabetes, and is the leading cause of blindness in the under-60 age group in the developed world. Background retinopathy is asymptomatic and is usually detected during the screening of diabetic patients. The pathological processes leading to retinopathy relate to increased retinal arterial blood flow secondary to hyperglycaemia, which results in the formation of microaneurysms, increased capillary permeability and retinal ischaemia. The pathognomonic changes of background retinopathy are blot haemorrhages, capillary microaneurysms (dots) and hard exudates, which appear as yellow plaques and contain lipid deposits. If a patient is shown to have background retinopathy, they must receive annual fundoscopy or diabetic photography to assess disease progression. If there is concern that disease is progressing, the patient should be reviewed again after 3 months. Each patient must undergo a holistic review of their glycaemic control, blood pressure, lipid profile and lifestyle. If the glycaemic control, hypertension and hyperlipidaemia are not corrected, the background retinopathy can progress to pre-proliferative and proliferative retinopathy, which is associated with a significant risk of blindness, vitreous haemorrhage and retinal detachment.

How well did you know this?
1
Not at all
2
3
4
5
Perfectly
51
Q

A 67-year-old woman with a long history of poorly controlled type 2 diabetes is having problems with her sight. Examination reveals reduced visual acuity. Fundoscopy demonstrates evidence of background retinopathy but no other pathology.

A.	 Background retinopathy
B.	 Cataract
C.	 Glaucoma
D.	 Hypertensive retinopathy
E.	 Macular degeneration
F.	 Macular oedema
G.	 Pre-proliferative retinopathy
H.	 Proliferative retinopathy
I.	 Retinal detachment
J.	 Vitreous haemorrhage
A

F – Macular oedema

Diabetic maculopathy is most commonly seen in older patients with type 2 diabetes and is the leading cause of blindness in this group. Macular oedema cannot be seen on fundoscopy, but must always be suspected if the patient complains of deteriorating vision or if there is objective evidence of deteriorating visual acuity (i.e. using a Snellen chart). In addition to macular oedema, the macula can be affected by retinopathy, including the presence of haemorrhages, exudates and neovascularization. If maculopathy is suspected, the patient should be referred to an ophthalmologist for assessment and possible retinal photocoagulation therapy.

How well did you know this?
1
Not at all
2
3
4
5
Perfectly
52
Q

A 47-year-old man with type 1 diabetes presents to the emergency department with a sudden deterioration in the vision of his left eye associated with flashes and objects floating across his visual field. He describes the visual loss as a curtain moving across his visual field.

A.	 Background retinopathy
B.	 Cataract
C.	 Glaucoma
D.	 Hypertensive retinopathy
E.	 Macular degeneration
F.	 Macular oedema
G.	 Pre-proliferative retinopathy
H.	 Proliferative retinopathy
I.	 Retinal detachment
J.	 Vitreous haemorrhage
A

I – Retinal detachment

Retinal detachment occurs when the outer pigment epithelium detaches from the inner retina, which contains the neurons. In patients with proliferative retinopathy, the retina detaches as a result of traction caused by the formation and deposition of fibrous bands. Patients complain of visual floaters, reduced visual acuity and visual field loss (often described as a curtain passing across their visual field). If the macula becomes detached, which usually occurs when the upper half of the retina detaches, the patient will suffer permanent damage to their central vision. On fundoscopy, the retina often appears grey and lifeless and bulges forward. If retinal detachment is suspected, the patient should be referred for urgent ophthalmological review.

How well did you know this?
1
Not at all
2
3
4
5
Perfectly
53
Q

You are asked to see a 68-year-old woman who had an anterior myocardial infarction 2 days ago. She has become acutely short of breath and is coughing up pink frothy sputum. On examination, her jugular venous pressure is raised and she has a systolic murmur. On auscultation, you can hear bilateral crackles at the lung bases.

A.	 First-degree heart block
B.	 Acute mitral valve regurgitation
C.	 Acute pericarditis
D.	 Atrial ectopic beats
E.	 Atrial fibrillation
F.	 Cardiac tamponade
G.	 Complete (third-degree) heart block
H.	 Dressler’s syndrome
I.	 Pulmonary embolism
J.	 Subacute left ventricular failure
K.	 Ventricular aneurysm
L.	 Ventricular ectopic beats
M.	Ventricular tachycardia	
N.	 Ventricular wall rupture
A

B – Acute mitral valve regurgitation

Acute mitral regurgitation is usually seen 2–10 days after an inferior myocardial infarction (MI) and is often fatal. It is usually secondary to rupture of a papillary muscle or the chordae tendineae. It presents with symptoms and signs of acute left ventricular failure, including shortness of breath and the production of pink frothy sputum. On auscultation, there is a loud pansystolic murmur best herd at the apex that radiates into the axilla. The patient is usually haemodynamically compromised, with severe hypotension, low oxygen saturation and poor urine output. If the patient survives the initial period, a surgical opinion should be sought regarding possible repair/valve replacement.

How well did you know this?
1
Not at all
2
3
4
5
Perfectly
54
Q

A 66-year-old man, known to have had an anterior myocardial infarction 2 days previously, collapses. On examination, you notice that he has a raised jugular venous pressure that rises with inspiration. Auscultation seems normal. His blood pressure is 80/50 mmHg and his heart rate is 100 beats/min. The ECG shows sinus tachycardia with small QRS complexes.

A.	 First-degree heart block
B.	 Acute mitral valve regurgitation
C.	 Acute pericarditis
D.	 Atrial ectopic beats
E.	 Atrial fibrillation
F.	 Cardiac tamponade
G.	 Complete (third-degree) heart block
H.	 Dressler’s syndrome
I.	 Pulmonary embolism
J.	 Subacute left ventricular failure
K.	 Ventricular aneurysm
L.	 Ventricular ectopic beats
M.	Ventricular tachycardia	
N.	 Ventricular wall rupture
A

F – Cardiac tamponade

In cardiac tamponade, there is rapid accumulation of fluid within the pericardial space, which compresses the heart, causing pump failure and cardiogenic shock. The patient is usually hypotensive, with distended neck veins and quiet heart sounds – features described by Beck’s triad (not to be confused with Beck’s cognitive triad: negative view of oneself, negative interpretation of past experiences and negative expectation of the future). Cardiac tamponade can be confirmed by echocardiography, but the rapidly fatal cardiogenic shock usually necessitates a clinical diagnosis and immediate treatment with pericardial aspiration by pericardiocentesis.

How well did you know this?
1
Not at all
2
3
4
5
Perfectly
55
Q

A 37-year-old woman presents with a bright-red nodule on the end of her index finger that bleeds easily. This lesion has grown rapidly in the last week and is now 1 cm in diameter

A.	 Cellulitis
B.	 Erysipelas
C.	 Ganglion
D.	 Granuloma annulare
E.	 Impetigo
F.	 Kaposi’s sarcoma
G.	 Necrotizing fasciitis
H.	 Neurofibroma
I.	 Pyogenic granuloma
J.	 Sebaceous cyst
K.	 Seborrhoeic keratosis
A

I – Pyogenic granuloma

A pyogenic granuloma is an acquired haemangioma (note that it is neither pyogenic nor a granuloma!) that occurs most often on the head, trunk, hands and feet. It develops at a site of trauma (e.g. a thorn prick) as a bright-red nodule that bleeds easily and enlarges rapidly over 2–3 weeks. It affects the young and old extremes of age, but is most common in pregnant women. These lesions are benign and are managed by excision, although smaller lesions may resolve spontaneously.

How well did you know this?
1
Not at all
2
3
4
5
Perfectly
56
Q

A 48-year-old woman attends the emergency department with pain and a rash on her left ear. This is associated with dizziness, vertigo and weakness of the left side of her face.

A.	 Benign paroxysmal positional vertigo
B.	 Drug-induced ototoxicity
C.	 Hyperventilation
D.	 Labyrinthitis
E.	 Ménière’s disease
F.	 Multiple sclerosis
G.	 Ramsay Hunt syndrome
H.	 Posterior circulation stroke
I.	 Trauma
A

G – Ramsay Hunt syndrome

Ramsay Hunt syndrome is caused by herpes zoster infection of the geniculate ganglion, affecting the facial nerve and sometimes the vestibulocochlear nerve, which is in close proximity. Features include paralysis of the facial muscles on the affected side, with a herpetic eruption on the ear canal. Patients may also suffer with tinnitus, hearing loss and vertigo. Oral aciclovir improves the prognosis and reduces the risk of post-herpetic neuralgia.

How well did you know this?
1
Not at all
2
3
4
5
Perfectly
57
Q

A 58-year-old man woke up in the night with sudden-onset severe dizziness and nausea. He got up, but found it difficult to walk down the stairs because of the dizziness. The symptoms were no worse in any particular position and there was no hearing loss.

A.	 Benign paroxysmal positional vertigo
B.	 Drug-induced ototoxicity
C.	 Hyperventilation
D.	 Labyrinthitis
E.	 Ménière’s disease
F.	 Multiple sclerosis
G.	 Ramsay Hunt syndrome
H.	 Posterior circulation stroke
I.	 Trauma
A

D – Labyrinthitis

Labyrinthitis (also known as vestibular neuronitis) is the most common cause of vertigo, and may be viral in origin. Patients present with explosive severe vertigo, vomiting and ataxia (without tinnitus and deafness). Symptoms settle down over a few days. Management is with antiemetics, often prochlorperazine.

How well did you know this?
1
Not at all
2
3
4
5
Perfectly
58
Q

A 36-year-old man presents with a 4-month history of worsening lower back pain and stiffness. The pain is worse in the morning, but gets better throughout the day. Examination is unremarkable.

A.	 Ankylosing spondylitis
B.	 Enteropathic arthritis
C.	 Golfer’s elbow
D.	 Gout
E.	 Osteoarthritis
F.	 Pseudogout
G.	 Psoriatic arthritis
H.	 Reiter’s syndrome
I.	 Rheumatoid arthritis
J.	 Septic arthritis
K.	 Still’s disease
L.	 Tennis elbow
A

A – Ankylosing spondylitis

Ankylosing spondylitis (AS) is an inflammatory disorder of the back that is more common in males. Patients present with insidious-onset lower back pain and stiffness that is worse in the morning and gets better with exercise. There is poor spinal flexion, and in severe cases patients develop a rigid lower spine with a hunch (known as the ‘question mark posture’ or ‘hang dog posture’). AS may also affect the large joints asymmetrically. AS is associated with a number of extra-articular features, which are remembered by the ‘five As’: Apical lung fibrosis, Anterior uveitis, Achilles’ tendonitis/plantar fasciitis, Aortic regurgitation and Amyloidosis.

This diagnosis of AS is made using Schober’s test: two fingers are placed 10 cm apart on the lower back of the patient (5 cm above and below the L5 vertebra in the midline), and the patient is asked to flex. An increase between the fingers of <5 cm indicates spinal stiffness. X-ray of the hip shows blurred margins of the sacroiliac joints (sacroiliitis). Characteristic radiological features of the spine in AS include erosion of the corners of the vertebral bodies (Romanus lesions), the development of bony spurs (syndesmophytes) and calcification of the spinal ligaments (bamboo spine). Treatment options in AS are physiotherapy, exercise and slow-release NSAIDs, e.g. indometacin. Most patients manage to lead a normal life, although severe cases may impair ventilation.

AS is an example of a seronegative spondyloarthropathy, i.e. a disease associated with HLA-B27 that is characterized by a lack of rheumatoid factor (hence ‘seronegative’). Other spondyloarthropathies are psoriatic arthritis, Reiter’s disease and enteropathic arthritis.

How well did you know this?
1
Not at all
2
3
4
5
Perfectly
59
Q

A 3-year-old girl presents with a 2-week history of pain in her joints and muscles. She feels very unwell and is difficult to settle. On examination, she has a raised temperature.

A.	 Ankylosing spondylitis
B.	 Enteropathic arthritis
C.	 Golfer’s elbow
D.	 Gout
E.	 Osteoarthritis
F.	 Pseudogout
G.	 Psoriatic arthritis
H.	 Reiter’s syndrome
I.	 Rheumatoid arthritis
J.	 Septic arthritis
K.	 Still’s disease
L.	 Tennis elbow
A

K – Still’s disease

Juvenile idiopathic arthritis (JIA) was previously referred to as juvenile chronic arthritis or juvenile rheumatoid arthritis. JIA is defined as joint inflammation persisting for 6 weeks or more, with initial onset in a person under 16 years of age, in the absence of another specific cause.

Pauciarticular JIA usually occurs in younger children, affecting the knees, ankles and wrists most commonly. Polyarticular JIA is more common in girls of all ages, usually symmetrically involving the hands and wrists. Complications of JIA include chronic anterior uveitis, flexion contraction of the joints and amyloidosis.

Still’s disease is a systemic form of juvenile arthritis that is thought to be an autoimmune disorder. It usually begins at the age of 3–4 years and is more common in girls. Features of Still’s disease include intermittent high pyrexia and a salmon-pink rash, with aches and pains of the joints and muscles. Other features are hepatosplenomegaly, lymphadenopathy and pericarditis. Inflammatory markers such as C-reactive protein are raised; however, antinuclear antibody and rheumatoid factor are usually negative. Management options include physiotherapy, resting splints, NSAIDs, disease-modifying drugs (e.g. methotrexate and ciclosporin) and steroids. The younger the age of onset of Still’s disease, the worse the prognosis.

How well did you know this?
1
Not at all
2
3
4
5
Perfectly
60
Q

Aspirin used to treat a 5-year-old boy with fever.

A.	 Agranulocytosis
B.	 Bronchospasm
C.	 Dry cough
D.	 Dry mouth
E.	 Nausea and vomiting
F.	 Peripheral oedema
G.	 Oculogyric crisis
H.	 Reye’s syndrome
I.	 Vomiting after alcohol
A

H – Reye’s syndrome

Reye’s syndrome is associated with aspirin use in children, and causes fatty infiltration of the liver and severe encephalopathy. Children can initially present with a mild illness, but often progress to coma. Treatment is directed at managing complications such as raised intracranial pressure and hypoglycaemia. This condition is fatal in up to 40% of cases. For this reason, aspirin should never be given to children.

How well did you know this?
1
Not at all
2
3
4
5
Perfectly
61
Q

A 26-year-old man presents with a 2-day history of feeling generally unwell and short of breath. Today, he started coughing up thick green sputum that had flecks of dry blood in. He complains of a sharp pain over the right side of his chest, which is worse on breathing in. Auscultation reveals bronchial breathing at the right base.

A.	 Acute coryza
B.	 Asthma
C.	 Bronchiectasis
D.	 Chronic obstructive pulmonary disease
E.	 Closed non-tension pneumothorax
F.	 Cryptogenic fibrosing alveolitis
G.	 Cystic fibrosis
H.	 Extrinsic allergic alveolitis
I.	 Open non-tension pneumothorax
J.	 Pneumoconiosis
K.	 Pneumonia
L.	 Pulmonary embolism
M.	 Sleep apnoea
N.	 Tension pneumothorax
A

K – Pneumonia

Pneumonia is an acute respiratory illness with recent radiological pulmonary shadowing. A chest infection without chest X-ray changes is known as a lower respiratory tract infection. Community-acquired pneumonias are spread by droplet inhalation, and present with cough, fever and pleuritic chest pain. Cough is initially dry, but later becomes productive and may be blood stained. Examination reveals bronchial breath sounds in the affected area (due to consolidation) and coarse crepitations. Severity of pneumonia is assessed using the CURB-65 score.

The most common pathogens involved in community-acquired pneumonia are Streptococcus pneumoniae (30%), Mycoplasma pneumoniae, Staphylococcus aureus and Chlamydophila pneumoniae (previously known as Chlamydia pneumoniae). The resulting infection may be lobar pneumonia (homogeneous consolidation of one or more lobes) or bronchopneumonia (patchy alveolar consolidation that commonly affects both lower lobes). Investigations include chest X-ray and sputum culture. Treatment is with relevant antibiotics. Empirical regimens include oral amoxicillin for uncomplicated cases and intravenous clarithromycin plus co-amoxiclav for severe disease. Seven to ten days of treatment is normally sufficient.

How well did you know this?
1
Not at all
2
3
4
5
Perfectly
62
Q

A 16-year-old boy with a transplanted kidney is shown to have worsening renal function over the previous 2 weeks. He is complaining of pain around the transplanted kidney. He has had no significant complications since his transplant 4 months ago. An ultrasound scan of his transplanted kidney showed no significant abnormalities.

A.	 Angiography
B.	 Cystoscopy
C.	 DMSA scan
D.	 End-stream urine sample for microscopy, culture and sensitivity
E.	 Kidney–ureter–bladder X-ray
F.	 Micturating cystourethrogram
G.	 Midstream urine sample for microscopy, culture and sensitivity
H.	 MRI
I.	 Renal biopsy
J.	 Renal tract ultrasound scan
K.	 Three early morning urine samples for microscopy, culture and Ziehl–Neelsen staining
L.	 Water deprivation test
A

I – Renal biopsy

Deteriorating renal function in a patient with a transplanted kidney should always be taken seriously, as there is a possibility of organ rejection. Other causes include vessel thrombosis, infection and recurrence of existing disease, e.g. immunoglobulin A (IgA) nephropathy. A percutaneous renal biopsy will reveal the presence, aetiology and severity of any rejection process. Acute rejection is usually treated with pulsed steroid therapy (methylprednisolone). In severe episodes of rejection, antithymocyte globulin, which destroys T lymphocytes, can be given to slow the rejection process. Chronic rejection is often due to the fibrosis of the microcirculation within the graft and is usually irreversible. Chronic rejection usually means that the patient is re-listed for transplantation.

How well did you know this?
1
Not at all
2
3
4
5
Perfectly
63
Q

A 68-year-old woman who is being investigated for altered bowel habit is found to have an Hb of 8.3 g/dL, an MCV of 67 fL, a low ferritin level and a raised total iron-binding capacity.

Α. α-Thalassaemia
B. β-Thalassaemia major
C. β-Thalassaemia trait
D. Anaemia of chronic disease
E. Autoimmune haemolytic anaemia
F. Glucose-6-phosphate dehydrogenase deficiency
G. Hereditary spherocytosis
H. Iron-deficiency anaemia secondary to gastrointestinal bleeding
I. Iron-deficiency anaemia secondary to iron malabsorption
J. Iron-deficiency anaemia secondary to menorrhagia
K. Megaloblastic macrocytic anaemia secondary to vitamin B12deficiency
L. Megaloblastic macrocytic anaemia secondary to folate deficiency
M. Megaloblastic microcytic anaemia secondary to vitamin B12 deficiency
N. Normal variant
O. Sickle cell anaemia

A

H – Iron-deficiency anaemia secondary to gastrointestinal bleeding

Iron-deficiency anaemia (IDA) is defined as a haemoglobin concentration below 13.5 g/dL in males or 11.5 g/dL in females, in association with a low MCV and evidence of depleted iron stores (i.e. a low ferritin and a raised total ironbinding capacity). In the developed world, IDA is usually secondary to chronic blood loss from gastrointestinal, uterine and urinary tract sources. (Worldwide, hookworm infection and schistosomiasis are common causes.) In cases where the source of bleeding is obvious, further investigation is usually not necessary and treatment can begin. However, in many instances, bleeding goes unnoticed and is secondary to a more sinister cause such as gastrointestinal malignancy. Therefore, patients with IDA without an obvious cause must be referred for investigation of the upper and lower gastrointestinal tracts in the first instance.

To treat IDA, the underlying cause must be corrected and iron stores replenished. The most appropriate method of replacing iron is with oral supplementation

How well did you know this?
1
Not at all
2
3
4
5
Perfectly
64
Q

A 24-year-old woman with active Crohn’s disease is shown to have an Hb of 9.5 g/dL, an MCV of 101 fL and a normal ferritin level. A peripheral blood film shows a number of hypersegmented neutrophils.

Α. α-Thalassaemia
B. β-Thalassaemia major
C. β-Thalassaemia trait
D. Anaemia of chronic disease
E. Autoimmune haemolytic anaemia
F. Glucose-6-phosphate dehydrogenase deficiency
G. Hereditary spherocytosis
H. Iron-deficiency anaemia secondary to gastrointestinal bleeding
I. Iron-deficiency anaemia secondary to iron malabsorption
J. Iron-deficiency anaemia secondary to menorrhagia
K. Megaloblastic macrocytic anaemia secondary to vitamin B12deficiency
L. Megaloblastic macrocytic anaemia secondary to folate deficiency
M. Megaloblastic microcytic anaemia secondary to vitamin B12 deficiency
N. Normal variant
O. Sickle cell anaemia

A

K – Megaloblastic macrocytic anaemia secondary to vitamin B12 deficiency

Vitamin B 12 is an essential water-soluble vitamin required for DNA synthesis and red blood cell production. It is absorbed in the terminal ileum only after binding intrinsic factor, which is secreted by gastric parietal cells. The liver is able to store approximately 1 mg of vitamin B12, which is sufficient for 3–4 years; hence vitamin B12 deficiency takes years to manifest. The main causes of vitamin B 12 deficiency are pernicious anaemia, poor dietary intake (vegans) and malabsorption secondary to disease of the terminal ileum. Pernicious anaemia describes the autoimmune loss of parietal cells and/or intrinsic factor, thus preventing the absorption of vitamin B12. It is most common in women over 60 years. Around 90% of patients demonstrate antiparietal antibodies (but some normal women also have these) and 60% are found to have anti-intrinsic factor antibody (which is a more specific marker).

Patients with bowel pathology, such as Crohn’s disease (this scenario), have normal levels of intrinsic factor but cannot absorb the vitamin B 12–intrinsic factor complex due to disease of the terminal ileum. Although most patents with pernicious anaemia complain of lethargy and general malaise, specific features of vitamin B 12 deficiency include peripheral neuropathy, smooth tongue, angular stomatitis, depression, dementia and subacute degeneration of the spinal cord. The blood film is likely to show a macrocytic megaloblastic anaemia with hypersegmented neutrophil nuclei (>6 lobes). In addition, serum vitamin B 12 levels are low and ferritin levels are normal, reflecting normal iron stores.

How well did you know this?
1
Not at all
2
3
4
5
Perfectly
65
Q

A previously well 8-month-old baby boy becomes generally unwell. His health visitor notices that he is failing to thrive. Investigation shows a microcytic, hypochromic anaemia in association with high levels of HbF and HbA 2

Α. α-Thalassaemia
B. β-Thalassaemia major
C. β-Thalassaemia trait
D. Anaemia of chronic disease
E. Autoimmune haemolytic anaemia
F. Glucose-6-phosphate dehydrogenase deficiency
G. Hereditary spherocytosis
H. Iron-deficiency anaemia secondary to gastrointestinal bleeding
I. Iron-deficiency anaemia secondary to iron malabsorption
J. Iron-deficiency anaemia secondary to menorrhagia
K. Megaloblastic macrocytic anaemia secondary to vitamin B12deficiency
L. Megaloblastic macrocytic anaemia secondary to folate deficiency
M. Megaloblastic microcytic anaemia secondary to vitamin B12 deficiency
N. Normal variant
O. Sickle cell anaemia

A

B – β-Thalassaemia major

β-Thalassaemia major is an autosomal recessive disorder in which there is a complete lack of production of the haemoglobin β-globin chain. It occurs mainly in Mediterranean and Middle Eastern families and is due to a point mutation on chromosome 11. Because patients with β-thalassaemia major have mutations on both alleles and cannot synthesize any β-globin, they cannot produce functioning adult haemoglobin (HbA: α2β2). This condition typically presents within the first year of life when the production of fetal haemoglobin (HbF: α2γ2) begins to fall. Affected children become generally unwell and fail to thrive secondary to a severe microcytic anaemia. Ferritin levels are normal since there is no iron deficiency. A compensatory increase in the synthesis of HbF and haemoglobin A2 (HbA2: α2δ2) occurs, which can be detected on serum electrophoresis.

Clinical features include failure to thrive, lethargy, pallor and jaundice. On examination, there is often hepatosplenomegaly (secondary to extramedullary haematopoiesis), with bossing of the skull and long bone deformity (due to excessive intramedullary haematopoiesis). The treatment of β-thalassaemia major is with regular blood transfusions, aiming to maintain the haemoglobin concentration above 10 g/dL, or with allogeneic bone marrow transplantation. Regular iron chelation therapy (with desferrioxamine) is required to prevent iron overload and deposition in vital organs such as the heart, liver and endocrine glands. If untreated, death is inevitable in the first years of life.

How well did you know this?
1
Not at all
2
3
4
5
Perfectly
66
Q

A 25-year-old woman presents to the clinic with a slowly enlarging lump on the left side of her neck. She denies any other symptoms. On examination, there is a 2 cm, smooth, regular, firm lump that moves up with swallowing, but not with tongue protrusion. Lymph nodes are palpable in the left side of the neck.

A.	 Anaplastic carcinoma
B.	 De Quervain’s thyroiditis
C.	 Follicular carcinoma
D.	 Graves’ disease
E.	 Hashimoto’s thyroiditis
F.	 Haemorrhage into a cyst
G.	 Medullary carcinoma
H.	 Papillary carcinoma
I.	 Riedel’s thyroiditis
J.	 Struma ovarii
K.	 Toxic multinodular goitre
A

H – Papillary carcinoma

The majority (70%) of thyroid tumours are papillary adenocarcinomas; 20% are follicular carcinomas. Both of these tumours occur most commonly in adolescents and young adults, who present with a discrete thyroid nodule. Papillary tumours may be multifocal and they spread to lymph nodes (as in this case). Follicular tumours occur as a single encapsulated lesion, and they spread via blood to the lungs and bone. Treatment is by total thyroidectomy (except for tumours <1 cm, which can be treated by a thyroid lobectomy). Papillary and follicular carcinomas may be thyroid-stimulating hormone (TSH) dependent (i.e. the presence of TSH stimulates their growth). For this reason, after thyroid surgery, patients take lifelong thyroxine, in order to suppress endogenous TSH secretion and reduce the risk of recurrence.

How well did you know this?
1
Not at all
2
3
4
5
Perfectly
67
Q

A 54-year-old man presents to the clinic with a midline neck mass that has been increasing in size over a few months. On examination, the thyroid gland is enlarged, firm and irregular, although the patient does not complain of any pain. No cervical nodes are palpable. A core biopsy is taken, and the histology report denies the presence of malignancy.

A.	 Anaplastic carcinoma
B.	 De Quervain’s thyroiditis
C.	 Follicular carcinoma
D.	 Graves’ disease
E.	 Hashimoto’s thyroiditis
F.	 Haemorrhage into a cyst
G.	 Medullary carcinoma
H.	 Papillary carcinoma
I.	 Riedel’s thyroiditis
J.	 Struma ovarii
K.	 Toxic multinodular goitre
A

I – Reidel’s thyroiditis

Reidel’s thyroiditis is characterized by idiopathic fibrosis of the thyroid gland. Patients present with a slowly growing goitre that is firm and irregular, and for this reason it is difficult to distinguish from cancer without a biopsy. Initially, thyroid function tests are normal, but 30% of affected patients will develop hypothyroidism and hypoparathyroidism. Complications of the fibrosis include tracheal/oesophageal compression and recurrent laryngeal nerve palsy. There is no treatment for Reidel’s thyroiditis, but palliative surgery can be performed if there are compressive symptoms (e.g. dysphagia or stridor).

How well did you know this?
1
Not at all
2
3
4
5
Perfectly
68
Q

An 84-year-old woman presents with a lump in her neck that was first noticed last month and has since been growing rapidly. On further questioning, she admits to having problems swallowing her food. On examination, there is a 5 cm, irregular, hard mass on the left side, which is fixed to the overlying skin.

A.	 Anaplastic carcinoma
B.	 De Quervain’s thyroiditis
C.	 Follicular carcinoma
D.	 Graves’ disease
E.	 Hashimoto’s thyroiditis
F.	 Haemorrhage into a cyst
G.	 Medullary carcinoma
H.	 Papillary carcinoma
I.	 Riedel’s thyroiditis
J.	 Struma ovarii
K.	 Toxic multinodular goitre
A

A – Anaplastic carcinoma

This woman has anaplastic carcinoma as indicated by her advanced age and acute presentation. Anaplastic carcinoma accounts for <5% of thyroid tumours but is the most aggressive. It presents in older patients with a hard, symmetrical, rapidly enlarging goitre. Spread is to lymph nodes and local structures, e.g. the trachea (resulting in stridor) and the recurrent laryngeal nerve (leading to hoarseness). There is no effective treatment for anaplastic thyroid tumours, although palliative radiotherapy or debulking surgery can be performed if there is tracheal compression. Most patients (>90%) with anaplastic carcinoma are dead within 1 year.

How well did you know this?
1
Not at all
2
3
4
5
Perfectly
69
Q

A 55-year-old woman is noted to have an elevated blood pressure on three separate occasions by her GP. She has a recent history of persisting acne and hirsutism, for which a dermatology referral has been made. Marked bruising is observed on examination.

A.	 Acromegaly
B.	 Adult polycystic disease
C.	 Aortic dissection
D.	 Berry aneurysm
E.	 Coarctation of the aorta
F.	 Conn’s syndrome
G.	 Cushing’s disease
H.	 Non-steroidal anti-inflammatory drug use
I.	 Oral contraception
J.	 Phaeochromocytoma
K.	 Pre-eclampsia
L.	 Stress at work	
M.	White coat syndrome
A

G – Cushing’s disease

Adrenocorticotropic hormone (ACTH; also known as corticotropin) is secreted from the anterior pituitary gland in response to corticotropin-releasing hormone (CRH) from the hypothalamus. ACTH acts on the adrenal cortex to stimulate the release of glucocorticoids and androgens into the circulation. In Cushing’s disease, a pituitary adenoma secretes large amounts of ACTH into the circulation, causing uninhibited glucocorticoid secretion. Patients with glucocorticoid and androgen excess suffer from Cushing’s syndrome, which consists of a vast array of clinical features, including hypertension, central weight gain, a moon face, poor quality skin, bruising, poor wound healing, hirsutism, acne, abdominal striae, oligomenorrhoea, osteoporosis, hyperglycaemia, polyuria, polydipsia, myopathy, depression and psychosis.

Cushing’s syndrome can also be caused by iatrogenic steroid prescriptions or the presence of an ectopic ACTH-secreting tumour such as a small cell lung carcinoma. Treatment of Cushing’s syndrome requires correction of the underlying condition (e.g. removal of the pituitary adenoma in Cushing’s disease) and management of any complications such as hypertension. In cases of iatrogenic disease caused by excess steroid therapy, the offending drug should be gradually reduced over a number of weeks and replaced by an alternative ‘steroid-sparing’ drug (e.g. the immunosuppressant azathioprine).

How well did you know this?
1
Not at all
2
3
4
5
Perfectly
70
Q

A 58-year-old man complains of left-sided chest discomfort that has been progressing over the last fortnight. He is otherwise well. On examination, the left lower lobe is dull to percussion, and a chest Xray confirms a left-sided pleural effusion. A pleural tap is performed and reveals blood-stained fluid.

A.	 Bronchoscopy and biopsy
B.	 Chest X-ray
C.	 CT pulmonary angiography
D.	 D-dimers
E.	 Lung function tests
F.	 Kveim–Siltzbach test
G.	 Mantoux test
H.	 No investigation required
I.	 Pleural biopsy
J.	 Sputum culture
K.	 Sweat test
L.	 Ventilation–perfusion scan
A

I – Pleural biopsy

Mesothelioma is a malignant tumour of the pleura. The effusion of mesothelioma tends to be blood stained, but the definitive diagnosis is made by pleural biopsy

How well did you know this?
1
Not at all
2
3
4
5
Perfectly
71
Q

A 25-year-old man is taken to theatre after fracturing his hip in a motorbike accident. Postoperatively, it is noticed that he is unable to flex his left foot either up or down. There is no sensation below the left knee except over the medial aspect of the leg.

A.	 Common peroneal nerve
B.	 Femoral nerve
C.	 Lateral femoral cutaneous nerve
D.	 Lateral plantar nerve
E.	 Medial plantar nerve
F.	 Obturator nerve
G.	 Sciatic nerve
H.	 Saphenous nerve
I.	 Sural nerve
J.	 Tibial nerve
A

G – Sciatic nerve

The sciatic nerve can be damaged with fracture dislocations of the hip or by misplaced gluteal injections. Sciatic nerve palsy results in paralysis of the hamstrings and all the muscles of the leg and foot. Sensation is lost below the knee, except for the medial leg (supplied by the saphenous nerve, a branch of the femoral nerve) and the upper calf (supplied by the posterior femoral cutaneous nerve).

How well did you know this?
1
Not at all
2
3
4
5
Perfectly
72
Q

A 28-year-old woman is involved in a road traffic collision. Her right knee had hit the dashboard of the car. On arrival at the emergency department, she is unable to flex her toes on the right side. Examination reveals an absence of the ankle jerk and loss of sensation over the sole of the foot.

A.	 Common peroneal nerve
B.	 Femoral nerve
C.	 Lateral femoral cutaneous nerve
D.	 Lateral plantar nerve
E.	 Medial plantar nerve
F.	 Obturator nerve
G.	 Sciatic nerve
H.	 Saphenous nerve
I.	 Sural nerve
J.	 Tibial nerve
A

J – Tibial nerve

The tibial nerve is particularly vulnerable to damage during posterior dislocations of the knee. It can also be compressed in the posterior tarsal tunnel behind the medial malleolus. A branch of the sciatic nerve, the tibial nerve supplies the flexor compartment of the leg (calf muscles). It also gives rise to the medial and lateral plantar nerves, which supply the intrinsic muscles of the foot as well as plantar sensation. Tibial nerve palsy results in loss of toe flexion, ankle inversion and the ankle jerk. Sensation over the plantar surface of the foot is lost. Affected patients walk with a shuffling gait, as the take-off phase of walking is impaired. There is loss of the lateral longitudinal arch of the foot, and atrophy of the intrinsic foot muscles eventually results in a claw foot.

How well did you know this?
1
Not at all
2
3
4
5
Perfectly
73
Q

A 16-year-old boy was previously admitted to the orthopaedic ward with an ankle fracture, which was treated with a plaster cast. When the cast is removed, the boy is unable to dorsiflex his foot.

A.	 Common peroneal nerve
B.	 Femoral nerve
C.	 Lateral femoral cutaneous nerve
D.	 Lateral plantar nerve
E.	 Medial plantar nerve
F.	 Obturator nerve
G.	 Sciatic nerve
H.	 Saphenous nerve
I.	 Sural nerve
J.	 Tibial nerve
A

A – Common peroneal nerve

The common peroneal nerve (or common fibular nerve) is a branch of the sciatic nerve that supplies the dorsiflexors and evertor muscles of the foot and sensation to the lateral lower leg and upper foot. The common peroneal nerve lies in close proximity to the fibula, and may become trapped by below-knee plaster casts or damaged with fibular fractures. Features of common peroneal nerve lesions include lack of dorsiflexion (with a resulting foot drop) and loss of sensation in the anterolateral lower leg and dorsum of the foot (except for the lateral aspect of the foot, which is supplied by the sural nerve). The inability to dorsiflex the foot will result in a ‘high-stepping’ gait to ensure that the foot is not scraped along the ground.

How well did you know this?
1
Not at all
2
3
4
5
Perfectly
74
Q

You are called to see a 43-year-old woman following an open cholecystectomy. She is making noises but no understandable words. Her eyes open in response to speech and she withdraws appropriately from painful stimuli.

What’s the GCS?

A

9 - This woman is opening her eyes in response to speech (E3), is making noises but no words (V2) and withdraws from painful stimuli (M4): GCS = 3 + 2 + 4 = 9.

How well did you know this?
1
Not at all
2
3
4
5
Perfectly
75
Q

A 12-year-old girl presents with jaundice. Her mother says that she has been behaving differently over the last few weeks. Investigation shows deranged liver function tests, and an ultrasound scan confirms cirrhotic changes within the liver.

A.	 α1-Antitrypsin level
B.	 α-Fetoprotein
C.	 Antimitochondrial antibody
D.	 Antinuclear antibodies
E.	 Ceruloplasmin level
F.	 Cytomegalovirus PCR
G.	 Hepatitis A antigen
H.	 Hepatitis A IgG
I.	 Hepatitis A IgM
J.	 Hepatitis C RNA
K.	 Hepatitis C antibody
L.	 Total iron-binding capacity
A

E – Ceruloplasmin level

Wilson’s disease is an autosomal recessive disease of copper metabolism. In health, copper is transported in the serum bound to ceruloplasmin (ferroxidase) and is excreted in the bile. In Wilson’s disease, there is a defect in copper metabolism such that it is not excreted in the bile and instead accumulates in the tissues. This process suppresses the synthesis of ceruloplasmin, which allows unbound (free) copper to enter the circulation. The high levels of free serum copper accumulate in vital organs such as the liver (resulting in cirrhosis), eye (causing Keiser–Fleischer rings), kidney and basal ganglia of the brain (leading to personality changes). If left untreated, the patient is at risk of developing cirrhosis, renal tubular disease, neurological disease and neuropsychiatric complications. Patients with Wilson’s disease have elevated serum free copper, urinary copper and hepatic copper (via biopsy). Paradoxically, total serum copper is usually low.

How well did you know this?
1
Not at all
2
3
4
5
Perfectly
76
Q

A 42-year-old woman presents with a 4-week history of generalized itching. She has developed yellow lesions in the skin around her eyes. Blood tests reveal deranged liver function tests.

A.	 α1-Antitrypsin level
B.	 α-Fetoprotein
C.	 Antimitochondrial antibody
D.	 Antinuclear antibodies
E.	 Ceruloplasmin level
F.	 Cytomegalovirus PCR
G.	 Hepatitis A antigen
H.	 Hepatitis A IgG
I.	 Hepatitis A IgM
J.	 Hepatitis C RNA
K.	 Hepatitis C antibody
L.	 Total iron-binding capacity
A

C – Antimitochondrial antibody

Primary biliary cirrhosis is thought to be an autoimmune disease in which chronic granulomatous inflammation of the interlobular bile ducts causes cirrhosis, portal hypertension and liver failure. It tends to affect middle-aged women, who usually present with pruritis and deranged liver function tests. Xanthelasmas are a recognized feature (as in this case). Up to 98% of patients with primary biliary cirrhosis are positive for the highly specific antimitochondrial antibody M2 subtype. Other investigations used to diagnose primary biliary cirrhosis include hepatic ultrasound imaging, endoscopic retrograde cholangiopancreatography (ERCP) and liver biopsy. The treatment of primary biliary cirrhosis is largely symptomatic. Cholestyramine is used to treat pruritis, and ursodeoxycholic acid may improve ascites and jaundice but is unlikely to prolong life. Without liver transplantation, most patients will die approximately 2 years after the development of jaundice.

How well did you know this?
1
Not at all
2
3
4
5
Perfectly
77
Q

A 32-year-old man has recently returned from holiday in India, where he stayed with locals and dined with them. Two weeks later, he presents with jaundice and lethargy. Liver function tests reveal a massively raised ALT and a significantly raised bilirubin.

A.	 α1-Antitrypsin level
B.	 α-Fetoprotein
C.	 Antimitochondrial antibody
D.	 Antinuclear antibodies
E.	 Ceruloplasmin level
F.	 Cytomegalovirus PCR
G.	 Hepatitis A antigen
H.	 Hepatitis A IgG
I.	 Hepatitis A IgM
J.	 Hepatitis C RNA
K.	 Hepatitis C antibody
L.	 Total iron-binding capacity
A

I – Hepatitis A IgM

Hepatitis A is caused by an RNA virus that is transmitted by the faecal–oral route. It often presents with a non-specific lethargic illness that is followed by the development of jaundice. The earliest serological marker of acute infection is hepatitis A immunoglobulin M (IgM) antibody. Hepatitis A IgG antibodies develop later in the course of the disease, and remain for many years. Hepatitis IgG antibodies usually convey immunity to the disease.

How well did you know this?
1
Not at all
2
3
4
5
Perfectly
78
Q

A 27-year-old nurse sustained a needlestick injury 4 months ago. She attends an occupational health appointment, where a blood test is done to exclude hepatitis C infection.

A.	 α1-Antitrypsin level
B.	 α-Fetoprotein
C.	 Antimitochondrial antibody
D.	 Antinuclear antibodies
E.	 Ceruloplasmin level
F.	 Cytomegalovirus PCR
G.	 Hepatitis A antigen
H.	 Hepatitis A IgG
I.	 Hepatitis A IgM
J.	 Hepatitis C RNA
K.	 Hepatitis C antibody
L.	 Total iron-binding capacity
A

K – Hepatitis C antibody

Hepatitis C is caused by an RNA virus that is transmitted through sexual activities and blood products, including the sharing of needles among intravenous drug users. The acute infection is often silent, with many individuals being completely asymptomatic. Hepatitis C antibodies can be used to diagnose previous infection, but take up to 3 months to develop. Approximately 85% of patients with acute hepatitis C infection go on to become chronic carriers, which carries a significant risk of developing cirrhosis, liver failure and hepatocellular carcinoma.

How well did you know this?
1
Not at all
2
3
4
5
Perfectly
79
Q

A 60-year-old woman with chronic renal failure mentions that she is becoming more generally lethargic. Blood tests show a calcium of 1.73 mmol/L, a phosphate of 2.7 mmol/L and an albumin of 45 g/L.

A.	 Advise exercise
B.	 Alfacalcidol
C.	 Intravenous bisphosphonate infusion
D.	 Intravenous calcium gluconate
E.	 Intravenous erythropoietin
F.	 Iron sulphate supplementation
G.	 Low-potassium diet
H.	 Oral ACE inhibitor and fluid restriction
I.	 Oral calcium channel blocker and fluid restriction
J.	 Oral erythropoietin
K.	 Phosphate supplements
L.	 Statin therapy
M.	 Stop phosphate binders
N.	 Urgent dialysis
A

B – Alfacalcidol

This patient is hypocalcaemic. Hypocalcaemia in chronic renal failure is due to reduced synthesis of vitamin D. In health, cholecalciferol is formed within the skin in response to sunlight. Cholecalciferol is hydroxylated in the liver to form 25-hydroxycholecalciferol, which is then hydroxylated again in the kidney by 1α-hydroxylase to form 1,25-dihydroxycholecalciferol. This is the active form of vitamin D, which increases the absorption of calcium from the gastrointestinal tract, increases the reabsorption of calcium and phosphate from the kidneys, and reduces parathyroid hormone secretion. The hypocalcaemia of renal failure is treated by supplementing the active form of vitamin D with calcitriol (1,25- dihydroxycholecalciferol) and alfacalcidol (1-hydroxycholecalciferol).

How well did you know this?
1
Not at all
2
3
4
5
Perfectly
80
Q

A 45-year-old man with chronic renal failure is found to have a blood pressure of 180/95 mmHg during an outpatient assessment. On reviewing the records, it is apparent that his blood pressure has been climbing for a number of weeks.

A.	 Advise exercise
B.	 Alfacalcidol
C.	 Intravenous bisphosphonate infusion
D.	 Intravenous calcium gluconate
E.	 Intravenous erythropoietin
F.	 Iron sulphate supplementation
G.	 Low-potassium diet
H.	 Oral ACE inhibitor and fluid restriction
I.	 Oral calcium channel blocker and fluid restriction
J.	 Oral erythropoietin
K.	 Phosphate supplements
L.	 Statin therapy
M.	 Stop phosphate binders
N.	 Urgent dialysis
A

I – Oral calcium channel blocker and fluid restriction

Hypertension is an important and common complication of chronic renal failure. Hypertension is largely due to fluid retention, although increased renin secretion may play a role, especially when renal vascular disease is present. Fluid restriction may be enough to reduce the patient’s blood pressure. Oral antihypertensive agents such as calcium channel blockers or β-blockers may also be prescribed. Caution should be taken in prescribing ACE inhibitors, as they can worsen renal failure and promote hyperkalaemia. In patients receiving dialysis, the regimen can be altered to remove fluid as well as waste products in a process known as ultrafiltration.

How well did you know this?
1
Not at all
2
3
4
5
Perfectly
81
Q

A 44-year-old woman presents following a faint after standing up from a sitting position. She did not lose consciousness during this episode, but merely reported feeling ‘dizzy and lightheaded’. She has no significant past medical history. On examination, you notice multiple areas of skin depigmentation but increased pigmentation in the palmar creases and on the elbows.

A. 17-Hydroxyprogesterone levels
B. 24-hour urinary vanillylmandelic acid
C. 24-hour urinary 5-hydroxyindole acetic acid
D. Aldosterone and renin levels
E. Dexamethasone suppression test
F. Parathyroid hormone and calcium levels
G. Serum calcitonin
H. Short Synacthen test
I. Vitamin D levels

A

H – Short Synacthen test

This woman presents with postural hypotension. She has vitiligo and some areas of increased skin pigmentation, making her likely to have Addison’s disease. The best investigation to perform is the short Synacthen test.

Addison’s disease is primary autoimmune-mediated adrenocortical failure. The action of the adrenal cortex can be described in simplified terms as the secretion of three things: glucocorticoids, mineralocorticoids and adrenal androgens. These usually feed back to the anterior pituitary to reduce ACTH (adrenocorticotropic hormone, also known as corticotropin) secretion. Therefore, failure of the adrenal cortex has many consequences: reduced glucocorticoids (leading to hypoglycaemia and weight loss), reduced mineralocorticoids (leading to hyperkalaemia, hyponatraemia and hypotension), reduced adrenal androgens (leading to decreased body hair and libido) and ACTH excess (leading to increased pigmentation in sun-exposed areas, pressure areas, palmar creases, buccal mucosa and recent scars). The diagnosis of Addison’s disease is by the short Synacthen test. In this investigation, plasma cortisol levels are measured before and half an hour after administration of a single intramuscular dose of ACTH. Normally, the ACTH will result in a rise in cortisol. If there is no rise in cortisol on the second reading, adrenal insufficiency is indicated.

Management of Addison’s disease is with the replacement of glucocorticoids and mineralocorticoids (with hydrocortisone and fludrocortisone).

How well did you know this?
1
Not at all
2
3
4
5
Perfectly
82
Q

A 5-year-old girl is referred to the paediatric clinic by her GP with precocious puberty. On examination, she is found to have clitoromegaly and some pubic hair. She is above the 98th centile for height and weight.

A. 17-Hydroxyprogesterone levels
B. 24-hour urinary vanillylmandelic acid
C. 24-hour urinary 5-hydroxyindole acetic acid
D. Aldosterone and renin levels
E. Dexamethasone suppression test
F. Parathyroid hormone and calcium levels
G. Serum calcitonin
H. Short Synacthen test
I. Vitamin D levels

A

A - 17-Hydroxyprogesterone levels

The presentation of clitoromegaly, precocious puberty and accelerated growth in this young girl is indicative of congenital adrenal hyperplasia (CAH). This is an autosomal recessive deficiency of the enzyme 21-hydroxylase. This enzyme is required to synthesize mineralocorticoids and glucocorticoids (but not adrenal androgens) from the hormone precursor 17-hydroxyprogesterone. Because there is a lack of mineralocorticoids and glucocorticoids, there is no negative feedback on the anterior pituitary, resulting in increased ACTH secretion. The high ACTH then causes an increased secretion of adrenal androgens, since this does not require the deficient hormone. The androgens result in the physical features of CAH, namely ambiguous genitalia (in girls), precocious puberty (in girls), accelerated growth in childhood and virilization. The diagnosis of CAH is suggested by finding a raised concentration of the precursor 17- hydroxyprogesterone. Treatment is with hydrocortisone and fludrocortisone to replace the deficient steroids

How well did you know this?
1
Not at all
2
3
4
5
Perfectly
83
Q

A 50-year-old man attends the emergency department complaining of episodes of flushing and diarrhoea associated with difficulty in breathing. On further questioning, you find that these episodes are precipitated by stress and alcohol. On examination, his blood pressure is 130/86 mmHg and his heart rate is 82 beats/min.

A. 17-Hydroxyprogesterone levels
B. 24-hour urinary vanillylmandelic acid
C. 24-hour urinary 5-hydroxyindole acetic acid
D. Aldosterone and renin levels
E. Dexamethasone suppression test
F. Parathyroid hormone and calcium levels
G. Serum calcitonin
H. Short Synacthen test
I. Vitamin D levels

A

C - 24-hour urinary 5-hydroxyindole acetic acid

The features of paroxysmal flushing, diarrhoea, bronchospasm and abdominal pain precipitated by stress, alcohol and caffeine strongly suggest carcinoid syndrome.

Carcinoid tumours are tumours of enterochromaffin cells of the gastrointestinal tract (most commonly of the appendix, ileum or rectum) that secrete serotonin (5-hydroxytryptamine, 5-HT). The secreted 5-HT is carried from the bowel, via the portal vein, to the liver, where it is harmlessly broken down. However, when carcinoid tumours metastasize to the liver, they can secrete 5-HT directly into the bloodstream, bypassing liver metabolism and resulting in the symptoms described above. The presence of carcinoid metastases in the liver that result in symptoms is known as carcinoid syndrome. The diagnosis of carcinoid syndrome is by measuring 24-hour urinary 5-hydroxyindole acetic acid (5-HIAA), a breakdown product of 5-HT. Management is by resection or, in widespread disease, symptomatic treatment with octreotide (a somatostatin analogue that inhibits 5-HT release). Carcinoid tumours are slow growing so, even if disseminated disease is present, patients can live for many years.

How well did you know this?
1
Not at all
2
3
4
5
Perfectly
84
Q

A 48-year-old man is brought to the GP by his wife. She says that he has been confused over the last 7 days and has been falling over a lot. Examination reveals a nystagmus but no other neurological signs.

A.	 Alzheimer’s disease
B.	 Creutzfeldt–Jakob disease
C.	 Depressive pseudodementia
D.	 HIV dementia
E.	 Huntington’s disease
F.	 Lewy body dementia
G.	 Neurosyphilis
H.	 Normal-pressure hydrocephalus
I.	 Parkinson’s disease
J.	 Pick’s disease
K.	 Vascular dementia
L.	 Wernicke’s encephalopathy
A

L – Wernicke’s encephalopathy

Wernicke’s encephalopathy is a reversible condition caused by a severe deficiency of thiamine (vitamin B1). It is often associated with alcohol abuse, the processes involved being a lack of adequate oral intake, hyperemesis and malabsorption caused by gastrointestinal lesions. The triad of features in Wernicke’s syndrome is confusion, ataxia and nystagmus. Ophthalmoplegia is also an important feature. If untreated, Wernicke’s encephalopathy can lead to the irreversible Korsakoff’s syndrome, characterized by confusion, anterograde and retrograde amnesia, and confabulation.

How well did you know this?
1
Not at all
2
3
4
5
Perfectly
85
Q

A 67-year-old man presents with symptoms of acute left ventricular failure. On examination, you notice that his heart rate is 120 beats/ min. On auscultation, there are more than two heart sounds present, but you cannot distinguish them from each other.

A. Austin Flint murmur
B. Continuous murmur radiating to the back
C. Ejection systolic murmur heard at the right second intercostal space only
D. Ejection systolic murmur heard best in the right second intercostal space that radiates into the right carotid artery
E. Graham Steell murmur
F. Muffled heart sounds
G. Mid-diastolic click loudest at the apex
H. Midsystolic murmur
I. Normal heart sounds
J. Opening snap best heard at the apex
K. Pansystolic murmur heard best at the apex with radiation into the axilla
L. Physiological splitting of the second heart sound
M. Reverse splitting of the second heart sound
N. Summation gallop rhythm

A

N – Summation gallop rhythm

A gallop rhythm is heard when the first (S1) and second (S2) heart sounds are followed by a pathological third (S3) and/or fourth (S4) heart sound. It is most commonly associated with left ventricular failure. When this rhythm is associated with tachycardia, the heart sounds cannot be individually distinguished and therefore ‘summate’ into a single sound. The third heart sound occurs in early diastole, and is caused by the rush of blood entering the ventricle as it relaxes. The presence of a third heart sound can be a normal finding in those below 40 years of age, but when pathological is associated with cardiac failure, mitral regurgitation and dilated cardiomyopathy. On auscultation, the presence of a third heart sound is thought to resemble the phonetic pronunciation of the word ‘Kentucky’.

The fourth heart sound occurs just before the first heart sound in the cardiac cycle. It is caused by the atria contracting against abnormally stiff ventricles, and is always pathological. A fourth heart sound can be heard in left ventricular hypertrophy, e.g. caused by aortic stenosis, systemic hypertension, amyloidosis and hypertrophic obstructive cardiomyopathy. On auscultation, the presence of a fourth heart sound is thought to resemble the phonetic pronunciation of the word ‘Tennessee’.

How well did you know this?
1
Not at all
2
3
4
5
Perfectly
86
Q

You are called to see an 87-year-old female inpatient who is complaining of chest pain. On examination, you notice a murmur. A brief look in her hospital notes confirms that she has known ischaemic heart disease, hypertension and aortic valve sclerosis.

A. Austin Flint murmur
B. Continuous murmur radiating to the back
C. Ejection systolic murmur heard at the right second intercostal space only
D. Ejection systolic murmur heard best in the right second intercostal space that radiates into the right carotid artery
E. Graham Steell murmur
F. Muffled heart sounds
G. Mid-diastolic click loudest at the apex
H. Midsystolic murmur
I. Normal heart sounds
J. Opening snap best heard at the apex
K. Pansystolic murmur heard best at the apex with radiation into the axilla
L. Physiological splitting of the second heart sound
M. Reverse splitting of the second heart sound
N. Summation gallop rhythm

A

C – Ejection systolic murmur heard at the right second intercostal space only

Aortic sclerosis is caused by age-related degeneration of the aortic valve. The condition is asymptomatic and is usually picked up incidentally. On auscultation, there is an ejection systolic murmur heard only in the aortic area, with no associated radiation or ejection click.

Aortic stenosis, on the other hand, is usually symptomatic, causing a classic triad of exertional dyspnoea, exertional angina and exertional syncope. Sudden death may also be precipitated. The symptoms of aortic stenosis are due to left ventricular outflow obstruction and myocardial ischaemia secondary to left ventricular hypertrophy. It is a pathological condition caused by rheumatic fever, senile calcification of the valve or a congenital bicuspid aortic valve. On auscultation, there is an ejection systolic murmur that is best heard with the diaphragm in the aortic area (second right intercostal space) and that radiates into the carotid arteries. This may or may not be associated with an ejection click. Other signs of aortic stenosis include a non-displaced left ventricular heave and a slow-rising carotid pulse.

87
Q

A 45-year-old man presents with abdominal pain. He is known to have Marfan’s syndrome with advanced aortic valve regurgitation.

A. Austin Flint murmur
B. Continuous murmur radiating to the back
C. Ejection systolic murmur heard at the right second intercostal space only
D. Ejection systolic murmur heard best in the right second intercostal space that radiates into the right carotid artery
E. Graham Steell murmur
F. Muffled heart sounds
G. Mid-diastolic click loudest at the apex
H. Midsystolic murmur
I. Normal heart sounds
J. Opening snap best heard at the apex
K. Pansystolic murmur heard best at the apex with radiation into the axilla
L. Physiological splitting of the second heart sound
M. Reverse splitting of the second heart sound
N. Summation gallop rhythm

A

A – Austin Flint murmur

Causes of aortic regurgitation include aortic dilatation (in Marfan’s syndrome), infective endocarditis and rheumatic fever. The Austin Flint murmur is heard in patients with significant aortic regurgitation. It is a mid-diastolic low-pitched rumbling murmur best heard at the apex. The murmur is caused when a regurgitated jet of blood passes through the damaged aortic valve and strikes the anterior leaflet of the mitral valve. The more common murmur of aortic regurgitation is a high-pitched early diastolic murmur best heard along the left sternal border with the patient leaning forward at the end of expiration (which is not an option in the present case).

Other signs of aortic regurgitation include a collapsing pulse, capillary pulsation in the nail beds (Quincke’s sign), ‘pistol shot’ femoral bruits (Traube’s sign) and visible carotid pulsation (Corrigan’s sign).

88
Q

A 5-year-old girl is brought to the GP by her mother because she has longstanding freckling in her mouth. On examination, the child has multiple bluish-black macular lesions around the lips and nose. She is otherwise well.

A.	 5-Fluorouracil
B.	 Analgesia
C.	 Cryotherapy
D.	 Fluid resuscitation
E.	 Intravenous antibiotics
F.	 Oral antibiotics
G.	 Surgical debridement and intravenous antibiotics
H.	 Surgical excision
I.	 Reassurance
A

I – Reassurance

This boy has Peutz–Jeghers syndrome, a condition characterized by multiple bluish-black freckles around the lips, nose, oral mucosa and fingers, as well as multiple gastrointestinal hamartomatous polyps. These polyps are benign and have only a very low malignant potential. The polyps may predispose to gastrointestinal bleeding or intussusceptions, but, in the asymptomatic patient, reassurance is sufficient.

89
Q

A 45-year old woman presents with a 2-month history of hip and shoulder weakness. She is finding it especially difficult to climb stairs. The weakness is demonstrable on examination and she is tender over the proximal joints. No skin lesions are visible.

A.	 Antiphospholipid syndrome
B.	 Dermatomyositis
C.	 Diffuse cutaneous systemic sclerosis
D.	 Limited cutaneous systemic sclerosis
E.	 Overlap syndromes
F.	 Polymyositis
G.	 Sjögren’s syndrome
H.	 Systemic lupus erythematosus
A

F – Polymyositis

Polymyositis is a rare inflammatory disorder of skeletal muscle. Patients present with proximal muscle weakness (hips and shoulder), which may manifest as difficulty in climbing stairs or in standing up from a sitting position. This may be accompanied by swelling, tenderness and wasting of the affected muscle. Blood tests reveal a raised creatine kinase (CK), but the definitive investigation is muscle biopsy, which demonstrates inflammation and necrosis. Polymyositis is associated with underlying malignancy. Anti-Jo-1 antibodies may be present. Treatment is with steroids and immunosuppressants.

90
Q

A 32-year-old woman presents with sudden-onset shortness of breath with a cough productive of blood. She has had two pulmonary emboli in the past. On examination, the chest is clear and no skin lesions are seen.

A.	 Antiphospholipid syndrome
B.	 Dermatomyositis
C.	 Diffuse cutaneous systemic sclerosis
D.	 Limited cutaneous systemic sclerosis
E.	 Overlap syndromes
F.	 Polymyositis
G.	 Sjögren’s syndrome
H.	 Systemic lupus erythematosus
A

A – Antiphospholipid syndrome

Antiphospholipid syndrome is characterized by recurrent arterial and venous thromboses (pulmonary embolism, deep vein thrombosis, stroke and peripheral thrombosis), recurrent miscarriage and thrombocytopenia. The diagnosis is confirmed by the presence of specific autoantibodies (anticardiolipin and lupus anticoagulase). Note that the presence of the anticardiolipin antibody will lead to a false-positive VDRL (Venereal Disease Reference Laboratory – syphilis) test. The management of antiphospholipid syndrome involves avoidance of thrombotic risk factors, including smoking and the contraceptive pill, and treating hypertension, hyperlipidaemia and diabetes. Patients who have no history of thrombosis should take low-dose aspirin. After the first thrombotic episode, lifelong warfarin is taken, aiming for an INR of 2.5. If a patient is planning on getting pregnant, warfarin should be stopped (as it is teratogenic) and subcutaneous heparin used instead.

91
Q

A 49-year-old woman has noticed that the skin on her face and hands is becoming tight and uncomfortable. She denies any other symptoms. On examination, there are multiple telangiectasia on her face. The skin on both her hands and her face is tight and shiny, but the skin of the proximal arms is unaffected.

A.	 Antiphospholipid syndrome
B.	 Dermatomyositis
C.	 Diffuse cutaneous systemic sclerosis
D.	 Limited cutaneous systemic sclerosis
E.	 Overlap syndromes
F.	 Polymyositis
G.	 Sjögren’s syndrome
H.	 Systemic lupus erythematosus
A

D – Limited cutaneous systemic sclerosis

Systemic sclerosis is a connective tissue disorder characterized by thickening and fibrosis of the skin (scleroderma) with involvement of internal organs. There are two forms: a limited cutaneous type (60%) and a diffuse cutaneous type (40%). Limited cutaneous scleroderma is limited to the distal limbs (i.e. distal to the elbows and knees). Other features include a beaked nose and small, furrowed mouth (microstoma). Limited cutaneous scleroderma also encompasses the CREST syndrome, which is characterized by CALCINOSIS, RAYNAUD’S phenomenon, oESOPHAGEAL dysmotility, SCLERODACTYLY and TELANGIECTASIA. Calcinosis is the formation of calcium deposits in the soft tissues, often seen on the pulps of the fingers. Oesophageal dysmotility is manifested as dysphagia and reflux. Sclerodactyly describes the presence of tight, shiny skin over the fingers, producing a fixed flexion deformity. In limited cutaneous scleroderma, the anticentromere antibody is characteristically positive. Pulmonary hypertension is a common internal manifestation.

Diffuse cutaneous scleroderma can involve the whole skin of the body. Patients are characteristically positive for the anti-SCL-70 antibody (also known as antitopoisomerase II). In this form of the disease, patients are particularly at risk of a ‘renal crisis’ – a life-threatening malignant hypertension with rapid renal impairment. In contrast, pulmonary hypertension is less common

92
Q

An 18-year-old woman presents with a long history of dry, itchy eyes. She denies having any problems with her vision. There is no abnormality on examination.

A.	 Antiphospholipid syndrome
B.	 Dermatomyositis
C.	 Diffuse cutaneous systemic sclerosis
D.	 Limited cutaneous systemic sclerosis
E.	 Overlap syndromes
F.	 Polymyositis
G.	 Sjögren’s syndrome
H.	 Systemic lupus erythematosus
A

G – Sjögren’s syndrome

The main features of Sjogren’s syndrome are dry eyes (keratoconjunctivitis sicca) and dry mouth (xerostoma). Other features are corneal ulcers, oral candida, vaginal dryness, dyspareunia and respiratory hoarseness. Diagnosis is with Schirmer’s test: a 35 mm-long piece of filter paper is placed under the lower eyelid for 5 minutes – if less than 10 mm becomes moist, it indicates Sjögren’s syndrome. Anti-Ro and Anti-La antibodies may be present. Treatment is with artificial tears and artificial saliva.

93
Q

A 27-year-old woman has developed a red, scaly rash on her checks and nose that is worse in the sunlight. She also complains of pain in the small joints of her hands. On examination, there are no additional cutaneous lesions, and no deformity is seen in the hands.

A.	 Antiphospholipid syndrome
B.	 Dermatomyositis
C.	 Diffuse cutaneous systemic sclerosis
D.	 Limited cutaneous systemic sclerosis
E.	 Overlap syndromes
F.	 Polymyositis
G.	 Sjögren’s syndrome
H.	 Systemic lupus erythematosus
A

H – Systemic lupus erythematosus

Systemic lupus erythematosus (SLE) is most common in middle-aged women, and is characterized by arthralgia (joint pain) and rashes. Flare-ups of disease may be triggered by sunlight, the contraceptive pill, infection and stress. Common features are non-erosive arthritis (Jaccoud’s arthritis), a discoid rash on the body, a rash over the cheeks and nose (butterfly rash), fever, depression and myalgia.

Treatment of SLE is with analgesia, steroids and immunosuppressants. Lupus can be induced by certain drugs, e.g. isoniazid and penicillamine. Drug-induced lupus is associated with antihistone antibodies. Discoid lupus is a benign variant in which there is skin involvement only.

94
Q

An 83-year-old man is referred for bone marrow aspiration after presenting with hepatomegaly, splenomegaly and pancytopenia. No marrow could be aspirated during the first attempt. A trephine biopsy is successfully performed, and histology reveals hypercellular marrow containing many abnormal megakaryocytes.

A.	 Chronic myeloid leukaemia
B.	 Dehydration
C.	 Essential thrombocythaemia
D.	 Hodgkin’s lymphoma
E.	 Monoclonal gammopathy of uncertain significance
F.	 Multiple myeloma
G.	 Myelofibrosis
H.	 Non-Hodgkin’s lymphoma
I.	 Osteoporosis
J.	 Polycythaemia rubra vera
K.	 Secondary polycythaemia
L.	 Waldenström’s macroglobulinaemia
A

G – Myelofibrosis

Myelofibrosis is a myeloproliferative disease characterized by the replacement of erythropoietic bone marrow with inert fibrotic material. The disease originates from a single abnormal pluripotent stem cell that populates the marrow with abnormal megakaryocytes, which in turn secrete factors known to stimulate fibrosis. Intramedullary fibrosis causes marrow failure and subsequent pancytopenia. In response to anaemia, extramedullary erythropoiesis in the liver and spleen produces the common finding of massive hepatosplenomegaly. Diagnosis is usually based on the combination of a pancytopenic blood film with characteristic tear-drop poikilocytes, a dry bone marrow aspirate and a trephine biopsy showing dense fibrosis of the bone marrow. Myelofibrosis is treated symptomatically with blood transfusion, chemotherapy and splenectomy in severe cases. The average survival time is less than 5 years, although some patients survive for many years. Death usually occurs due to the development of AML, infection or liver failure.

95
Q

You are called to see a 55-year-old man on the ward who has developed a painful pulsatile mass in his groin while awaiting discharge following an elective angiogram.

A. Annual ultrasound monitoring
B. Elective surgical repair
C. Intravenous antibiotics
D. Maintain systolic blood pressure above 150 mmHg with fluid resuscitation while awaiting emergency surgical repair
E. Maintain systolic blood pressure at approximately 100 mmHg with fluid resuscitation while awaiting emergency surgical repair
F. No intervention necessary
G. Oral antihypertensive medication and close blood pressure monitoring
H. Reduce systolic blood pressure to 100 mmHg using an intravenous antihypertensive agent and refer for emergency surgical repair
I. Urgent abdominal ultrasound scan
J. Urgent CT scan
K. Urgent magnetic resonance angiography
L. Urgent ultrasound scan and compression

A

L – Urgent ultrasound scan and compression

This man has developed a false aneurysm, which, by definition, involves only one or two layers of the vessel wall. A false femoral artery aneurysm is a recognized complication of angiography, as this artery is often used to gain access to the vasculature. People who use illict drugs may cause a false femoral aneurysm by inadvertently piercing the artery instead of the vein. Initial management is by compression. Ultrasound confirms the diagnosis. Some cases of false aneurysm will require surgical correction because of the risk of rupture, thrombosis and infection.

96
Q

A 65-year-old man has recently suffered an acute right-sided hemiparesis. His wife now says that he talks gibberish. There is no evidence of a change in quality or speed of his speech. When you ask him what his name is, he replies ‘Pleased to breakfast you’. Apart from the hemiparesis, examination is unremarkable.

A.	 Broca’s aphasia
B.	 Bulbar palsy
C.	 Cerebellar speech
D.	 Dysphonia
E.	 Lesion of the arcuate fasciculus
F.	 Psudobulbar palsy
G.	 Wernicke’s aphasia
A

G – Wernicke’s aphasia

Language processing is done predominantly in the left hemisphere. Wernicke’s area (in the left temporal lobe near the primary auditory cortex) is responsible for recognizing and analysing spoken language. Broca’s area (in the left frontal lobe near the primary motor cortex) is responsible for producing coherent speech.

Wernicke’s and Broca’s areas are connected by the arcuate fasciculus. In fluent aphasia, the normal number of words is produced, but the wrong words are spoken. In non-fluent aphasia, verbal output is reduced. Damage to Wernicke’s area (Wernicke’s aphasia) results in a fluent aphasia with poor comprehension and poor repetition. This is because the patient cannot analyse what has been said, but is still able to say something thanks to an intact Broca’s area.

Conversely, in Broca’s aphasia, there is a non-fluent aphasia with good comprehension and poor repetition. In other words, the patient can wholly understand what is said, but, because they have problems with the motor side of speech, they are unable to repeat what you have said or indeed to say much at all. A lesion in the arcuate fasciculus (which joins the two areas) results in a fluent aphasia with good comprehension but poor repetition – i.e. an intact Wernicke’s area means that the patient can understand what is said, an intact Broca’s area means that they can speak, but the lack of a connection between the two means that they would not be able to repeat a phrase that is told to them.

97
Q

A 67-year-old man presents to the GP with difficulty speaking and swallowing. This has been getting worse since a stroke 4 months ago. He has no problems with comprehension and is able to answer questions appropriately. On examination, the tongue appears small and contracted. The jaw jerk is brisk.

A.	 Broca’s aphasia
B.	 Bulbar palsy
C.	 Cerebellar speech
D.	 Dysphonia
E.	 Lesion of the arcuate fasciculus
F.	 Psudobulbar palsy
G.	 Wernicke’s aphasia
A

F – Pseudobulbar palsy

The lower cranial nerves (IX to XII) supply the muscles involved in speech and swallowing. Damage to these nerves results in dysphagia and dysarthria. The lesions can either involve the lower motor neurons (leading to bulbar palsy) or the upper motor neurons (leading to pseudobulbar palsy). Pseudobulbar palsy results in a small, contracted tongue and a brisk jaw jerk. Causes include motor neuron disease, multiple sclerosis, brain-stem tumour and brain-stem stroke. Bulbar palsy results in a wasted, fasciculating tongue and a palate that moves very little. Causes include motor neuron disease, syringobulbia, Guillain–Barré syndrome and myasthenia gravis.

98
Q

A 73-year-old woman, who recently suffered a stroke, is now having problems speaking. Her husband says that her speech is laboured and that she finds it difficult to get the right words out. When you ask her how she is, she replies ‘Me….Margaret…..doctors…..well….’. When you ask her to point to the clock, she promptly does so.

A.	 Broca’s aphasia
B.	 Bulbar palsy
C.	 Cerebellar speech
D.	 Dysphonia
E.	 Lesion of the arcuate fasciculus
F.	 Psudobulbar palsy
G.	 Wernicke’s aphasia
A

A – Broca’s aphasia

Language processing is done predominantly in the left hemisphere. Wernicke’s area (in the left temporal lobe near the primary auditory cortex) is responsible for recognizing and analysing spoken language. Broca’s area (in the left frontal lobe near the primary motor cortex) is responsible for producing coherent speech.

Wernicke’s and Broca’s areas are connected by the arcuate fasciculus. In fluent aphasia, the normal number of words is produced, but the wrong words are spoken. In non-fluent aphasia, verbal output is reduced. Damage to Wernicke’s area (Wernicke’s aphasia) results in a fluent aphasia with poor comprehension and poor repetition. This is because the patient cannot analyse what has been said, but is still able to say something thanks to an intact Broca’s area.

Conversely, in Broca’s aphasia, there is a non-fluent aphasia with good comprehension and poor repetition. In other words, the patient can wholly understand what is said, but, because they have problems with the motor side of speech, they are unable to repeat what you have said or indeed to say much at all. A lesion in the arcuate fasciculus (which joins the two areas) results in a fluent aphasia with good comprehension but poor repetition – i.e. an intact Wernicke’s area means that the patient can understand what is said, an intact Broca’s area means that they can speak, but the lack of a connection between the two means that they would not be able to repeat a phrase that is told to them.

99
Q

A 65-year-old woman is admitted with worsening shortness of breath and sputum production. A chest X-ray shows an air–fluid level behind her heart, but no evidence of consolidation.

A.	 Achalasia
B.	 Crohn’s disease
C.	 Hiatus hernia
D.	 Ischaemic colitis
E.	 Oesophageal spasm
F.	 Oesophagitis
G.	 Pancreatitis
H.	 Primary sclerosing cholangitis
I.	 Toxic megacolon
J.	 Ulcerative colitis
K.	 Visceral perforation
A

C – Hiatus hernia

A hiatus hernia occurs when the stomach herniates through the diaphragm into the thorax. Some patients remain completely asymptomatic, whereas others suffer from intolerable gastro-oesophageal reflux disease and dyspepsia. Complications include weight loss, oesophagitis, oesophageal ulceration, oesophageal stricture and aspiration pneumonia. Diagnosis is usually made during endoscopy or with barium-swallow studies, although many cases are identified incidentally on chest X-ray as a shadow behind the heart with or without an air–fluid level (caused by the presence of gastric contents). Treatment is usually conservative, and involves lifestyle adaptations such as weight loss, smoking cessation and a healthy balanced diet. Proton pump inhibitors may be used to improve symptoms. Surgical repair of medically resistant reflux is with Nissen’s fundoplication, where the fundus of the stomach is wrapped around the lower oesophagus.

100
Q

A 51-year-old man presents with acute abdominal pain and blood stained diarrhoea. An abdominal X-ray reveals a 12 cm dilatation of his transverse colon with colonic wall thickening.

A.	 Achalasia
B.	 Crohn’s disease
C.	 Hiatus hernia
D.	 Ischaemic colitis
E.	 Oesophageal spasm
F.	 Oesophagitis
G.	 Pancreatitis
H.	 Primary sclerosing cholangitis
I.	 Toxic megacolon
J.	 Ulcerative colitis
K.	 Visceral perforation
A

I – Toxic megacolon

Toxic megacolon refers to life-threatening dilatation of the colon secondary to infective or inflammatory colitis. Patients are usually known to have colitis, and present acutely with abdominal pain, abdominal distension, diarrhoea, bloodstained stools, fever and tachycardia. Abdominal X-ray shows colonic dilatation of >6 cm with or without colonic wall thickening, multiple air–fluid levels and disruption of the haustral pattern. Investigation will usually reveal significantly raised inflammatory markers and electrolyte imbalance. Patients with proven toxic megacolon should be managed in a high-dependency environment and receive fluid resuscitation, corticosteroids, antibiotics and daily abdominal Xrays to assess disease progression. If the dilatation is becoming worse or the patient is clinically deteriorating, a colectomy should be considered.

101
Q

A 67-year-old woman complaining of weight loss, abdominal pain and an increasing abdominal girth is found to have a large mass in her left lower abdomen and ascites on transvaginal ultrasound imaging.

A.	 α-Fetoprotein
B.	 Alkaline phosphatase
C.	 BRCA-1
D.	 CA 125	
E.	 CA 15-3
F.	 CA 19-9
G.	 Calcitonin	
H.	 Human carcinoembryonic antigen	
I.	 Human chorionic gonadotropin	
J.	 Neuron-specific enolase	
K.	 Prostatic acid phosphatase	
L.	 Prostate-specific antigen
A

D – CA 125

The most likely diagnosis in this case is ovarian carcinoma, for which CA 125 is a tumour marker. The clinical features of ovarian malignancy include weight loss, abdominal pain, increasing abdominal girth and ascites. Unfortunately, the symptoms of ovarian malignancy usually occur late, with the result that the disease is often advanced and metastatic at presentation. Diagnosis usually involves transvaginal ultrasonography, CT, ovarian biopsy and measurement of CA 125. Both chemotherapy and surgery are used in the management of ovarian cancer, and have both curative and palliative roles. Radiotherapy is now limited to the palliation of symptoms. Because ovarian malignancy usually presents late, the prognosis is poor. However, if the disease is diagnosed early and treated appropriately, 5-year survival rates for early stage disease can be as high as 95%.

102
Q

During endoscopic retrograde cholangiopancreatography, a 45-year old man with ulcerative colitis and jaundice is found to have multiple strictures throughout his common bile duct and biliary tree.

A.	 Achalasia
B.	 Crohn’s disease
C.	 Hiatus hernia
D.	 Ischaemic colitis
E.	 Oesophageal spasm
F.	 Oesophagitis
G.	 Pancreatitis
H.	 Primary sclerosing cholangitis
I.	 Toxic megacolon
J.	 Ulcerative colitis
K.	 Visceral perforation
A

H – Primary sclerosing cholangitis

PSC is a condition in which chronic inflammation of the biliary tract results in the development of multiple strictures within the intraand extrahepatic ducts. This condition is associated with inflammatory bowel disease, which is implicated in up to 75% of cases. It is usually seen in young men, and presents with lethargy and pruritis. As the disease progresses, the patient is at risk of developing the symptoms and complications of chronic liver disease, such as jaundice, cholangitis, hepatomegaly, coagulopathy, portal hypertension, oesophageal varices, ascites, encephalopathy and liver failure. There is also a 10% risk of developing cholangiocarcinoma.

Diagnosis is based on the presence of deranged liver function tests, hypergammaglobulinaemia and biliary tract imaging – endoscopic retrograde cholangiopancreatography (ERCP) or magnetic resonance cholangiopancreatography (MRCP). Treatment is with steroids, ursodeoxycholic acid, cholestyramine and the supplementation of fatsoluble vitamins (A, D, E and K). If there is evidence of liver failure, oesophageal varices or ascites, the patient should be considered for liver transplantation. Without a transplant, death usually occurs within 12 years

103
Q

A 55-year-old man presents with an episode of central crushing chest pain lasting 30 minutes. An ECG demonstrates widened QRS complexes, S-waves in lead V1, and an ‘M’ pattern in the QRS complex in leads V5 and V6 with associated T-wave inversion.

A.	 First-degree heart block
B.	 Second-degree: Mobitz type I heart block
C.	 Second-degree: Mobitz type II heart block
D.	 Third-degree (complete) heart block
E.	 Bifasicular block
F.	 Digitalis toxicity
G.	 Left bundle-branch block
H.	 Long-QT syndrome
I.	 Right bundle-branch block
J.	 Sick sinus syndrome
K.	 Sinus bradycardia
A

G – Left bundle-branch block

The ECG findings in this scenario are classic for left bundle-branch block (LBBB). LBBB is always pathological, and is seen in ischaemic heart disease, myocardial infarction, hypertension and cardiomyopathy. In this scenario, it is likely to be a new finding secondary to acute myocardial infarction, in which case it is an indication for thrombolysis. Since the conduction down the left bundle branch is delayed, the QRS complex is prolonged (>0.12 s) and the myocardium depolarizes from right to left, producing dominant R-waves in V6. The subsequent delayed depolarization of the left ventricle produces deep Swaves in V1 and a further R-wave in V6, which completes the ‘M’ pattern in V6. It should be noted that LBBB makes interpretation of the remaining ECG impossible.

104
Q

A tall 23-year-old medical student presents to his GP. Earlier that day, he had an ECG taken by a colleague while practising clinical skills for his final clinical examination. The tracing showed wide QRS complexes, an RSR1 pattern in V1 and V2, and large S-waves in V6. His colleague informed him that he had a conduction defect.

A.	 First-degree heart block
B.	 Second-degree: Mobitz type I heart block
C.	 Second-degree: Mobitz type II heart block
D.	 Third-degree (complete) heart block
E.	 Bifasicular block
F.	 Digitalis toxicity
G.	 Left bundle-branch block
H.	 Long-QT syndrome
I.	 Right bundle-branch block
J.	 Sick sinus syndrome
K.	 Sinus bradycardia
A

I – Right bundle-branch block

The ECG of right bundle-branch block (RBBB) shows a prolonged QRS complex (>0.12 s), an RSR pattern in V1 and prominent S-waves in V6. RBBB can be a normal variant in tall, thin adults (as in this case). More sinister causes of RBBB include pulmonary embolus, cor pulmonale and atrial septal defect. Both RBBB and LBBB have an RSR 1 pattern on one side (seen as an ‘M’), with a ‘W’ pattern on the other. Remember ‘MarroW’ (RBBB has an ‘M’ in V1 and a ‘W’ in V6) and ‘WilliaM’ (LBBB has a ‘W’ in V1 and an ‘M’ in V6).

105
Q

A 14-year-old girl develops an intensely itchy blistering rash on her forearms. She is on a special diet for coeliac disease, but is otherwise well.

A.	 Acrodermatitis enteropathica
B.	 Candidiasis
C.	 Circinate balanitis
D.	 Dermatitis herpetiformis
E.	 Erythema nodosum
F.	 Ichthyosis
G.	 Kaposi’s sarcoma
H.	 Keratoderma blenorrhagicum
I.	 Leukoplakia
J.	 Linea nigra
K.	 Lupus pernio
L.	 Pyoderma gangrenosum
M.	 Scurvy
N.	 Spider naevi
A

D – Dermatitis herpetiformis

Dermatitis herpetiformis is a blistering, intensely itchy rash that develops on the extensor surfaces. It is associated with coeliac disease and is treated with dapsone. There are many cutaneous features of gastrointestinal disease, including the following:

  • malabsorption: ichthyosis (dry, scaly skin), eczema, oedema
  • liver disease: jaundice, spider naevi, palmar erythema, leukonychia
  • renal failure: itching, half white and half red nails
  • Crohn’s disease: perianal abscess, fistulae, skin tags, aphthous ulcers
  • ulcerative colitis: erythema nodosum, pyoderma gangrenosum
  • sarcoidosis: erythema nodosum, lupus pernio (purple indurated lesions).
106
Q

A 39-year-old man develops a scaly, hyperkeratotic lesion on the soles of his feet. He also complains of pain in his left knee and pain on passing urine. He has no previous medical history.

A.	 Acrodermatitis enteropathica
B.	 Candidiasis
C.	 Circinate balanitis
D.	 Dermatitis herpetiformis
E.	 Erythema nodosum
F.	 Ichthyosis
G.	 Kaposi’s sarcoma
H.	 Keratoderma blenorrhagicum
I.	 Leukoplakia
J.	 Linea nigra
K.	 Lupus pernio
L.	 Pyoderma gangrenosum
M.	 Scurvy
N.	 Spider naevi
A

H – Keratoderma blenorrhagicum

This man has Reiter’s syndrome, characterized by arthritis, urethritis and iritis. There are two skin features that can develop alongside this disease: keratoderma blenorrhagicum and circinate balanitis. Keratoderma blenorrhagicum describes the presence of pustular, crusty, yellow–brown papular lesions on the soles of the feet that are clinically and histologically indistinguishable from pustular psoriasis. It is seen in 15% of men with Reiter’s syndrome. Circinate balanitis describes an annular, erythematous reaction on the glans penis, and occurs in 30% of men with Reiter’s syndrome.

107
Q

A 2-year-old girl is referred to the community paediatrician by her GP for failure to thrive. Examination reveals a number of poorly defined masses in her distended abdomen. Blood tests show a potassium of 3.7 mmol/L, a sodium of 138 mmol/L, a urea of 9 mmol/L and a creatinine of 130 µmol/L. An abdominal ultrasound scan is requested, and shows enlarged, irregular kidneys on both sides.

A.	 Alport’s syndrome
B.	 Autosomal recessive polycystic kidney disease
C.	 Autosomal dominant polycystic kidney disease
D.	 Bilateral renal agenesis causing Potter’s syndrome
E.	 Congenital nephrotic syndrome
F.	 Cranial diabetes insipidus
G.	 Cystinosis
H.	 Diabetes mellitus
I.	 Down’s syndrome
J.	 Duplex ureters
K.	 Nephrogenic diabetes insipidus
L.	 Posterior urethral valve obstruction
M.	Unilateral renal agenesis
A

B – Autosomal recessive polycystic kidney disease

Autosomal RECESSIVE polycystic kidney disease (ARPKD) is a genetic condition caused by mutations in the PKD1 or PKD2 genes that result in the formation of multiple fluid-filled cysts within the collecting ducts of the kidney and the liver. Severe ARPKD can present during labour with respiratory difficulties and acute renal failure. More insidious presentations are common, and typically combine failure to thrive, hypertension and worsening renal function. Patients who present earlier tend to have more renal disease in proportion to hepatic disease, whereas the opposite is true of those who present later on in life. The majority of patients with ARPKD eventually require renal replacement therapy.

This condition should not be confused with autosomal DOMINANT polycystic kidney disease, which presents later in life and is associated with berry aneurysms and subarachnoid haemorrhage.

108
Q

A 23-year-old woman from Africa gives birth to a baby who has a flat face, low-set ears and many joint abnormalities, including club feet. On further questioning, the woman admits that the doctors in Africa were worried because she did not have enough amniotic fluid. The baby dies 2 hours after birth.

A.	 Alport’s syndrome
B.	 Autosomal recessive polycystic kidney disease
C.	 Autosomal dominant polycystic kidney disease
D.	 Bilateral renal agenesis causing Potter’s syndrome
E.	 Congenital nephrotic syndrome
F.	 Cranial diabetes insipidus
G.	 Cystinosis
H.	 Diabetes mellitus
I.	 Down’s syndrome
J.	 Duplex ureters
K.	 Nephrogenic diabetes insipidus
L.	 Posterior urethral valve obstruction
M.	Unilateral renal agenesis
A

M – Unilateral renal agenesis

Unilateral renal agenesis is a relatively common condition in which one kidney does not develop. The remaining kidney hypertrophies and compensates for the missing kidney. Patients are often completely asymptomatic and are diagnosed incidentally during the investigation of other disease or post mortem. Sometimes the condition is discovered during the investigation of hypertension or an abdominal mass.

109
Q

While examining a 65-year-old man who is currently being treated for a suspected aortic dissection and acute pulmonary oedema, you hear an early diastolic murmur. In addition, you notice an exaggerated pulsation of his carotid arteries and an abnormal, short, sharp, loud systolic sound on auscultation of the femoral arteries.

A.	 First-degree heart block
B.	 Third-degree heart block
C.	 Acute aortic regurgitation
D.	 Aortic stenosis
E.	 Atrial fibrillation
F.	 Atrial flutter
G.	 Chronic aortic regurgitation
H.	 Heterozygous familial hypercholesterolaemia
I.	 Homozygous familial hypercholesterolaemia
J.	 Mitral regurgitation
K.	 Mobitz type II heart block
L.	 Myocardial infarction
M.	Rheumatic fever
N.	 Secondary hypercholesterolaemia block
O.	 Subacute bacterial endocarditis
A

C – Acute aortic regurgitation

This is a case of acute aortic regurgitation secondary to a type A (i.e. involving the ascending aorta) dissecting aortic aneurysm. Since this is an acute event, the left ventricle has had no time to compensate for the increased demands placed upon it, resulting in left ventricular failure and cardiogenic shock. On auscultation of aortic regurgitation, there is usually an early diastolic murmur at the left sternal border, which is best heard with the patient sitting forward at the end of expiration. Patients are said to have a water-hammer (collapsing) pulse and a wide blood pressure index (i.e. a high systolic and a low diastolic blood pressure). In chronic aortic regurgitation, seen in conditions such as Marfan’s syndrome and congenital bicuspid aortic valve, the apex is often displaced laterally due to left ventricular dilatation. Aortic regurgitation is associated with a number of eponymous signs that are often referred to in examinations:

  • Corrigan’s sign: visible pulsation of the carotid arteries
  • de Musset’s sign: head bobbing
  • Quincke’s sign: pulsation of the nail beds
  • Traube’s phenomenon: pistol shot sound on femoral artery auscultation.
110
Q

A 9-year-old boy is brought to the GP by his mother, who feels that he is very lethargic and bruises easily. The doctor requested a full blood count, which showed a haemoglobin concentration of 9.3 g/dL, a white cell count of 80 × 109/L and a platelet count of 30 × 109/L.

A.	 Acute lymphoblastic leukaemia
B.	 Acute myeloid leukaemia
C.	 Burkitt’s lymphoma
D.	 Chronic lymphocytic leukaemia
E.	 Chronic myeloid leukaemia
F.	 Disseminated intravascular coagulation
G.	 Hodgkin’s lymphoma
H.	 Myelofibrosis
I.	 Myeloma
J.	 Neutrocytosis
K.	 Neutropenic sepsis
L.	 Non-Hodgkin’s lymphoma
M.	Tumour lysis syndrome
A

A – Acute lymphoblastic leukaemia

Acute lymphoblastic leukaemia (ALL) is the most common leukaemia in children, and is caused by the clonal proliferation of lymphoid precursor (blast) cells that have been arrested at an early stage of development. The blast cells infiltrate the marrow and lymphoid tissue, causing pancytopenia and lymphadenopathy, respectively. The central nervous system (CNS) can also be affected, causing headache, vomiting, meningism, cranial nerve palsies and seizure. The marrow failure and subsequent pancytopenia produce the common features of anaemia, bleeding/bruising and infection. Bone pain is another common presenting symptom. A full blood count confirms anaemia and thrombocytopenia, but demonstrates a high white cell count (due to circulating blast cells).

The investigation of choice is bone marrow aspiration, which shows a hypercellular marrow with >20% blasts. Treatment involves the use of chemotherapy, as well as supportive measures such as transfusion and antibiotics. Chemotherapy is traditionally delivered in three main stages: remission induction, consolidation and maintenance (which can be for a number of years). Since patients with ALL are at high risk of neurological disease, they are often given CNS prophylaxis in the form of intrathecal methotrexate and radiotherapy. The prognosis of ALL is good with appropriate treatment.

111
Q

A 2-year-old girl with Down’s syndrome becomes generally unwell and develops multiple bruises during a 3-day illness. Blood tests show a haemoglobin concentration of 6.8 g/dL, a white cell count of 60 × 109/L and a platelet count of 10 × 109/L. Auer rods can be seen in blast cells on microscopy.

A.	 Acute lymphoblastic leukaemia
B.	 Acute myeloid leukaemia
C.	 Burkitt’s lymphoma
D.	 Chronic lymphocytic leukaemia
E.	 Chronic myeloid leukaemia
F.	 Disseminated intravascular coagulation
G.	 Hodgkin’s lymphoma
H.	 Myelofibrosis
I.	 Myeloma
J.	 Neutrocytosis
K.	 Neutropenic sepsis
L.	 Non-Hodgkin’s lymphoma
M.	Tumour lysis syndrome
A

B – Acute myeloid leukaemia

Acute myeloid leukaemia (AML) is caused by the proliferation of myeloid precursor cells. This condition is generally seen in adulthood and is rapidly fatal if untreated. AML is seen as a new disease and in patients with myeloproliferative disorders. Patients with Down’s syndrome are at increased risk of both ALL and AML, with the risk of AML being highest in the first 3 years of life. AML is increasingly seen in patients who have received chemotherapy in the past for other malignancies such as lymphoma. The presentation of AML is variable. Many sufferers are asymptomatic and are diagnosed incidentally whereas others become very unwell very quickly. Marrow failure caused by blast infiltration causes anaemia, thrombocytopenia and neutropenia. Investigation usually shows a significantly raised white blood cell count, although it may be normal or even low. Cellular inclusions called Auer rods which are seen on microscopy are pathognomonic of AML. Management of AML involves blood and platelet transfusion, treatment of infections and chemotherapy. In patients less than 55 years of age, stem cell transplantation is considered. The prognosis of AML is worse than that of ALL. Up to 80% of patients can be expected to achieve remission, but most relapse, and the cure rate is only 30%. The prognosis is worse in elderly people and in patients with a history of myeloproliferative disorders.

112
Q

A 74-year-old man complains of a 6-month history of lethargy, itching and severe abdominal discomfort. On examination, the doctor notices several areas of bruising on the skin and what he thinks to be an enlarged spleen on palpation of the abdomen. A full blood count shows a white cell count of 100 × 109/L. He is later shown to possess the Philadelphia chromosome.

A.	 Acute lymphoblastic leukaemia
B.	 Acute myeloid leukaemia
C.	 Burkitt’s lymphoma
D.	 Chronic lymphocytic leukaemia
E.	 Chronic myeloid leukaemia
F.	 Disseminated intravascular coagulation
G.	 Hodgkin’s lymphoma
H.	 Myelofibrosis
I.	 Myeloma
J.	 Neutrocytosis
K.	 Neutropenic sepsis
L.	 Non-Hodgkin’s lymphoma
M.	Tumour lysis syndrome
A

E – Chronic myeloid leukaemia

Chronic myeloid leukaemia (CML) is a myeloproliferative disorder. There is excessive proliferation of myeloid cells in the bone marrow. Patients typically have massive splenomegaly, anaemia, bruising and infection on a background generalized illness. Up to 90% of CML patients have a genetic translocation involving chromosomes 9 and 22 known as the Philadelphia chromosome. A blood film shows myeloblasts (granulocyte precursors) and granulocytosis. Patients with CML but without the Philadelphia chromosome have a worse prognosis than those with the translocation.

There are three stages of disease in CML: the chronic phase (responsive to treatment), the accelerated phase (where disease is difficult to control) and the blast crisis phase (where disease progresses into an acute leukaemia, usually AML). The median survival is 5 years. Death usually occurs within months of blast transformation (from bleeding and infection). Chemotherapy is often used, but allogeneic stem cell transplantation is the only hope of cure.

Chronic lymphocytic leukaemia (CLL) accounts for 30% of leukaemias in the over- 50s. It is characterized by the accumulation of incompetent CD5+ B cells that fail to secrete antibody (instead secreting light chains). Patients present with anaemia, painless lymphadenopathy, splenomegaly and infection. A blood film reveals a mature lymphocytosis (with anaemia and thrombocytopenia). The median survival is 6 years. CLL can occasionally transform into a high-grade aggressive lymphoma (Richter’s transformation).

113
Q

A 52-year-old woman presents to the GP with a 2-month history of worsening headaches. The headaches are worse in the morning, and are exacerbated by coughing and laughing. More recently, she has been complaining of blurred vision and nausea.

A.	 Cluster headache
B.	 Coitus-induced headache
C.	 Giant cell arteritis
D.	 Ice-cream headache
E.	 Meningitis
F.	 Migraine
G.	 Sagittal sinus thrombosis
H.	 Sinusitis
I.	 Space-occupying lesion
J.	 Tension headache
K.	 Trigeminal neuralgia
A

I – Space-occupying lesions

A constant, dull headache that is worse in the morning (after periods of being supine) and exacerbated by coughing, straining and laughing is typical of an underlying space-occupying lesion. The headache may be accompanied by nausea and vomiting, blurring of vision and focal neurological signs. Fundoscopy may show bilateral papilloedema. This history requires an urgent CT head scan to confirm the presence of a lesion.

114
Q

A 64-year-old woman presents to the emergency department with malaise and a left-sided headache that has been getting worse over the last 24 hours. She also complains of some pain in her shoulders. On examination, she is tender over the left side of her head. Fundoscopy is unremarkable.

A.	 Cluster headache
B.	 Coitus-induced headache
C.	 Giant cell arteritis
D.	 Ice-cream headache
E.	 Meningitis
F.	 Migraine
G.	 Sagittal sinus thrombosis
H.	 Sinusitis
I.	 Space-occupying lesion
J.	 Tension headache
K.	 Trigeminal neuralgia
A

C – Giant cell arteritis

Giant cell arteritis is an inflammatory vasculitis of the cranial branches of the arteries arising from the aorta. It is most common in the over-50s and twice as frequent in women. Patients present with malaise, fever, temporal headache, scalp tenderness and pain on chewing (jaw claudication). On examination, the temporal artery is tender, enlarged and non-pulsatile. Some cases are associated with polymyalgia rheumatica (proximal muscle pain and stiffness without weakness). Diagnosis is with temporal artery biopsy, which shows patchy granulomatous necrosis with giant cells. Not all of the artery may be affected, so a negative biopsy result does not rule out disease. Furthermore, arteritis of the ophthalmic artery can lead to ischaemic optic neuritis and permanent blindness, so it is important to treat this condition immediately if there is clinical suspicion before performing biopsy. Treatment is with oral prednisolone.

115
Q

A 32-year-old woman presents to the emergency department with a sudden-onset frontal headache and vomiting. Fundoscopy shows bilateral papilloedema, but examination is otherwise unremarkable. She has no significant medical history and takes the oral contraceptive pill. While in the department, she has a seizure.

A.	 Cluster headache
B.	 Coitus-induced headache
C.	 Giant cell arteritis
D.	 Ice-cream headache
E.	 Meningitis
F.	 Migraine
G.	 Sagittal sinus thrombosis
H.	 Sinusitis
I.	 Space-occupying lesion
J.	 Tension headache
K.	 Trigeminal neuralgia
A

G – Sagittal sinus thrombosis

Cerebral venous sinus thrombosis describes a clot in one of the dural venous sinuses of the brain. The clinical features can depend on the underlying location of the thrombosis. Presentation is generally with a sudden-onset headache, seizures and signs of raised intracranial pressure, e.g. papilloedema, hypertension and bradycardia. More specific symptoms may be seen with cavernous sinus thrombosis (proptosis, ptosis, ophthalmoplegia, reduced sensation in the first division of the trigeminal nerve) and transverse sinus thrombosis (hemiparesis).

Risk factors for the development of venous thromboses include contraceptive pill use, pregnancy, nephrotic syndrome and thrombophilias (protein S or C deficiency and antithrombin III deficiency). The diagnosis is confirmed by demonstrating the clot on CT or MRI. In sagittal sinus thrombosis, the ‘empty delta sign’ may be seen (a triangular area of enhancement within the sagittal sinus with a relatively low-attenuating centre). Management is with anticoagulation.

116
Q

A 32-year-old woman presents with a long history of worsening neck and shoulder pain associated with sensory loss in both upper limbs. On examination, the patient is insensate to pain and temperature in the distal upper limbs, although joint position sense is intact. There is no other motor or sensory loss.

A.	 Anterior cord syndrome
B.	 Brown-Séquard syndrome
C.	 Cauda equina syndrome
D.	 Central cord syndrome
E.	 Posterior cord syndrome
F.	 Syringomyelia
A

F – Syringomyelia

Syringomyelia describes the presence of a longitudinal fluid cavity (syrinx) within the spinal cord. These cavities are usually in the cervical segments, and disrupt the spinothalamic tracts. Patients present in their 20s or 30s with a segmental dissociated loss of spinothalamic function (i.e. spinothalamic function above and below the lesion is preserved). Dorsal column and motor function remain intact. When a syrinx affects the brain stem, the condition is called syringobulbia. Diagnosis of syringomyelia is by MRI, and management is by surgical decompression of the syrinx.

Syringomyelia may be associated with an Arnold–Chiari malformation, congenital herniation of the cerebellar tonsils through the foramen magnum at the base of the skull. Syringomyelia may also be caused by tumours of, or trauma to, the spinal cord.

117
Q

A 72-year-old woman attends the emergency department after falling down the stairs. She banged her forehead during the fall. On examination, she has bilateral upper limb weakness.

A.	 Anterior cord syndrome
B.	 Brown-Séquard syndrome
C.	 Cauda equina syndrome
D.	 Central cord syndrome
E.	 Posterior cord syndrome
F.	 Syringomyelia
A

D – Central cord syndrome

Central cord syndrome is the most common spinal cord lesion. It occurs in older people with cervical spondylosis who sustain a hyperextension injury. There is flaccid weakness of the arms, but motor and sensory fibres to the lower limb are comparatively preserved, as these are located more peripherally in the spinal cord. Central cord lesions have a fair prognosis.

118
Q

A 22-year-old builder falls from scaffolding and hits his head. He says that he felt his head tug backwards during the fall. On examination, there is no motor or sensory loss in the arms or legs, but he has difficulty coordinating his walking.

A.	 Anterior cord syndrome
B.	 Brown-Séquard syndrome
C.	 Cauda equina syndrome
D.	 Central cord syndrome
E.	 Posterior cord syndrome
F.	 Syringomyelia
A

E – Posterior cord syndrome

Hyperextension injuries can result in loss of dorsal column function (posterior cord syndrome). These injuries are very rare, and motor and sensory function is preserved. Gait is impaired due to impaired proprioception. The prognosis of posterior cord lesions is good.

Anterior cord syndrome occurs secondary to a flexion–compression injury. There is loss of neurological function of the anterior two-thirds of the spinal cord, namely the spinothalamic (pain and temperature) and corticospinal (motor) tracts. There is greater motor loss in the legs than in the arms. Anterior cord syndrome has the worst prognosis of all spinal cord lesions.

119
Q

A 14-year-old boy is brought to the GP with weakness in his left leg. This started 2 days ago and was preceded by a flu-like illness. On examination, the left leg is markedly weaker than the right. There is no evidence of cognitive impairment.

A.	 Cerebral abscess
B.	 Kuru
C.	 Meningitis
D.	 Poliomyelitis
E.	 Rabies
F.	 Subacute sclerosing panencephalitis
G.	 Tetanus
A

D – Poliomyelitis

Poliomyelitis is caused by the poliovirus. Infection occurs through the nasopharynx and is more common in developing countries. The virus infects the grey matter of the nervous system, especially the anterior horn cells in the lumbar region. Initial infection causes a mild fever and headache, progressing to aseptic meningitis. Weakness then starts in one muscle group and can progress to widespread paresis. Respiratory failure occurs if the intercostal muscles are affected.

Diagnosis is confirmed by culturing poliovirus from cerebrospinal fluid (CSF) or stool. Management is with bed rest (as exercise worsens or precipitates paralysis) and ventilation if required. Any muscle weakness that remains after 1 month of initial infection is likely to remain permanent. Prevention of poliomyelitis is with immunization by a live vaccine.

120
Q

A 28-year-old woman presents to the emergency department with a 3-day history of worsening headache, fever and drowsiness. Examination is unremarkable, but there is no evidence of cognitive impairment. While you are taking blood from her, she has a seizure.

A.	 Cerebral abscess
B.	 Kuru
C.	 Meningitis
D.	 Poliomyelitis
E.	 Rabies
F.	 Subacute sclerosing panencephalitis
G.	 Tetanus
A

A – Cerebral abscess

Bacteria can enter the cerebrum either via the blood or directly (from the paranasal sinuses or the middle ear). Features of cerebral abscesses include fever, headache, meningism and drowsiness. An abscess can also present chronically as a mass lesion (with seizures, focal neurological signs and raised intracranial pressure). A CT scan will demonstrate an abscess as an area of low density surrounded by a capsule. Treatment is with antibiotics, although a burr-hole aspiration may be needed. The mortality rate is 20%

121
Q

A 12-year-old boy is brought to the GP by his mother. She is worried that his performance at school has deteriorated markedly this term and that he does not seem himself anymore. He was previously well, but had measles as a child. Examination is unremarkable, but there is evidence of cognitive impairment.

A.	 Cerebral abscess
B.	 Kuru
C.	 Meningitis
D.	 Poliomyelitis
E.	 Rabies
F.	 Subacute sclerosing panencephalitis
G.	 Tetanus
A

F – Subacute sclerosing panencephalitis

Subacute sclerosing panencephalitis is a fatal disorder caused by the measles virus. It begins many years after the original measles infection with insidious intellectual deterioration, apathy, myoclonic jerks, rigidity and dementia. The EEG is distinctive (showing periodic bursts of triphasic waves). There is no effective treatment, and death occurs within years.

122
Q

A 65-year-old man presents with a lesion on his upper ear that has been present for months but that has now begun to ulcerate. On examination, there is a non-pigmented, hyperkeratotic, crusty lesion with raised everted edges on the pinna.

A.	 Acral lentiginous melanoma
B.	 Actinic keratosis
C.	 Amelanotic melanoma
D.	 Basal cell carcinoma
E.	 Bowen’s disease
F.	 Keratoacanthoma
G.	 Lentigo maligna melanoma
H.	 Nodular melanoma
I.	 Squamous cell carcinoma
J.	 Superficial spreading malignant melanoma
A

I – Squamous cell carcinoma

Squamous cell carcinoma (SCC) is a malignant tumour of keratinocytes that occurs in those aged over 50 in sun-damaged sites. Predisposing factors for their development include X-ray exposure, smoking, human papillomavirus and a genetic susceptibility. SCCs typically have raised everted edges with a central scab. Management is by surgical excision, with lymph node dissection or radiotherapy if there is evidence of spread.

An actinic keratosis (also known as a solar keratosis) is a hyperkeratotic, yellow– brown crusty lesion that occurs on sun-damaged sites. Although these lesions are benign, they are premalignant to SCC. For this reason, actinic keratoses should be removed, e.g. by excision, shaving or cryotherapy.

123
Q

A 37-year-old woman presents with a flat, irregular, pigmented lesion on her leg. This has grown in size in the past month, and occasionally bleeds.

A.	 Acral lentiginous melanoma
B.	 Actinic keratosis
C.	 Amelanotic melanoma
D.	 Basal cell carcinoma
E.	 Bowen’s disease
F.	 Keratoacanthoma
G.	 Lentigo maligna melanoma
H.	 Nodular melanoma
I.	 Squamous cell carcinoma
J.	 Superficial spreading malignant melanoma
A

J – Superficial spreading malignant melanoma

Malignant melanoma is a malignant tumour of melanocytes and is the most lethal of skin tumours. It is most common in white people living near the equator, and is twice as common in women. Risk factors include repeated ultraviolet exposure, previous malignant melanoma, multiple melanocytic naevi and having a large congenital naevus. Primary treatment is by surgical excision. Prognosis is related to tumour depth (Breslow score). Superficial spreading malignant melanoma is the most common type of malignant melanoma. It occurs most often in younger females on the leg. The tumour is macular with an irregular edge, and may itch or bleed.

124
Q

A 78-year-old woman has had a large pigmented lesion on her left cheek for some years. She presents to her GP because a thickened, pigmented, irregular lesion is growing on the edge of the previous one.

A.	 Acral lentiginous melanoma
B.	 Actinic keratosis
C.	 Amelanotic melanoma
D.	 Basal cell carcinoma
E.	 Bowen’s disease
F.	 Keratoacanthoma
G.	 Lentigo maligna melanoma
H.	 Nodular melanoma
I.	 Squamous cell carcinoma
J.	 Superficial spreading malignant melanoma
A

G – Lentigo maligna melanoma

The original lesion that this woman has on her cheek is a lentigo maligna (or Hutchinson’s malignant freckle) – a large irregular pigmented area that occurs most often in elderly people on sun-exposed skin. Lentigo maligna is associated with malignant transformation to melanoma (lentigo maligna melanoma). Thickening, darkening or ulceration within a lentigo maligna signals the onset of malignancy.

125
Q

A 12-year-old boy presents to the GP 2 weeks after contracting chickenpox. He has a non-blanching, palpable, purple rash all over his body and is bleeding from his gums. The full blood count shows a haemoglobin concentration of 14 g/dL, a platelet count of 40 × 109/L and a white cell count of 7 × 109/L. A clotting screen shows a normal PT and a normal APTT.

A.	 Aplastic anaemia
B.	 Disseminated intravascular coagulation
C.	 Essential thrombocythaemia
D.	 Haemophilia A
E.	 Haemophilia B
F.	 Heparin-induced thrombocytopenia
G.	 Idiopathic thrombocytopenic purpura
H.	 Pancytopenia secondary to bone marrow infiltration
I.	 Post-transfusion thrombocytopenia
J.	 Reactive thrombocytosis
K.	 Vitamin K deficiency
L.	 Von Willebrand disease
A

G – Idiopathic thrombocytopenic purpura

In idiopathic thrombocytopenic purpura (ITP), autoantibodies are formed against platelets, resulting in their premature destruction. This condition, which is usually seen shortly after a viral illness such as varicella-zoster infection, causes the platelet count to fall rapidly. The patient usually develops a purpuric rash and experiences bleeding from the skin and mucous membranes. In very severe cases, there may be more significant bleeding, including intracranial haemorrhage.

Investigation shows a normal prothrombin time (PT) and a normal activated partial thromboplastin time (APTT), since the clotting factors and clotting cascade are unaffected. The bleeding time is prolonged, as the thrombocytopenia reduces the rate of platelet activation and aggregation at the site of injury. The disease is usually self-limiting and requires no specific management. In severe disease, corticosteroids or immunoglobulin may be used.

126
Q

A 23-year-old woman presents to her GP complaining of easy bruising and heavy periods. Investigation reveals a mild iron-deficiency anaemia, a normal platelet count and a normal PT. The APTT is 50 s (normal 35–45 s). The bleeding time is 15 s (normal <7 s). Her father suffered from a bleeding disorder.

A.	 Aplastic anaemia
B.	 Disseminated intravascular coagulation
C.	 Essential thrombocythaemia
D.	 Haemophilia A
E.	 Haemophilia B
F.	 Heparin-induced thrombocytopenia
G.	 Idiopathic thrombocytopenic purpura
H.	 Pancytopenia secondary to bone marrow infiltration
I.	 Post-transfusion thrombocytopenia
J.	 Reactive thrombocytosis
K.	 Vitamin K deficiency
L.	 Von Willebrand disease
A

L – Von Willebrand disease

In von Willebrand’s disease, there is a quantitative or qualitative deficiency of von Willebrand factor (vWF), which is a protein that promotes platelet adhesion to the vascular wall and subsequent aggregation. It also plays a role in the transport of factor VIII and in preventing its premature destruction.

There are three main types of von Willebrand disease. In type 1 disease, there are reduced levels of normal vWF. Type 2 disease is due to the production of defective vWF. In type 3 disease, there is a total lack of vWF production and a significantly reduced level of factor VIII. Types 1 and 2 disease are autosomal dominant conditions. Type 3 disease is autosomal recessive. Types 1 and 2 disease are usually mild, with patients complaining of bruising, epistaxis and menorrhagia. Type 3 disease is more severe, and may cause bleeding into joints and soft tissues (haemarthrosis).

Investigation of vWF shows a normal platelet level and a normal PT. APTT may be slightly prolonged due to reduced levels of factor VIII in severe disease. The bleeding time is increased, as platelets fail to adhere to the vessel wall and aggregate.

127
Q

A 65-year-old man attends the GP for lifestyle advice. He is being treated for high blood pressure but is otherwise well. A random capillary blood glucose is 13.0 mmol/L.

A.	 Acromegaly
B.	 Acute pancreatitis
C.	 Corticosteroid therapy
D.	 Cushing’s syndrome
E.	 Liver cirrhosis
F.	 Polycystic ovarian syndrome
G.	 Ramipril
H.	 Thiazide diuretics
I.	 Viral hepatitis
A

H – Thiazide diuretics

Thiazide diuretics, such as bendroflumethiazide, are known to trigger hyperglycaemia and exacerbate diabetes. The risk is more prominent in patients taking higher doses. Other classes of drug that may induce diabetes include β-blockers, antipsychotics and steroids.

128
Q

A 34-year-old man has lost a significant amount of blood following a motorbike accident. His blood test results include a serum urea of 20.2 mmol/L, a creatinine of 220 µmol/L, a potassium of 5.7 mmol/L and a sodium of 137 mmol/L. Urine electrolytes and osmolarity are normal. A CT scan confirms that there was no damage to the kidneys during the accident.

A. Acute tubular necrosis
B. Analgesia nephropathy
C. Haemolytic uraemic syndrome
D. Idiopathic thrombocytopenic purpura
E. Interstitial renal failure secondary to glomerular nephritis
F. Interstitial renal failure secondary to non-steroidal anti-inflammatory drugs
G. Interstitial renal failure secondary to nephrotoxic antibiotics
H. Post-renal acute renal failure secondary to prostatic hypertrophy
I. Post-renal acute renal failure secondary to renal calculi
J. Pre-renal acute renal failure secondary to cardiac failure
K. Pre-renal acute renal failure secondary to coarctation of the aorta
L. Pre-renal acute renal failure secondary to hypovolaemia
M. Pre-renal acute renal failure secondary to renal artery stenosis

A

L – Pre-renal acute renal failure secondary to hypovolaemia

Acute renal failure can be defined as the deterioration of renal function over hours or days. By far the most common cause of acute renal failure is hypoperfusion of the kidney, referred to as pre-renal acute renal failure. This is usually seen with hypovolaemia secondary to conditions such as acute blood loss, vomiting, diarrhoea and burns. It may also occur in normovolaemic patients with sepsis, cardiac failure or renal artery stenosis. Investigation demonstrates high serum urea, creatinine and potassium concentrations, and a poor urine output. Prerenal acute renal failure is treated by correcting the underlying condition and restoring the patient’s circulating volume with intravenous fluids.

If the kidneys are hypoperfused for a significant amount of time, autoregulatory mechanisms within the renal vasculature fail, causing tubular cell damage and death. This condition is known as acute tubular necrosis (ATN) and results in failure of active sodium reabsorption and failure to concentrate the urine. The urine produced in ATN typically has a high sodium concentration, a low osmolarity, a low urine: serum urea ratio and a low urine:plasma osmolarity ratio. ATN usually resolves if the underlying cause is corrected and appropriate fluid therapy is delivered, although recovery may take weeks.

129
Q

A 14-year-old boy has a 3-day history of diarrhoea and vomiting. Blood tests show a serum urea of 23.1 mmol/L, a creatinine of 400 µmol/L, a potassium of 5.9 mmol/L and a sodium of 134mmol/L. The full blood count and blood film show a normocytic anaemia with fractionated red blood cells. A stool sample later confirms the presence of Escherichia coli O157.

A. Acute tubular necrosis
B. Analgesia nephropathy
C. Haemolytic uraemic syndrome
D. Idiopathic thrombocytopenic purpura
E. Interstitial renal failure secondary to glomerular nephritis
F. Interstitial renal failure secondary to non-steroidal anti-inflammatory drugs
G. Interstitial renal failure secondary to nephrotoxic antibiotics
H. Post-renal acute renal failure secondary to prostatic hypertrophy
I. Post-renal acute renal failure secondary to renal calculi
J. Pre-renal acute renal failure secondary to cardiac failure
K. Pre-renal acute renal failure secondary to coarctation of the aorta
L. Pre-renal acute renal failure secondary to hypovolaemia
M. Pre-renal acute renal failure secondary to renal artery stenosis

A

C – Haemolytic uraemic syndrome

In haemolytic uraemic syndrome (HUS), there is acute renal failure, microangiopathic haemolytic anaemia and thrombocytopenia. This condition is classically seen in childhood following gastroenteritis caused by Escherichia coli O157, which produces nephrotoxic verotoxins. Other gastrointestinal and respiratory tract pathogens may also cause HUS, but are seen less frequently. The haemolytic anaemia and thrombocytopenia are due to erythrocyte and platelet destruction within fibrin meshes that are deposited on affected endothelial membranes.

Other features of HUS include haematuria, proteinuria, purpuric rash and central nervous system complications. Treatment of HUS is supportive. Some patients require temporary peritoneal dialysis or haemodialysis. Occasionally, renal function is permanently compromised, necessitating longterm renal replacement therapy and renal transplantation.

130
Q

A 55-year-old man develops severe vomiting and watery diarrhoea 3 hours after eating fried rice from a late-night takeaway. Immediately before this, he was well.

A.	 Bacillus cereus
B.	 Campylobacter jejuni
C.	 Clostridium botulinum
D.	 Clostridium difficile
E.	 Escherichia coli
F.	 Rotavirus
G.	 Staphylococcus aureus
H.	 Shigella sonnei
A

A – Bacillus cereus

Bacillus cereus is a spore-producing bacterium that can induce diarrhoea and vomiting within hours of ingestion. Infection is usually due to consuming rewarmed rice that has been contaminated with spores, e.g. rice that has been boiled and left several hours before frying (hence the term ‘Chinese fried rice syndrome’). It is usually a relatively short-lived infection and clears up within 24 hours. Antibiotics are not indicated.

131
Q

A 16-year-old boy presents to his GP with vomiting that developed hours after eating a rewarmed rare steak. He denies diarrhoea.

A.	 Bacillus cereus
B.	 Campylobacter jejuni
C.	 Clostridium botulinum
D.	 Clostridium difficile
E.	 Escherichia coli
F.	 Rotavirus
G.	 Staphylococcus aureus
H.	 Shigella sonnei
A

G – Staphylococcus aureus

Staphylococcus aureus food poisoning is caused by heat-resistant enterotoxins that interact with the vagus nerve, causing vomiting within hours of ingestion. Diarrhoea is not usually a feature. The illness is self-limiting and requires symptomatic treatment only.

132
Q

A 3-year-old boy develops diarrhoea and vomiting the week before Christmas. A number of the other children at his nursery and his older sister are affected by a similar illness.

A.	 Bacillus cereus
B.	 Campylobacter jejuni
C.	 Clostridium botulinum
D.	 Clostridium difficile
E.	 Escherichia coli
F.	 Rotavirus
G.	 Staphylococcus aureus
H.	 Shigella sonnei
A

F – Rotavirus

Rotavirus infection commonly occurs in children and hospitalized patients during the winter period. It spreads readily from person to person and has an incubation period of 2 days. The patient may have a prodrome of upper respiratory tract illness followed by development of diarrhoea and vomiting. The illness is selflimiting and usually requires no treatment other than supportive measures. Some children and frail adults require intravenous fluids to treat dehydration and electrolyte imbalance that is not responding to oral rehydration.

133
Q

A 21-year-old woman is sitting in the pub with friends. At one point, her friends notice that she is staring blankly into space, licking her lips. She remains sitting in her chair, and at no point does she begin fitting. After a few minutes, she returns to normal. She denies being aware of what happened.

A.	 Anoxic seizure
B.	 Complex partial seizure
C.	 Febrile convulsion
D.	 Narcolepsy
E.	 Partial motor seizure
F.	 Partial sensory seizure
G.	 Pseudo-seizure
H.	 Status epilepticus
I.	 Tonic–clonic seizure
J.	 Tonic seizure
K.	 Versive seizure
A

B – Complex partial seizure

A simple partial seizure is one where consciousness is preserved. In complex partial seizures, consciousness is affected. Patients having a complex partial seizure stop what they are doing, stare blankly and may display automatisms (e.g. lip smacking). This may be preceded by complex hallucinations. After a few minutes, consciousness returns and the patient is drowsy.

134
Q

A 47-year-old woman presents with a 2-month history of hip and shoulder weakness. She is finding it especially difficult to climb stairs. On examination, you notice a pink, scaly lesion on her knuckles.

A.	 Angiography
B.	 Anti-U1-RNP
C.	 cANCA
D.	 Joint aspiration and culture
E.	 Joint aspiration and microscopy
F.	 Muscle biopsy
G.	 Rheumatoid factor
H.	 Schirmer’s test
I.	 Schober’s test
J.	 Spine X-ray
K.	 Temporal artery biopsy
A

F – Muscle biopsy

This woman has dermatomyositis, a connective tissue disease characterized by proximal muscle weakness (hips and shoulders) with characteristic skin lesions (a purple rash on the eyelids and scaly pink papules on the knuckles). The best investigation is a muscle biopsy, which demonstrates inflammation and necrosis.

135
Q

A 22-year-old woman presents with a long history of dry, itchy eyes and a dry mouth. She denies having any problems with her vision. There is no abnormality on examination.

A.	 Angiography
B.	 Anti-U1-RNP
C.	 cANCA
D.	 Joint aspiration and culture
E.	 Joint aspiration and microscopy
F.	 Muscle biopsy
G.	 Rheumatoid factor
H.	 Schirmer’s test
I.	 Schober’s test
J.	 Spine X-ray
K.	 Temporal artery biopsy
A

H – Schirmer’s test

This woman has Sjögren’s syndrome, characterized by dry eyes (keratoconjunctivitis sicca) and a dry mouth (xerostoma). Diagnosis is with Schirmer’s test. A 35 mm-long piece of filter paper is placed under the lower eyelid for 5 minutes – if less than 10 mm becomes moist, this indicates Sjögren’s syndrome.

136
Q

A 29-year-old woman presents with a 3-month history of general malaise, fever and weight loss. Over the last week, she has developed an intermittent cramping pain in her right arm. On examination, the upper limbs appear normal, but the radial pulses are not palpable.

A.	 Angiography
B.	 Anti-U1-RNP
C.	 cANCA
D.	 Joint aspiration and culture
E.	 Joint aspiration and microscopy
F.	 Muscle biopsy
G.	 Rheumatoid factor
H.	 Schirmer’s test
I.	 Schober’s test
J.	 Spine X-ray
K.	 Temporal artery biopsy
A

A – Angiography

This woman has Takayasu’s arteritis, a vasculitis of the aorta and the other major arteries. Inflammation causes stenosis of the arteries, which leads to absence of distal pulses and limb claudication. Angiography will demonstrate stenosis of the affected vessels.

137
Q

A 61-year-old man presents with sudden-onset, severe pain in his left toe, which occurred at rest. He is otherwise well. On examination, the small joint of the great toe is swollen, red and tender.

A.	 Angiography
B.	 Anti-U1-RNP
C.	 cANCA
D.	 Joint aspiration and culture
E.	 Joint aspiration and microscopy
F.	 Muscle biopsy
G.	 Rheumatoid factor
H.	 Schirmer’s test
I.	 Schober’s test
J.	 Spine X-ray
K.	 Temporal artery biopsy
A

E – Joint aspiration and microscopy

The likely diagnosis in this man is acute gout. The diagnosis can be confirmed by performing a joint aspiration and sending for microscopy. The presence of negatively birefringent, needle-shaped crystals is indicative of gout. In practice, you would send some joint fluid for culture anyway to rule out septic arthritis.

138
Q

A 28-year-old woman goes to her GP following a sudden, sharp pain in her neck earlier in the day. Since then, she has found it difficult to breathe. On examination, you find a swelling on the right side of her neck. The patient tells you that this swelling has been there for some time and has never previously caused her any trouble.

A.	 Anaplastic carcinoma
B.	 De Quervain’s thyroiditis
C.	 Follicular carcinoma
D.	 Graves’ disease
E.	 Hashimoto’s thyroiditis
F.	 Haemorrhage into a cyst
G.	 Medullary carcinoma
H.	 Myxoedema coma
I.	 Primary myxoedema
J.	 Thyroid storm
K.	 Toxic multinodular goitre
A

F – Haemorrhage into a cyst

This woman has a history of a thyroid cyst. The acute presentation of neck pain and growth of the lesion suggest haemorrhage into the cyst. Haemorrhage can result in tracheal compression and stridor. If possible, aspiration of the cyst contents should be performed to alleviate tracheal compression and maintain airway patency. If aspiration is not possible, surgical intervention may be required.

139
Q

A 30-year-old woman with a known congenital heart defect presents to her GP with a 4-week history of left-sided chest pain and palpitations that are worse on exertion. The pain does not radiate, has no other associated symptoms and is not worse on inspiration. On examination, there is a mid-systolic click best heard at the apex on auscultation of the heart.

A.	 Acute myocardial infarction
B.	 Aortic dissection
C.	 Boerhaave’s syndrome
D.	 Decubitus angina
E.	 Gastro-oesophageal reflux disease
F.	 Herpes zoster
G.	 Mitral valve prolapse
H.	 Oesophageal spasm
I.	 Pericarditis
J.	 Pulmonary oedema
K.	 Pulmonary embolism
L.	 Stable angina
M.	Tietze’s syndrome
A

G – Mitral valve prolapse

Mitral valve prolapse is a relatively common condition that tends to affect young women and those with a history of congenital heart disease such as atrial septal defect and patent ductus arteriosus. Many people with mitral valve prolapse are asymptomatic. Those who experience symptoms tend to complain of atypical chest pain and palpitations that are exacerbated by exercise. Less common complications include postural hypotension, syncope, bacterial endocarditis, cerebral and systemic emboli, and ventricular arrhythmia.

On auscultation, there may be a midsystolic click, which is sometimes associated with a late-systolic murmur. Diagnosis is by echocardiography. Treatment is mainly symptomatic with ‘pill in the pocket’ β-blockers that are taken during episodes of pain and palpitations. Due to the increased risk of bacterial endocarditis, patients with mitral valve prolapse should receive antibiotic prophylaxis prior to invasive procedures.

140
Q

A 70-year-old man presents to his GP following an episode of central crushing chest pain that started 30 minutes after going to bed. The pain did not radiate and there were no associated symptoms. Following two puffs of sublingual glyceryl trinitrate spray, his chest pain resolved. The ECG shows no abnormalities. This is the third time in the last 2 weeks that these symptoms have occurred on going to bed.

A.	 Acute myocardial infarction
B.	 Aortic dissection
C.	 Boerhaave’s syndrome
D.	 Decubitus angina
E.	 Gastro-oesophageal reflux disease
F.	 Herpes zoster
G.	 Mitral valve prolapse
H.	 Oesophageal spasm
I.	 Pericarditis
J.	 Pulmonary oedema
K.	 Pulmonary embolism
L.	 Stable angina
M.	Tietze’s syndrome
A

D – Decubitus angina

Decubitus angina describes typical angina pain on lying flat at night time. The reabsorption of peripheral fluid into the venous circulation that occurs while lying flat places additional strain on the heart. The diseased coronary arteries are unable to deliver adequate blood to supply the increased myocardial oxygen demand. Decubitus angina occurs when anti-angina therapy is suboptimal and when coronary artery disease is severe.

141
Q

A 70-year-old man presents to the emergency department with retrosternal chest pain that is crushing in nature. This has happened on several occasions previously, and usually occurs after drinking hot fluids. Despite there being no ECG changes, he is given glyceryl trinitrate, which partially relieves the pain. After 30 minutes, the pain relieves spontaneously.

A.	 Acute myocardial infarction
B.	 Aortic dissection
C.	 Boerhaave’s syndrome
D.	 Decubitus angina
E.	 Gastro-oesophageal reflux disease
F.	 Herpes zoster
G.	 Mitral valve prolapse
H.	 Oesophageal spasm
I.	 Pericarditis
J.	 Pulmonary oedema
K.	 Pulmonary embolism
L.	 Stable angina
M.	Tietze’s syndrome
A

H – Oesophageal spasm

Oesophageal spasm is rare, and is often confused with angina. It classically presents with retrosternal chest pain, with or without dysphagia. It is caused by abnormal peristaltic contraction in the distal half of the oesophagus, which may occur spontaneously or secondary to precipitating factors such as hot food. The pain is very similar to angina and is classically partially relieved by nitrates and calcium channel blockers.

Twenty-four-hour oesophageal manometry may show paroxysmal oesophageal contractions. Barium swallow may demonstrate a ‘corkscrew oesophagus’ caused by the abnormal peristalsis, although this is seen only during active spasm. Conservative treatment is with nitrates or calcium channel blockers. In severe disease, botulinum toxin may be injected into the oesophagus to provide temporary relief. Surgical treatment is indicated in disabling disease.

142
Q

A 40-year-old woman has a long history of shaking in her hands. She finds that it is worse with writing, and this is interfering with her job. Alcohol relieves the tremor, and she is worried that she may develop alcohol dependence.

A.	 Akathisia
B.	 Asterixis
C.	 Athetosis
D.	 Chorea
E.	 Dystonia
F.	 Essential tremor
G.	 Hemiballismus
H.	 Intention tremor
I.	 Physiological tremor
J.	 Resting tremor
K.	 Rubral tremor
L.	 Tardive dyskinesia
A

F – Essential tremor

Tremors are rhythmic oscillating movements of a limb, part of a limb or the head. Essential tremors are slow, are often familial, and may be made more obvious during certain actions such as writing. Essential tremors are characteristically improved with alcohol. β-Blockers can be used to improve such tremors.

Physiological tremors are seen in normal people, and have a frequency of 8– 15Hz. Physiological tremors can be exaggerated with anxiety, fatigue, fever and alcohol withdrawal.

The resting tremor is pathognomonic of Parkinson’s disease and has a frequency of 4–6 Hz. It is characteristically pill-rolling and is usually asymmetrical.

143
Q

A 43-year-old woman with multiple sclerosis has started to get involuntary, violent tremors in her hands for no reason. These can come on at any time and are socially disabling, as they often result in dropping or spilling whatever she is holding.

A.	 Akathisia
B.	 Asterixis
C.	 Athetosis
D.	 Chorea
E.	 Dystonia
F.	 Essential tremor
G.	 Hemiballismus
H.	 Intention tremor
I.	 Physiological tremor
J.	 Resting tremor
K.	 Rubral tremor
L.	 Tardive dyskinesia
A

K – Rubral tremor

Lesions in the superior cerebellar peduncle cause a rubral tremor – a violent, large-amplitude, postural tremor that worsens as the target is approached. It is common in advanced multiple sclerosis and can be disabling.

144
Q

A 56-year-old man presents to the GP with a 2-month history of excessive urine production, constipation and general lethargy. A set of blood tests is requested, and shows a corrected calcium of 2.8 mmol/L (reference range 2.2–2.6 mmol/L) and a significantly raised parathyroid hormone.

A.	 Bone metastases
B.	 Ectopic parathyroid hormone secretion
C.	 Hypomagnesaemia
D.	 Hypoparathyroidism
E.	 Hypothyroidism
F.	 Iatrogenic calcium overdose
G.	 Parathyroid gland malignancy
H.	 Parathyroid hormone-related protein secretion
I.	 Primary hyperparathyroidism	
J.	 Pseudohypoparathyroidism
K.	 Pseudo-pseudohypoparathyroidism
L.	 Secondary hyperparathyroidism
M.	Tertiary hyperparathyroidism
A

I – Primary hyperparathyroidism

Primary hyperparathyroidism is one of the most common causes of nonmalignant hypercalcaemia. The symptoms of hypercalcaemia include lethargy, polyuria, polydipsia, peptic ulcer disease, stone formation and depression. Patients with very high serum calcium levels are at risk of cardiac arrest.

In health, calcium metabolism is largely regulated by parathyroid hormone (PTH), which is secreted from the parathyroid glands. PTH increases serum calcium concentration by promoting bone resorption, renal phosphate excretion and vitamin D synthesis. The net effect of PTH is a rise in serum calcium and a fall in serum phosphate. Primary hyperparathyroidism is usually due to a single parathyroid adenoma that autonomously secretes excessive amounts of PTH. It is diagnosed by identifying hypercalcaemia in the presence of raised PTH levels. Treatment is by parathyroidectomy. Significant hypercalcaemia should be treated with fluid resuscitation and bisphosphonates.

145
Q

A 26-year-old woman with chronic renal failure secondary to renal dysplasia has a corrected calcium of 2.6 mmol/L, a phosphate of 2.1 mmol/L and a significantly raised parathyroid hormone.

A.	 Bone metastases
B.	 Ectopic parathyroid hormone secretion
C.	 Hypomagnesaemia
D.	 Hypoparathyroidism
E.	 Hypothyroidism
F.	 Iatrogenic calcium overdose
G.	 Parathyroid gland malignancy
H.	 Parathyroid hormone-related protein secretion
I.	 Primary hyperparathyroidism	
J.	 Pseudohypoparathyroidism
K.	 Pseudo-pseudohypoparathyroidism
L.	 Secondary hyperparathyroidism
M.	Tertiary hyperparathyroidism
A

M – Tertiary hyperparathyroidism

Tertiary hyperparathyroidism occurs following a prolonged period of secondary hyperparathyroidism. The parathyroid glands hypertrophy and begin to secrete PTH autonomously, i.e. this is no longer a physiological response to negative feedback (as in secondary hyperparathyroidism).

The patient will usually have a high serum calcium level and a high PTH level. It is best to avoid the development of tertiary hyperparathyroidism by the correct and early management of secondary hyperparathyroidism. In severe disease, it is possible to remove the majority of the parathyroid tissue to reduce hormone levels.

146
Q

A 46-year-old man presents to the GP with a history of shortness of breath that occurs on exertion and when lying down at night. On general inspection, he notices multiple spider naevi on his chest and face. On examination of his heart, the apex beat is displaced laterally and there is a pansystolic murmur that is difficult to identify since the heartbeat is irregularly irregular.

A.	 Arrhythmogenic right ventricular cardiomyopathy
B.	 Atrial myxoma
C.	 Cardiac tamponade
D.	 Constrictive cardiomyopathy
E.	 Dilated cardiomyopathy
F.	 Hypertrophic obstructive cardiomyopathy
G.	 Myocarditis
H.	 Non-ST-elevation myocardial infarction
I.	 Restrictive cardiomyopathy
J.	 Subacute bacterial endocarditis
K.	 Uraemic pericarditis
L.	 Viral pericarditis
A

E – Dilated cardiomyopathy

In dilated cardiomyopathy, the left ventricle becomes dilated and inefficient, placing the patient at risk of arrhythmia (particularly atrial fibrillation), embolic disease and heart failure. The condition is occasionally familial, but is usually acquired and often secondary to excess alcohol consumption, ischaemic heart disease, metabolic disease, myocarditis and primary muscular diseases, including muscular dystrophy. Treatment is largely symptomatic, with arrhythmias and heart failure being treated as usual. Patients with dilated cardiomyopathy require anticoagulation to reduce the risk of embolic disease. Those who are known to experience dangerous ventricular arrhythmia require an internal cardiac defibrillator. Cardiac transplantation is a last resort for disease that is refractory to maximum medical therapy.

147
Q

A 64-year-old woman is referred for echocardiography following a history of weight loss, low-grade fever, non-specific chest pain and palpitations. While performing the echocardiogram, the technician notices a polypoid mass attached to the septal wall in the left atrium.

A.	 Arrhythmogenic right ventricular cardiomyopathy
B.	 Atrial myxoma
C.	 Cardiac tamponade
D.	 Constrictive cardiomyopathy
E.	 Dilated cardiomyopathy
F.	 Hypertrophic obstructive cardiomyopathy
G.	 Myocarditis
H.	 Non-ST-elevation myocardial infarction
I.	 Restrictive cardiomyopathy
J.	 Subacute bacterial endocarditis
K.	 Uraemic pericarditis
L.	 Viral pericarditis
A

B – Atrial myxoma

Atrial myxoma, despite being relatively rare, is the most common tumour of the heart. It is a benign gelatinous neoplasm and is predominantly found attached to the septal wall of the left atrium by a pedicle. Symptoms include chest pain, dyspnoea, fever, lethargy and weight loss. Auscultation my reveal a loud first heart sound and a diastolic ‘plop’ due to prolapse of the tumour through the mitral valve. The presentation is similar to infective endocarditis and can cause diagnostic difficulties. Atrial myxomas can often embolize (with typical embolic consequences), but they do not metastasize. Echocardiography is the investigation of choice, and usually identifies a cystic polypoid mass. Treatment is by surgical excision. If removal is not complete, myxoma may recur.

The Carney complex is an autosomal dominant condition characterized by atrial and cutaneous myxomas, spotty skin hyperpigmentation and endocrine overactivity. It is associated with around 10% of cases of atrial myxoma.

148
Q

A 46-year-old woman presents to the emergency department with a short history of worsening shortness of breath and a reduced exercise tolerance. She has been suffering with lethargy, muscle aches, fever and palpitations for 2 weeks. Routine blood tests show a raised white blood cell count and a raised C-reactive protein. A plasma troponin returns at 1.2 ng/mL.

A.	 Arrhythmogenic right ventricular cardiomyopathy
B.	 Atrial myxoma
C.	 Cardiac tamponade
D.	 Constrictive cardiomyopathy
E.	 Dilated cardiomyopathy
F.	 Hypertrophic obstructive cardiomyopathy
G.	 Myocarditis
H.	 Non-ST-elevation myocardial infarction
I.	 Restrictive cardiomyopathy
J.	 Subacute bacterial endocarditis
K.	 Uraemic pericarditis
L.	 Viral pericarditis
A

G – Myocarditis

Myocarditis is most commonly due to viral infection with Coxsackie B virus. Other causes include autoimmune disease, rheumatic fever, diphtheria, and a multitude of other viral, bacterial and fungal pathogens. Presentation is usually with a period of general illness, lethargy, fever and palpitations, followed by symptoms of left ventricular failure. Definitive diagnosis is often difficult, as evidence of infection is rarely found. Plasma troponin and cardiac enzymes are elevated in proportion to the extent of damage. Endomyocardial biopsy is sometimes employed to aid diagnosis.

Treatment is predominantly symptomatic, including management of congestive cardiac failure and arrhythmias. Patients are advised to avoid exertion during the illness, as this may precipitate fatal ventricular arrhythmias. Some people recover fully, while others have a prolonged illness resulting in dilated cardiomyopathy. A number of patients die in the acute stages due to arrhythmias, congestive cardiac failure and refractory cardiogenic shock.

149
Q

A 38-year-old woman complains of a 6-week history of worsening headaches. This morning, she noticed that she was occasionally getting double vision, which was most noticeable when she looked towards the left side.

A.	 Abducens nerve palsy
B.	 Oculomotor nerve palsy
C.	 Optic chiasm lesion
D.	 Optic nerve lesion
E.	 Optic tract lesion
F.	 Parietal lobe lesion
G.	 Temporal lobe lesion
H.	 Trochlear nerve palsy
A

A – Abducens nerve palsy

This woman may have a brain tumour. The consequent raised intracranial pressure can result in compression of the abducens nerve on the temporal bone. The abducens nerve innervates the lateral rectus, the muscle that abducts the eye. A palsy of this nerve results in diplopia on gazing laterally towards the side of the lesion, since there is no abduction of the eye on this side. In this scenario, the patient experiences diplopia on left lateral gaze, signifying a left abducens nerve palsy.

150
Q

A 67-year-old woman has noticed some problems with her vision. Over the last few days, when her pet parrot has flown down towards her from the right side, she has not noticed him. On examination, she is blind in the upper right quadrants of the visual field in both eyes.

A.	 Abducens nerve palsy
B.	 Oculomotor nerve palsy
C.	 Optic chiasm lesion
D.	 Optic nerve lesion
E.	 Optic tract lesion
F.	 Parietal lobe lesion
G.	 Temporal lobe lesion
H.	 Trochlear nerve palsy
A

G – Temporal lobe lesion

The upper fibres from the optic tract travel through the temporal lobe and the lower fibres pass through the parietal lobe. Whereas lesions in the optic tract cause a contralateral homonymous hemianopia (see above), a lesion in the temporal lobe will result in a contralateral homonymous upper quadrantopia (loss of the upper quarter of the visual field on the same side in both eyes).

Similarly, parietal lobe lesions cause a contralateral homonymous lower quadrantopia. Occipital lobe lesions result in a homonymous hemianopia, often with macular sparing.

151
Q

A 37-year-old woman with coeliac disease develops an intensely itchy blistering rash on her wrists and forearms.

A.	 Angular stomatitis
B.	 Dermatitis herpetiformis
C.	 Episcleritis
D.	 Erythema multiform
E.	 Erythema nodosum
F.	 Leukoplakia
G.	 Pyoderma gangrenosum
H.	 Sister Joseph nodule
I.	 Virchow’s node
A

B – Dermatitis herpetiformis

Dermatitis herpetiformis is an intensely itchy, blistering rash that is associated with coeliac disease. It is usually found on the extensor surfaces of the elbows and knees, but can also be found on the back, buttocks and forehead. Treatment usually involves the implementation of a gluten-free diet in combination with oral dapsone and topical corticosteroids.

152
Q

A 52-year-old woman presents to her GP with a raised, pink papule on her left arm. On examination, you note that the lesion is painless and firm and arises from an underlying scar. She mentions that the scar was from a burn injury that happened 20 years ago.

A.	 Arterial ulcer
B.	 Curling’s ulcer
C.	 Cushing’s ulcer
D.	 Marjolin’s ulcer
E.	 Martorell’s ulcer
F.	 Necrobiosis lipoidica
G.	 Neuropathic ulcer
H.	 Pyoderma gangrenosum
I.	 Rodent ulcer
J.	 Syphilitic ulcer
K.	 Venous ulcer
A

D – Marjolin’s ulcer

Marjolin’s ulcer is the development of a squamous cell carcinoma occurring in an area of scarred or traumatized skin, such as a burn injury, chronic wound or venous ulcer. Lesions appear as raised, fleshy, firm papules that grow slowly. Treatment is by wide local excision.

153
Q

A 67-year-old man attends his GP with a cramping pain in his buttocks that occurs on walking. When he develops the cramp, he has to rest until it disappears. On examination, it is difficult to feel his distal lower limb pulses. On the dorsal left foot there is a deep, sharply defined ulcer that the patient finds painful.

A.	 Arterial ulcer
B.	 Curling’s ulcer
C.	 Cushing’s ulcer
D.	 Marjolin’s ulcer
E.	 Martorell’s ulcer
F.	 Necrobiosis lipoidica
G.	 Neuropathic ulcer
H.	 Pyoderma gangrenosum
I.	 Rodent ulcer
J.	 Syphilitic ulcer
K.	 Venous ulcer
A

A – Arterial ulcer

This man has features of intermittent claudication (cramping in the lower limbs on exertion). This is due to underlying ischaemia (from arterial disease). Other features of ischaemia include cold feet, hair loss, toenail dystrophy, dusky cyanosis and ischaemic ulceration. Arterial ulcers are deep, painful and sharply defined, and usually occur on the shin or foot. The peripheral pulses may be reduced or absent on examination. Contrast angiography will help define arterial lesions, which may be improved by angioplasty or vascular reconstruction.

Venous ulcers occur most often in women after middle age. They occur over a background of deep venous insufficiency. There are many stages of skin changes in venous disease, beginning with oedema and a brown discoloration of the skin. The brown colour comes from haemosiderin deposits, which occur secondary to extravasation of red cells from leaky capillaries. The next stages are an eczema-like appearance, with hardening and constriction of the skin around the ankle (lipodermatosclerosis). The tightening of the skin around the ankle that occurs with lipodermatosclerosis, along with the oedema of the leg proximal to this, results in an ‘inverted champagne bottle’ appearance. Ulceration of the affected skin often follows trauma and usually affects the medial gaiter area (the gaiter area stretches from the ankle to the proximal calf, much like the gaiters used in hiking to prevent muddy water from getting into boots).

154
Q

A 23-year-old woman presents with a history of diarrhoea associated with the passage of blood and mucus. She claims to feel unwell and tired and to have lost weight in the recent months. On examination, you notice a deep, necrotic ulcer on her leg with dark-red edges.

A.	 Arterial ulcer
B.	 Curling’s ulcer
C.	 Cushing’s ulcer
D.	 Marjolin’s ulcer
E.	 Martorell’s ulcer
F.	 Necrobiosis lipoidica
G.	 Neuropathic ulcer
H.	 Pyoderma gangrenosum
I.	 Rodent ulcer
J.	 Syphilitic ulcer
K.	 Venous ulcer
A

H – Pyoderma gangrenosum

Pyoderma gangrenosum is a skin condition that is associated with inflammatory bowel disease (as in this patient), rheumatoid arthritis and myeloid blood dyscrasias (e.g. acute and chronic myeloid leukaemias). It initially appears as purple papules, which enlarge and break down to become deep, necrotic ulcers with a dark-red border. Pyoderma gangrenosum is most common on the legs, but can develop anywhere.

155
Q

A 76-year-old man with known ischaemic heart disease is referred for a 24-hour ECG tape after suffering occasional palpitations. The tape shows periods of sinus bradycardia followed by episodes of sinus tachycardia.

A.	 Atrioventricular node re-entry tachycardia
B.	 Chronic atrial fibrillation
C.	 Multifocal atrial tachycardia
D.	 Paroxysmal atrial fibrillation
E.	 Sick sinus syndrome
F.	 Sinus tachycardia
G.	 Torsades de pointes
H.	 Ventricular extrasystoles
I.	 Ventricular fibrillation	
J.	 Wolff–Parkinson–White syndrome
A

E – Sick sinus syndrome

Sick sinus syndrome is caused by a dysfunctional sinoatrial node, usually secondary to age-related fibrosis. It is often diagnosed using a 24-hour ECG tracing requested for patients experiencing palpitations or symptomatic bradycardia such as dizzy spells and syncope. The tracing usually shows sinus bradycardia, atrioventricular block or asystole interspersed with periods of supraventricular tachycardia, such as junctional tachycardias or atrial fibrillation. There is no specific treatment. Although pacemaker insertion may be indicated in cases with severe symptoms, this does not affect prognosis. Anticoagulation is recommended in patients at risk of thromboembolic disease.

156
Q

A 55-year-old man is brought to the emergency department following a sudden collapse while gardening. He is unconscious and unresponsive. He has no palpable pulse or respiratory effort. A heart monitor shows a disorganized, broad, complex, irregular rhythm with a fluctuating baseline.

A.	 Atrioventricular node re-entry tachycardia
B.	 Chronic atrial fibrillation
C.	 Multifocal atrial tachycardia
D.	 Paroxysmal atrial fibrillation
E.	 Sick sinus syndrome
F.	 Sinus tachycardia
G.	 Torsades de pointes
H.	 Ventricular extrasystoles
I.	 Ventricular fibrillation	
J.	 Wolff–Parkinson–White syndrome
A

I – Ventricular fibrillation

In ventricular fibrillation, the ventricles contract ineffectively at a rate of 300–600 beats/min. The ECG tracing shows a rapid, irregular rhythm with a fibrillating baseline. The speed and haphazard nature of ventricular contraction means that no cardiac output is possible, and vital organs, including the brain, are not perfused, resulting in rapid death.

The only treatment is DC cardioversion according to the Advanced Life Support Guidelines. Adjuncts to treatment include basic life support, intravenous adrenaline (epinephrine) and intravenous amiodarone.

157
Q

A 36-year-old woman presents to the emergency department with chest pain and a sensation of feeling her heart pounding. Apart from a history of schizophrenia, she has no medical problems. A heart monitor is attached and shows a broad complex tachycardia with a rate of 210 beats/min on a variable axis.

A.	 Atrioventricular node re-entry tachycardia
B.	 Chronic atrial fibrillation
C.	 Multifocal atrial tachycardia
D.	 Paroxysmal atrial fibrillation
E.	 Sick sinus syndrome
F.	 Sinus tachycardia
G.	 Torsades de pointes
H.	 Ventricular extrasystoles
I.	 Ventricular fibrillation	
J.	 Wolff–Parkinson–White syndrome
A

G – Torsades de pointes

Torsades de pointes, a specific form of ventricular tachycardia, is a broad complex ventricular arrhythmia caused by prolonged ventricular repolarization, which is seen on ECG as an extended QT interval. A prolonged QT interval may be congenital or acquired. Long-QT syndromes, such as Romano–Ward syndrome, are congenital diseases caused by ion channel abnormalities, and are commonly associated with torsades de pointes.

Acquired disease is usually secondary to electrolyte imbalance or drug side-effects. Patients with hypomagnesaemia and hypokalaemia are at particular risk of torsades de pointes. Drugs that are known to trigger torsades de pointes include class Ia antiarrhythmic agents (disopyramide and procainamide), tricyclic antidepressants (amitriptyline and imipramine), phenothiazine antipsychotics (chlorpromazine and prochlorperazine) and nonsedating antihistamines (loratidine).

Torsades de pointes is seen on ECG as a regular broad complex tachycardia with a varying axis whereby the QRS complexes repeatedly switch from an upright to a horizontal position. The patient often experiences palpitations, dizziness, chest pain and syncope. The rhythm usually aborts spontaneously, but has the potential to degenerate into ventricular fibrillation. Prolonged episodes of torsades de pointes that are associated with haemodynamic instability require urgent DC cardioversion.

158
Q

A 54-year-old woman with metastatic breast cancer presents to the emergency department feeling generally unwell and confused. Her daughter mentions that she has a poor appetite and has been vomiting.

A.	 Addisonian crisis
B.	 Anaphylaxis
C.	 Chemotherapy complication
D.	 Hypercalcaemia
E.	 Hyperkalaemia
F.	 Hypocalcaemia
G.	 Intracerebral bleed
H.	 Meningitis
I.	 Raised intracranial pressure
J.	 Spinal cord compression
K.	 Superior vena caval obstruction
L.	 Syndrome of inappropriate ADH secretion
M.	Tumour lysis syndrome
A

D – Hypercalcaemia

Hypercalcaemia of malignancy is usually due to the ectopic production of parathyroid hormone-related protein, which mimics the action of parathyroid hormone and mobilizes calcium into the bloodstream. Hypercalcaemia may also be due to lytic bone metastases or the activation of osteoclasts, as seen in breast cancer and myeloma, respectively. Acute hypercalcaemia usually presents with confusion, polyuria, polydipsia, constipation and mood changes, and is ultimately diagnosed by measuring serum calcium.

Treatment should begin with intravenous fluid hydration, aiming for 3–6 L of 0.9% saline over 24 hours. In severe cases, the patient should receive an intravenous bisphosphonate (e.g. pamidronate), which reduces bone resorption. Calcitonin may be given in hypercalcaemia refractory to hydration and bisphosphonate therapy.

159
Q

A 67-year-old man with a known small cell lung carcinoma is brought to the emergency department following a seizure. On questioning, he does not recall any details surrounding the event. Blood tests show a sodium of 112 mmol/L, a potassium of 4.6 mmol/L, a urea of 5mmol/L and a creatinine of 110µmol/L. His urine is very concentrated.

A.	 Addisonian crisis
B.	 Anaphylaxis
C.	 Chemotherapy complication
D.	 Hypercalcaemia
E.	 Hyperkalaemia
F.	 Hypocalcaemia
G.	 Intracerebral bleed
H.	 Meningitis
I.	 Raised intracranial pressure
J.	 Spinal cord compression
K.	 Superior vena caval obstruction
L.	 Syndrome of inappropriate ADH secretion
M.	Tumour lysis syndrome
A

L – Syndrome of inappropriate ADH secretion

SIADH in malignancy is usually due to ectopic ADH secretion from tumour cells. The most common tumour causing this condition is small cell lung carcinoma. The water retention caused by the excessive production of ADH results in a dilutional hyponatraemia, which, when severe, can cause seizures and permanent neurological damage. In addition to a low serum sodium concentration, the patient will also have raised urine osmolality and high urine sodium concentration.

Treatment requires fluid restriction, usually of 1 L/day. If intravenous fluids are required, only 0.9% saline should be used, to prevent a rapid increase in serum sodium. If the sodium is corrected too quickly, the patient may develop central pontine myelinolysis, which can cause severe and permanent neurological complications that can be fatal. Therefore, correction must occur over days, with regular assessment of serum sodium.

160
Q

A 32-year-old woman presents to her GP with a 1-month history of worsening diarrhoea and heat intolerance. On examination, she has a mild tremor and ‘bulging’ eyes. She would like treatment for her condition, but warns you that she is pregnant.

A.	 Iodine-131
B.	 Carbimazole
C.	 Desmopressin
D.	 Hydrocortisone and fludrocortisone
E.	 Octreotide
F.	 Spironolactone
G.	 Surgical resection and radiotherapy
H.	 Thyroxine alone
I.	 Total thyroidectomy and thyroxine
A

B – Carbimazole

This young pregnant woman presents with thyrotoxicosis and ophthalmoplegia, features typical of Graves’ disease. The first-line treatment in the under-40s is carbimazole, a drug that inhibits the production of thyroid hormones. Iodine- 131 should be avoided in women who are pregnant or who are around children, due to the risk of radioactivity-induced teratogenicity.

161
Q

An 18-year-old woman attends her GP with a lump in her neck that has been present for a few weeks. On examination, the lump is smooth and non-tender, and moves up with swallowing but not on tongue protrusion. Cervical lymphadenopathy is also noted.

A.	 Iodine-131
B.	 Carbimazole
C.	 Desmopressin
D.	 Hydrocortisone and fludrocortisone
E.	 Octreotide
F.	 Spironolactone
G.	 Surgical resection and radiotherapy
H.	 Thyroxine alone
I.	 Total thyroidectomy and thyroxine
A

I – Total thyroidectomy and thyroxine

This woman has an enlarging asymptomatic thyroid lump with lymphadenopathy. It is most likely that she has a papillary thyroid cancer. This is managed by total thyroidectomy followed by daily thyroxine to help prevent recurrence of the tumour.

162
Q

A 14-year-old girl attends the paediatric outpatient clinic following meningococcal septicaemia a few weeks previously. She complains of feeling tired and weak and having a poor appetite. She feels dizzy when standing up from a sitting position. Her blood tests show sodium 127 mmol/L and potassium 5.4 mmol/L.

A.	 Iodine-131
B.	 Carbimazole
C.	 Desmopressin
D.	 Hydrocortisone and fludrocortisone
E.	 Octreotide
F.	 Spironolactone
G.	 Surgical resection and radiotherapy
H.	 Thyroxine alone
I.	 Total thyroidectomy and thyroxine
A

D – Hydrocortisone and fludrocortisone

The features that this young girl has are typical of adrenal insufficiency. This, along with the recent meningococcal septicaemia, points to an underlying diagnosis of Waterhouse–Friderichsen syndrome, i.e. bilateral haemorrhage into the adrenal glands caused by meningococcal septicaemia. Management of adrenal insufficiency is by replacement of glucocorticoids (by hydrocortisone) and mineralocorticoids (by fludrocortisone).

163
Q

A 37-year-old man presents to the GP complaining of worsening headaches and vomiting. He also complains of neck stiffness and some blurring of vision. He is known to have HIV infection.

A.	 Candidiasis
B.	 Cryptococcus infection
C.	 Cryptosporidiosis
D.	 Cytomegalovirus infection
E.	 HIV dementia
F.	 HIV wasting syndrome
G.	 Kaposi’s sarcoma
H.	 Lichen planus
I.	 Pneumocystis infection
J.	 Seroconversion
K.	 Stevens–Johnson syndrome
L.	 Toxoplasmosis
A

B – Cryptococcus infection

This man with HIV presents with features typical of meningitis. In cases of subacute meningitis in immunocompromised patients, one must be wary of Cryptococcus infection and tuberculosis. Cryptococcus neoformans is an encapsulated fungus found in soil, and infection is by inhalation of contaminated material. Cryptococcus is diagnosed by detection of capsular material (e.g. in cerebrospinal fluid or sputum) using India ink stain. Treatment is with intravenous amphotericin B. Cryptococcus infection can also result in pulmonary disease.

164
Q

A 42-year-old woman presents with a 3-week history of difficult and painful swallowing, and she is losing weight as a result of not eating. She otherwise feels well. On examination, the mucous membranes of her mouth are sore and red. She is known to have HIV infection.

A.	 Candidiasis
B.	 Cryptococcus infection
C.	 Cryptosporidiosis
D.	 Cytomegalovirus infection
E.	 HIV dementia
F.	 HIV wasting syndrome
G.	 Kaposi’s sarcoma
H.	 Lichen planus
I.	 Pneumocystis infection
J.	 Seroconversion
K.	 Stevens–Johnson syndrome
L.	 Toxoplasmosis
A

A – Candidiasis

This woman with HIV infection, sore mouth, dysphagia and odynophagia has a Candida infection affecting her oesophagus. Diagnosis can be confirmed by endoscopy demonstrating white plaques typical of candidiasis. The presence of oesophageal candidiasis in anyone is highly suggestive of immunodeficiency. Treatment is with antifungals, such as fluconazole and amphotericin.

165
Q

This scoring system should be used as a quick guide to an elderly patient’s cognitive state.

A.	 Abbreviated Mental Test Score
B.	 Breslow Score
C.	 Curb Score
D.	 Geriatric Depression Score
E.	 Glasgow Coma Scale
F.	 Mini-Mental Test Score
G.	 Ranson’s Criteria
H.	 Rockall Score
I.	 Waterlow Score
A

A – Abbreviated Mental Test Score

The Abbreviated Mental Test Score (AMTS) is a quick and easily remembered way of initially and continuously assessing a patient’s cognitive function. It involves asking the patient 10 questions that assess several different areas of cognition. It is especially useful in elderly patients suffering from delirium and dementia. A score of less than 7/10 indicates cognitive impairment.

166
Q

This scoring system can be used to assess the risk of an adverse outcome following an upper gastrointestinal bleed.

A.	 Abbreviated Mental Test Score
B.	 Breslow Score
C.	 Curb Score
D.	 Geriatric Depression Score
E.	 Glasgow Coma Scale
F.	 Mini-Mental Test Score
G.	 Ranson’s Criteria
H.	 Rockall Score
I.	 Waterlow Score
A

H – Rockall Score

This scoring system can be used to predict adverse outcome following upper gastrointestinal bleeds by combining a number of independent risk factors (see below). A score >8 indicates a 40% risk of mortality.

167
Q

This scoring system should be used to assess a patient’s risk of developing a pressure score.

A.	 Abbreviated Mental Test Score
B.	 Breslow Score
C.	 Curb Score
D.	 Geriatric Depression Score
E.	 Glasgow Coma Scale
F.	 Mini-Mental Test Score
G.	 Ranson’s Criteria
H.	 Rockall Score
I.	 Waterlow Score
A

I – Waterlow Score

The Waterlow Score is an important tool for assessing the risk of pressure sore development. Pressure sores are likely to develop in bedbound patients who have poor mobility, poor nutrition, incontinence and multiple comorbidities. They can cause significant pain and become infected, leading to sepsis and even death. The Waterlow Score is generally used by nurses during the admission of patients, and is especially important in stroke victims, patients with quadriplegia and comatose patients.

168
Q

A 76-year-old woman is hanging up her laundry on the washing line outside. While reaching up to peg on the last towel, she suddenly becomes dizzy and collapses on the ground. She does not lose consciousness, but takes a few moments to recover.

A.	 Carotid sinus sensitivity
B.	 Epilepsy
C.	 Hyperventilation
D.	 Hypoglycaemia
E.	 Ménière’s disease
F.	 Micturition syncope
G.	 Postural hypotension
H.	 Stokes–Adams attack
I.	 Vasovagal attack
J.	 Vertebrobasilar insufficiency
A

J – Vertebrobasilar insufficiency

In vertebrobasilar insufficiency, there is a temporarily reduced flow in the posterior circulation of the brain. It typically affects those aged 60–70 and reflects underlying atherosclerosis. Transient ischaemia in vertebrobasilar insufficiency is precipitated by neck extension, and results in dizziness.

169
Q

A 64-year-old man presents to the GP with a 2-week history o fcoughing up blood-stained mucus. He has also lost a stone (6kg) in weight in the last month. He mentions that he cannot fully lift his left eyelid. On examination, the left eyelid is drooping and the associated pupil is smaller than the other. There is no other neurological deficit.

A.	 Argyll Robertson pupil
B.	 Brain-stem death
C.	 Holmes–Adie pupil
D.	 Horner’s syndrome
E.	 Marcus Gunn pupil
F.	 Oculomotor nerve palsy
G.	 Opiate intoxication
A

D – Horner’s syndrome

Horner’s syndrome is caused by an interruption of the sympathetic innervation to the eye. It is characterized by four features: a partial ptosis with ipsilateral anhidrosis (loss of sweating), enophthalmos (the impression that the eye is sunk in) and miosis (constricted pupil). Congenital Horner’s syndrome may be associated with heterochromia of the iris – a difference in colour between the two eyes. Causes of non-congenital Horner’s syndrome include Pancoast’s tumour (a tumour at the lung apex – as in this case), brachial plexus palsies, cervical rib, cluster headache and trauma to the base of the neck.

170
Q

A 42-year-old woman presents with a short history of pain and blurring in her left eye. She is known to have multiple sclerosis. On examination, the right pupil appears to react appropriately to light, but shining a torch in the left pupil causes it to dilate.

A.	 Argyll Robertson pupil
B.	 Brain-stem death
C.	 Holmes–Adie pupil
D.	 Horner’s syndrome
E.	 Marcus Gunn pupil
F.	 Oculomotor nerve palsy
G.	 Opiate intoxication
A

E – Marcus Gunn pupil

A Marcus Gunn pupil is one that reacts poorly to light in comparison with the contralateral pupil. This is elicited on examination using the ‘swinging flashlight test’: when a pen torch is held towards the good eye, both pupils constrict (as they should), but when it is then held against the affected eye, both pupils appear to dilate. This is because the affected pupil does not perceive the light as well, and so constricts less compared with when the light is shone into the normal pupil – hence it appears to dilate. The Marcus Gunn pupil is characteristic of optic nerve lesions (e.g. optic neuritis or compression of the nerve by an aneurysm or tumour).

171
Q

A 62-year-old man is admitted to hospital with difficulty walking and poor coordination. On examination, both of his pupils are small and unreactive to light. The accommodation reflex is present.

A.	 Argyll Robertson pupil
B.	 Brain-stem death
C.	 Holmes–Adie pupil
D.	 Horner’s syndrome
E.	 Marcus Gunn pupil
F.	 Oculomotor nerve palsy
G.	 Opiate intoxication
A

A – Argyll Robertson pupil

Argyll Robertson pupils are bilateral small pupils that accommodate but do not react to light. They are a feature of neurosyphilis, but can also occur with diabetes mellitus.

172
Q

This investigation should be performed in a patient with suspected Crohn’s disease affecting the small bowel.

A.	 Abdominal X-ray
B.	 Anti-endomyseal antibodies
C.	 Barium swallow and follow-through
D.	 Colonoscopy
E.	 CT scan
F.	 Full blood count
G.	 Gastrografin swallow
H.	 Helicobacter pylori CLO test
I.	 Helicobacter pylori stool antigen test
J.	 Non-urgent endoscopy and biopsy
K.	 Urgent endoscopy
A

E - CT scan

First investigation should be CT or MRI, before colonoscopy. Looking for skip lesions, bowel wall thickening.

Colonoscopy is the definitive test to diagnose Crohn’s - looking for aphthous ulcers, hyperaemia, oedema, cobblestoning, skip lesions.

173
Q

A 27-year-old woman with known primary pulmonary hypertension is admitted with a long history of abdominal pain. On examination, you find a murmur and peripheral oedema. Her liver is enlarged and tender to palpation.

A.	 Aortic regurgitation
B.	 Aortic stenosis
C.	 Atrial septal defect
D.	 Bicuspid aortic valve
E.	 Coarctation of the aorta
F.	 Marfan’s syndrome
G.	 Mitral regurgitation
H.	 Mitral stenosis
I.	 Prosthetic heart valve
J.	 Pulmonary stenosis
K.	 Tetralogy of Fallot
L.	 Tricuspid regurgitation
M.	Tricuspid stenosis
A

L – Tricuspid regurgitation

Pulmonary hypertension can be primary or secondary to cardiac or systemic disease. Increased pressures in the pulmonary vasculature result in a backpressure that feeds back to the right ventricle. This causes the right ventricle to hypertrophy and dilate, which prevents the tricuspid valve cusps from opposing. Thus, blood is able to regurgitate from the right ventricle into the right atrium during systole. On auscultation, there is a pansystolic murmur that is best heard at the left sternal border in the fifth intercostal space. In severe disease, patients have a raised jugular venous pressure with a prominent v-wave, peripheral oedema and tender, pulsatile hepatomegaly.

174
Q

A 34-year-old woman with systemic lupus erythematosus presents to her GP complaining of being generally puffy. On examination, there is periorbital oedema and ankle oedema. Her blood pressure is 170/98 mmHg, and a urine dipstick analysis shows 3+ of protein and 2+ of blood.

A.	 AA amyloidosis
B.	 AL amyloidosis
C.	 Diabetic microalbuminuria
D.	 Diabetic proteinuria
E.	 Lupus nephritis
F.	 Multiple myeloma
G.	 Polyarteritis nodosa
H.	 Rhabdomyolysis
I.	 Rhabdomyosarcoma
J.	 Streptococcal glomerulonephritis
K.	 Wegener’s granulomatosis
A

E – Lupus nephritis

Systemic lupus erythematosus (SLE) is a multisystem autoimmune disease characterized by the production of antinuclear antibodies. Up to 50% of patients with SLE have renal involvement. Proteinuria, hypertension, haematuria and peripheral oedema may all be seen. The renal damage is thought to arise from the deposition of immune complexes within the glomeruli that trigger an immunological response, damaging the basement membrane.

A renal biopsy is usually required for the diagnosis and staging of the disease. The pattern of glomerulonephritis is mesangial, focal proliferative, diffuse proliferative or membranous. Lupus nephritis is treated by systemic immunosuppression using agents such as prednisolone, ciclosporin and mycophenolate. Some patients require renal replacement therapy and eventual renal transplantation.

175
Q

A 45-year-old man with Crohn’s disease is shown to have deteriorating renal function and proteinuria. A renal biopsy is taken, and the sample stains positively with Congo red.

A.	 AA amyloidosis
B.	 AL amyloidosis
C.	 Diabetic microalbuminuria
D.	 Diabetic proteinuria
E.	 Lupus nephritis
F.	 Multiple myeloma
G.	 Polyarteritis nodosa
H.	 Rhabdomyolysis
I.	 Rhabdomyosarcoma
J.	 Streptococcal glomerulonephritis
K.	 Wegener’s granulomatosis
A

A – AA amyloidosis

In amyloidosis, there is deposition of extracellular fibrillar proteins in tissues and organs. The two main types of amyloidosis are AL and AA. AL amyloidosis, also incorrectly referred to as primary amyloidosis, is due to the clonal proliferation of plasma cells and particularly affects the cardiovascular system. AA amyloidosis, also known as reactive systemic amyloidosis, is due to the production and deposition of amyloid in chronic inflammatory conditions such as Crohn’s disease, rheumatoid arthritis and tuberculosis.

In AA amyloidosis, amyloid A protein is deposited in the kidneys, liver and spleen, causing renal failure and hepatosplenomegaly. Cardiovascular involvement is rare. The renal complications of amyloidosis include proteinuria, nephrotic syndrome and endstage renal failure. Amyloidosis is traditionally diagnosed based on a biopsy of the affected tissue. The fibrillar protein shows green birefringence when stained with Congo red and examined beneath a polarised light. AL amyloidosis can be managed with chemotherapy or stem cell transplantation, but is usually fatal within 2 years. AA amyloidosis may respond to treatment of the underlying cause.

176
Q

A 32-year-old male intravenous drug user presents with a 4-week history of fever, lethargy and weight loss. On examination, you find that he has swollen ankles and a raised jugular venous pressure. On auscultation, you notice a pansystolic murmur that is best heard over the left sternal border.

A.	 Candida albicans
B.	 Coxsackie B virus
C.	 Enterococcus faecalis
D.	 Mycobacterium tuberculosis
E.	 Neisseria meningitidis
F.	 Staphylococcus aureus
G.	 Streptococcus bovis
H.	 Staphylococcus epidermidis
I.	 Streptococcus pyogenes
J.	 Viridans streptococcus
K.	 Trypanosoma cruzi
A

F – Staphylococcus aureus

Intravenous drug users are at risk of developing bacterial endocarditis secondary to S. aureus bacteraemia introduced during non-sterile venepuncture. The valves that are affected tend to be (previously) normal and are often right-sided. In this case, the tricuspid valve is affected, causing significant regurgitation and right heart failure. The antibiotics of choice in acute S. aureus endocarditis are intravenous flucloxacillin and gentamicin. If resistant bacteria, such as meticillinresistant S. aureus (MRSA), are suspected or proven, intravenous vancomycin plus gentamicin should be initiated.

177
Q

A 45-year-old woman presents with a 3-week history of night sweats, weight loss and palpitations approximately 1 month after having a dental abscess removed. On auscultation, you notice a pansystolic murmur that is best heard at the apex. She had rheumatic fever as a child.

A.	 Candida albicans
B.	 Coxsackie B virus
C.	 Enterococcus faecalis
D.	 Mycobacterium tuberculosis
E.	 Neisseria meningitidis
F.	 Staphylococcus aureus
G.	 Streptococcus bovis
H.	 Staphylococcus epidermidis
I.	 Streptococcus pyogenes
J.	 Viridans streptococcus
K.	 Trypanosoma cruzi
A

J – Viridans streptococcus

Viridans streptococci are a large group of bacterial organisms that are usually commensals in the oral cavity. They are the most common cause of bacterial endocarditis, being responsible for approximately 60% of cases. Individuals with pre-existing valvular disease are at risk of viridans streptococcal endocarditis following invasive dental procedures such as tooth extraction. Antibiotic treatment is usually with intravenous benzylpenicillin, with the addition of gentamicin if resistant bacteria are present. Patients with known valvular heart disease should always receive prophylactic antibiotics prior to any invasive procedure.

178
Q

A 64-year-old man presents to the emergency department with fever. The observations include temperature 40.2°C, blood pressure 100/48 mmHg and heart rate 114 beats/min. Examination reveals a thoracotomy scar from a metallic aortic valve replacement 1 month previously. Transoesophageal echocardiography shows a number of vegetations that are visualized at the insertion point of the metallic ring.

A.	 Candida albicans
B.	 Coxsackie B virus
C.	 Enterococcus faecalis
D.	 Mycobacterium tuberculosis
E.	 Neisseria meningitidis
F.	 Staphylococcus aureus
G.	 Streptococcus bovis
H.	 Staphylococcus epidermidis
I.	 Streptococcus pyogenes
J.	 Viridans streptococcus
K.	 Trypanosoma cruzi
A

H – Staphylococcus epidermidis

This organism is a skin commensal. It commonly infects prosthetic valves and other indwelling foreign bodies such as peritoneal dialysis lines and venous lines. This is due to the bacterium’s ability to adhere to, and grow on, prosthetic surfaces. S. epidermidis should therefore be suspected in all patients with prosthetic valves who present with the signs and symptoms of bacterial endocarditis. Antibiotic therapy is with benzylpenicillin and gentamicin. If infection occurs within the initial weeks following valve replacement, an urgent surgical opinion should be sought.

179
Q

A 26-year-old man has a 2-day history of malaise, fever and shortness of breath associated with pain over the right side of his chest. He has developed a cough and is expectorating rusty sputum. A chest X-ray confirms consolidation of the right lower lobe.

A.	 Chlamydophila pneumoniae
B.	 Chlamydophila psittaci
C.	 Haemophilus influenzae
D.	 Klebsiella pneumoniae
E.	 Legionella pneumophila
F.	 Mycobacterium tuberculosis
G.	 Mycoplasma pneumoniae
H.	 Pneumocystis jiroveci
I.	 Pseudomonas aeruginosa
J.	 Staphylococcus aureus
K.	 Streptococcus pneumoniae
A

K – Streptococcus pneumoniae

S. pneumoniae is the most common cause of community-acquired pneumonia. It often affects young and middle-aged people, and presents as a rapid-onset illness with high fevers, rigors and pleuritic chest pain. The production of rustycoloured sputum (blood stained) is characteristic. Chest X-ray often shows lobar consolidation.

Chlamydophila pneumoniae (previously known as Chlamydia pneumoniae) causes a mild pneumonia in younger people, and may be associated with sinusitis, pharyngitis and laryngitis. Chest X-ray shows small segmental infiltrates.

Mycoplasma pneumoniae affects adolescents and young adults, and tends to cause epidemics in 3-year cycles. Patients often present with few chest signs, but can develop erythema nodosum, pericarditis and haemolytic anaemia. Chest X-ray shows patchy or lobar consolidation and hilar lymphadenopathy.

180
Q

A 67-year-old woman presents with a 5-day history of malaise and a cough productive of yellow sputum. Examination is unremarkable. A chest X-ray shows two cavities with air–fluid levels.

A.	 Chlamydophila pneumoniae
B.	 Chlamydophila psittaci
C.	 Haemophilus influenzae
D.	 Klebsiella pneumoniae
E.	 Legionella pneumophila
F.	 Mycobacterium tuberculosis
G.	 Mycoplasma pneumoniae
H.	 Pneumocystis jiroveci
I.	 Pseudomonas aeruginosa
J.	 Staphylococcus aureus
K.	 Streptococcus pneumoniae
A

J – Staphylococcus aureus

S. aureus often complicates an underlying viral pneumonia. Infection can arise from other foci, such as osteomyelitis. The characteristic feature is the X-ray finding of abscesses, seen as cavities with air–fluid levels.

181
Q

A 57-year-old man presents to the emergency department with severe upper abdominal pain and haematemesis. An emergency endoscopy is organized, and showed multiple peptic ulcers in his stomach, duodenum and jejunum.

A.	 Carcinoid tumour
B.	 Colorectal carcinoma
C.	 Gallbladder carcinoma
D.	 Gastric carcinoma
E.	 Insulinoma
F.	 Oesophageal adenocarcinoma
G.	 Pancreatic carcinoma
H.	 Small-bowel carcinoma
I.	 Zollinger–Ellison syndrome
A

I – Zollinger–Ellison syndrome

Zollinger–Ellison syndrome describes peptic ulceration secondary to a gastrin secreting adenoma (gastrinoma). The adenoma is usually located in the pancreas, but can be found in the stomach or small bowel. Approximately 60% of gastrinomas are malignant and have the potential to metastasize to local lymph nodes and the liver. The unregulated secretion of gastrin simulates the parietal cells in the gastric antrum to produce excessive amounts of hydrochloric acid. This disrupts the gastric and duodenal mucosa, causing peptic ulceration. The acid can also denature pancreatic enzymes, such as lipase, resulting in malabsorption and chronic diarrhoea.

Diagnosis is often based upon endoscopic identification of multiple peptic ulcers in association with a raised fasting serum gastrin level. Treatment involves high-dose protein pump inhibitors and surgical resection of the adenoma. Patients with Zollinger–Ellison syndrome associated with multiple endocrine neoplasia (MEN) type 1 often have multiple adenomas that are not suitable for resection. In this situation, the patient can usually be managed using a somatostatin analogue (e.g. octreotide), which decreases gastrin secretion. Metastatic disease necessitates systemic chemotherapy.

182
Q

A 46-year-old woman presents to her GP complaining of episodes of flushing and shortness of breath. On further questioning, she admits to occasional episodes of explosive diarrhoea.

A.	 Carcinoid tumour
B.	 Colorectal carcinoma
C.	 Gallbladder carcinoma
D.	 Gastric carcinoma
E.	 Insulinoma
F.	 Oesophageal adenocarcinoma
G.	 Pancreatic carcinoma
H.	 Small-bowel carcinoma
I.	 Zollinger–Ellison syndrome
A

A – Carcinoid tumour

Carcinoid tumours are serotonin-secreting neoplasms derived from the enterochromaffin cells of the gastrointestinal tract. They are usually found in the terminal ileum, appendix or rectum, but can occur in extraintestinal sites such as the bronchi, testes and ovary. If the tumour is limited to the gastrointestinal tract, the serotonin that is secreted enters the portal vein and is metabolized by the liver. If there are liver metastases, or if the primary tumour is extraintestinal, the secreted serotonin escapes hepatic metabolism and enters the systemic circulation, causing flushing, tachycardia, wheeze, diarrhoea, heart valve fibrosis and right heart failure. This is known as carcinoid syndrome.

Diagnosis is based on the history and elevated levels of 5-hydroxyindole acetic acid (5-HIAA), a serotonin metabolite, in the urine. Management is by resection or, in widespread disease, symptomatic treatment with octreotide. Carcinoid tumours are slow growing so, even if disseminated disease is present, patients can live for many years.

183
Q

A 38-year-old woman presents with a visual problem. She complains that she has a dark patch of visual loss within her field of vision in the right eye. Despite this, the rest of her vision is fine.

A.	 Amaurosis fugax
B.	 Bitemporal hemianopia
C.	 Cataract
D.	 Central scotoma
E.	 Cortical blindness
F.	 Fortification spectra
G.	 Homonymous hemianopia
H.	 Hypermetropia
I.	 Myopia
J.	 Presbycusis
K.	 Tunnel vision
A

D – Central scotoma

A scotoma describes an area of visual field loss within a field of normal vision. The blind spot is an example of a scotoma that is present in every human. Pathological scotomata can be caused by problems of the retina or optic nerve, e.g. multiple sclerosis, optic nerve glioma, glaucoma and vascular lesions.

A scintillating scotoma describes an area of flashing lights within the field of vision, often seen in migraine. This can take the form of zigzag lines, in which case it is known as a fortification spectrum.

184
Q

A 48-year-old woman presents to the GP with weight gain and increased hair on her abdomen and face. On examination, she has thin skin and multiple bruises.

A.	 24-hour catecholamine collection
B.	 Aldosterone levels
C.	 Clotting screen
D.	 Glucose tolerance test
E.	 High-dose dexamethasone suppression test
F.	 Low-dose dexamethasone suppression test
G.	 Random cortisol level
H.	 Random glucose
I.	 Short Synacthen test
J.	 Thyroid function tests
K.	 Water deprivation test
A

F – Low-dose dexamethasone suppression test

This patient could have glucocorticoid excess (Cushing’s syndrome), which can be diagnosed using the dexamethasone suppression test. This involves measuring a baseline serum cortisol level at 09:00 before giving 0.5 mg of dexamethasone (a synthetic glucocorticoid) four times daily for 48 hours. At 48 hours, the serum cortisol level is repeated. In healthy individuals, the dexamethasone suppresses cortisol secretion by negatively feeding back to the anterior pituitary gland and inhibiting ACTH release. In Cushing’s syndrome, the cortisol levels remain elevated and the test is positive.

If the low-dose dexamethasone suppression test is positive, a high-dose dexamethasone suppression test can be performed to differentiate between Cushing’s syndrome caused by pituitary disease (Cushing’s disease) and Cushing’s syndrome caused by the ectopic secretion of ACTH from a non-pituitary tumour (e.g. small cell lung carcinoma). This test involves measuring a baseline serum cortisol level at 09:00 before giving 2 mg of dexamethasone four times daily for 48 hours and then measuring a repeat serum cortisol level. If the 48-hour cortisol level is significantly lower than the pre-test sample, it is likely that pituitary disease is responsible, since it is partially sensitive to negative feedback. If the cortisol is not suppressed at 48 hours, it is likely that ectopic ACTH secretion is occurring, since it is completely unresponsive to negative feedback.

185
Q

A 56-year-old man with hypertension, hyperglycaemia and facial changes is referred to the general medicine clinic for investigation.

A.	 24-hour catecholamine collection
B.	 Aldosterone levels
C.	 Clotting screen
D.	 Glucose tolerance test
E.	 High-dose dexamethasone suppression test
F.	 Low-dose dexamethasone suppression test
G.	 Random cortisol level
H.	 Random glucose
I.	 Short Synacthen test
J.	 Thyroid function tests
K.	 Water deprivation test
A

D – Glucose tolerance test

It is possible that this patient has acromegaly. Growth hormone (GH) opposes the action of insulin, resulting in high plasma glucose levels. Features of acromegaly include swelling of the hands and feet, lower jaw protrusion, macroglossia (large tongue), bitemporal hemianopia, carpal tunnel syndrome, hypertension, hyperglycaemia and cardiomegaly. In health, high plasma glucose levels feed back to suppress GH secretion. In acromegaly, there is autonomous secretion of GH, usually from a pituitary adenoma, that escapes the control of negative feedback.

The glucose tolerance test can be used to help diagnose acromegaly. Failure to suppress GH levels to below 1 µg/L 2 hours after a 75 g oral glucose bolus is considered a positive result. Other investigations used in the diagnosis of acromegaly include insulin-like growth factor 1 levels and cranial MRI.

186
Q

A 22-year-old woman attends the dermatology outpatient clinic with multiple skin lesions on her body. On examination, she has eight light-brown macules on her arms and trunk, and four firm rubbery lesions.

A.	 Acoustic neuromas
B.	 Arnold–Chiari malformation
C.	 Ataxia telangiectasia
D.	 Brown-Séquard syndrome
E.	 Friedreich’s ataxia
F.	 Neurofibromatosis type 1
G.	 Neurofibromatosis type 2
H.	 Shy–Drager syndrome
I.	 Subacute sclerosing panencephalitis
J.	 Syringomyelia
K.	 Von Hippel–Lindau disease
A

F – Neurofibromatosis type 1

Neurofibromatosis is an autosomal dominant disorder characterized by the development of multiple neurofibromas from the neurilemmal sheaths of central and peripheral nerves. There are two types: in type 1 (NF1, also known as von Recklinghausen’s disease), the mutation is on chromosome 17; in type 2 (NF2), the mutation is on chromosome 22. NF1 is the ‘peripheral form’, accounting for 70% of cases. Features include multiple cutaneous neurofibromas, café-au-lait patches (at least six), axillary freckling and fibromas on the iris (Lisch nodules). Affected patients are also at risk of developing scoliosis and neural tumours (e.g. phaeochromocytoma, acoustic neuromas).

NF2 is the ‘central form’ and has few or no cutaneous lesion. Instead, patients develop bilateral neural tumours (acoustic neuromas, optic nerve gliomas and meningiomas).

Acoustic neuromas are benign tumours of the Schwann cells of cranial nerve VIII. They can occur in conjunction with neurofibromatosis or as isolated tumours. Because of their close relationship to cranial nerves VII and VIII, acoustic neuromas can result in hearing loss, motor symptoms of the face and vertigo.

187
Q

A 42-year-old woman has a 2-month history of shoulder pain associated with sensory loss in her arms. On examination, you note that she is insensate to pain and temperature in both hands, although her joint position sense is intact. There is no other motor or sensory loss.

A.	 Acoustic neuromas
B.	 Arnold–Chiari malformation
C.	 Ataxia telangiectasia
D.	 Brown-Séquard syndrome
E.	 Friedreich’s ataxia
F.	 Neurofibromatosis type 1
G.	 Neurofibromatosis type 2
H.	 Shy–Drager syndrome
I.	 Subacute sclerosing panencephalitis
J.	 Syringomyelia
K.	 Von Hippel–Lindau disease
A

J – Syringomyelia

Syringomyelia describes the presence of a fluid-filled cavity within the spinal cord, usually in the cervical segments. The expanding cavity disrupts spinothalamic neurons, resulting in sensory loss (pain and temperature) in the affected distribution. Syringomyelia may be associated with congenital herniation of the cerebellar tonsils (Arnold–Chiari malformation). The best investigation is MRI. Management is by surgical decompression of the syrinx.

188
Q

A 34-year-old man is referred to the general medical clinic by his GP for uncontrollable hypertension. He is presently on ramipril and bendroflumethazide. On further questioning, he admits to having paroxysms of anxiety, sweating and palpitations that are brought on by stress, which he describes as ‘panic attacks’. He has a past medical history of kidney cancer and ‘tumours in his eyes’.

A.	 Acoustic neuromas
B.	 Arnold–Chiari malformation
C.	 Ataxia telangiectasia
D.	 Brown-Séquard syndrome
E.	 Friedreich’s ataxia
F.	 Neurofibromatosis type 1
G.	 Neurofibromatosis type 2
H.	 Shy–Drager syndrome
I.	 Subacute sclerosing panencephalitis
J.	 Syringomyelia
K.	 Von Hippel–Lindau disease
A

K – Von Hippel–Lindau disease

This man presents with a phaeochromocytoma and has a history of other tumours. Von Hippel–Lindau disease is caused by an autosomal dominant defect on chromosome 3. The syndrome is characterized by retinal and intracranial haemangiomas and haemangioblastomas, renal cysts, renal cell adenocarcinoma, pancreatic tumours and phaeochromocytoma.

189
Q

An 8-year-old boy is brought to the GP with a 6-month history of difficulty walking and increasing clumsiness. He was previously developing normally. On examination, he appears to have some thin, red, spidery lesions on his cheeks.

A.	 Acoustic neuromas
B.	 Arnold–Chiari malformation
C.	 Ataxia telangiectasia
D.	 Brown-Séquard syndrome
E.	 Friedreich’s ataxia
F.	 Neurofibromatosis type 1
G.	 Neurofibromatosis type 2
H.	 Shy–Drager syndrome
I.	 Subacute sclerosing panencephalitis
J.	 Syringomyelia
K.	 Von Hippel–Lindau disease
A

C – Ataxia telangiectasia

Ataxia telangiectasia is an example of a hereditary ataxia. It is autosomal recessive and presents in childhood with progressive ataxia and athetosis (slow writhing movements). It is associated with the presence of telangiectasias on the conjunctiva, cheeks and ears. Patients also have immunodeficiency and are significantly predisposed to lymphoma and leukaemia.

Friedreich’s ataxia is an autosomal recessive condition that presents in adolescence with ataxia and muscle weakness, hearing loss, dysarthria and optic atrophy. It is also associated with diabetes mellitus and cardiomyopathy. Examination may reveal areflexia, scoliosis and pes cavus (high-arched feet). Affected people are often wheelchair bound by 20 years.

190
Q

A 58-year-old woman with rheumatoid arthritis is admitted with a chest infection. Routine blood tests show haemoglobin 9.4 g/dL, white cell count 6.2 × 109/L and platelet count 128 × 109/L. Examination reveals a large abdominal mass in the left upper quadrant.

A.	 Amyloidosis
B.	 Anaemia of chronic disease
C.	 Atlantoaxial subluxation
D.	 Caplan’s syndrome
E.	 Episcleritis
F.	 Felty’s syndrome
G.	 Mononeuritis multiplex
H.	 Palmar erythema
I.	 Pyoderma gangrenosum
J.	 Rheumatoid nodules
K.	 Scleritis
L.	 Scleromalacia perforans
M.	 Sjögren’s syndrome
A

F – Felty’s syndrome

Anaemia in RA has four main underlying causes. There can be an anaemia of chronic disease, which tends to cause a normochromic normocytic anaemia (although 25% of cases are associated with a hypochromic microcytic picture). There may be an autoimmune haemolysis associated with the disease process. Some drug treatments of RA can result in anaemia, especially non-steroidal antiinflammatory drugs and gold. The final reason is Felty’s syndrome.

Felty’s syndrome is a triad of RA, splenomegaly and hypersplenism (which results in pancytopenia). Sequelae of pancytopenia include anaemia and recurrent infections. Felty’s syndrome is also associated with leg ulcers and lymphadenopathy. In some cases, splenectomy can improve symptoms.

191
Q

A 59-year-old woman is having difficulty walking and has had multiple falls recently. On gait examination, you notice that she is walking very slowly and is carefully watching and placing her feet. On examination, there is no weakness or sensory deficit.

A.	 Antalgic gait
B.	 Cerebellar ataxia
C.	 Extrapyramidal gait
D.	 Gait apraxia
E.	 High-stepping gait
F.	 Marche à petit pas
G.	 Pyramidal gait
H.	 Sensory ataxia
I.	 Spastic gait
J.	 Trendelenburg gait
K.	 Waddling gait
A

H – Sensory ataxia

Sensory ataxia occurs with a loss of joint position sense. Affected patients literally have to watch their feet, and they tend to place the foot on the ground with greater emphasis, especially in conditions of poor lighting. This can lead to stamping of the feet with walking (‘stomping gait’). Sensory ataxia occurs with disorders affecting the dorsal column of the spinal cord (which controls joint position sense and vibration). Causes include tabes dorsalis of neurosyphilis and vitamin B 12 deficiency (pernicious anaemia).

192
Q

A 62-year old woman is having extreme difficulty walking up stairs. She also finds it difficult to stand up, and says that her legs feel weak. She has a history of severe chronic obstructive pulmonary disease, for which she has taken frequent courses of prednisolone.

A.	 Antalgic gait
B.	 Cerebellar ataxia
C.	 Extrapyramidal gait
D.	 Gait apraxia
E.	 High-stepping gait
F.	 Marche à petit pas
G.	 Pyramidal gait
H.	 Sensory ataxia
I.	 Spastic gait
J.	 Trendelenburg gait
K.	 Waddling gait
A

K – Waddling gait

Difficulty standing from a sitting position and trouble walking up stairs is indicative of proximal muscle weakness (i.e. hip girdles). This is a known sideeffect of steroid use. If the muscles that stabilize the hip joint (the gluteus medius and minimus) are weak, the hips will not be fixed when the contralateral leg is lifted off the ground. This results in exaggerated trunk movements bilaterally, described as a waddling gait.

193
Q

A 12-year-old boy is brought to the emergency department with abdominal pains. When he walks into the room, you notice that he has a characteristic posture. His knees are slightly flexed, his hips are adducted and his ankles are plantarflexed bilaterally. This results in his legs crossing each other when he walks.

A.	 Antalgic gait
B.	 Cerebellar ataxia
C.	 Extrapyramidal gait
D.	 Gait apraxia
E.	 High-stepping gait
F.	 Marche à petit pas
G.	 Pyramidal gait
H.	 Sensory ataxia
I.	 Spastic gait
J.	 Trendelenburg gait
K.	 Waddling gait
A

I – Spastic gait

This child has cerebral palsy. The described deformity is characteristic of spasticity (hips adducted and internally rotated, knees flexed and ankles plantarflexed). In these circumstances, both lower limbs are moved slowly and dragged along the ground, with crossing of the legs due to adduction at the hip. Movement in the patient with bilateral spasticity is known as a scissoring gait.

194
Q

A 23-year-old man with known asthma is brought to the emergency department with shortness of breath. He has widespread wheeze, is too breathless to talk, and has a heart rate of 115 beats/min and a respiratory rate of 36/min.

A. Inhaled high-dose steroids
B. Inhaled low-dose steroids
C. Inhaled short-acting β2-agonist
D. Intravenous magnesium sulphate
E. Nebulized short-acting β2-agonist
F. No further management required
G. No treatment required
H. Oral leukotriene receptor antagonist
I. Oral steroids
J. Oxygen via facemask and inhaled short-acting β2-agonist
K. Oxygen via facemask, inhaled short-acting β2-agonist and steroids
L. Oxygen via facemask and nebulized short-acting β2-agonist
M. Oxygen via facemask, nebulized short-acting β2-agonist and steroids

A

M – Oxygen via facemask, nebulized short-acting β2-agonist and steroids

This is a severe exacerbation of asthma. Features of severe asthma are:
• the patient is too breathless to finish sentences
• respiration rate ≥25/min
• pulse ≥100 beats/min
• peak flow ≤50% predicted or best.

Acute exacerbations of asthma should be treated immediately with 60% oxygen (via a mask), back-to-back nebulizers and systemic steroids (either oral prednisolone or intravenous hydrocortisone). Patients should have their oxygen saturations measured continuously, and have regular PEFR readings (e.g. every 15–30 minutes) and serial arterial blood gases.

195
Q

A 74-year-old man attends the emergency department after experiencing sudden-onset dizziness earlier in the day. He also has problems articulating his words and swallowing food.

A.	 Lacunar stroke
B.	 Partial anterior circulation stroke
C.	 Posterior circulation stroke
D.	 Total anterior circulation stroke
E.	 Transient ischaemic attack
A

C – Posterior circulation stroke

The classification of acute stroke was first described by Bamford and the Oxfordshire Community Stroke Project in the 1990s as follows. A total anterior circulation stroke (TACS) results in a triad of hemiplegia, hemianopia and higher cortical dysfunction (e.g. aphasia and visuospatial problems). A partial anterior circulation stroke (PACS) displays two of the above three features. A posterior circulation stroke (POCS) affects the brain stem, resulting in vertigo, dysphagia, dysarthria and facial weakness.

196
Q

A 68-year-old man presents with weakness in his left arm and leg. He also appears confused, and his wife says that this is new. On examination, there is no evidence of visual field deficit.

A.	 Lacunar stroke
B.	 Partial anterior circulation stroke
C.	 Posterior circulation stroke
D.	 Total anterior circulation stroke
E.	 Transient ischaemic attack
A

B – Partial anterior circulation stroke

The classification of acute stroke was first described by Bamford and the Oxfordshire Community Stroke Project in the 1990s as follows. A total anterior circulation stroke (TACS) results in a triad of hemiplegia, hemianopia and higher cortical dysfunction (e.g. aphasia and visuospatial problems). A partial anterior circulation stroke (PACS) displays two of the above three features. A posterior circulation stroke (POCS) affects the brain stem, resulting in vertigo, dysphagia, dysarthria and facial weakness.

197
Q

A 79-year-old woman suffers sudden-onset weakness in her left arm and leg. She denies any other problems. Examination reveals no cognitive or sensory impairment, or any visual field defect.

A.	 Lacunar stroke
B.	 Partial anterior circulation stroke
C.	 Posterior circulation stroke
D.	 Total anterior circulation stroke
E.	 Transient ischaemic attack
A

A – Lacunar stroke

A lacunar stroke (LACS) is a pure motor stroke (hemiparesis), a pure sensory stroke or a combination of the two. The deficit in an LACS must involve two of the face, leg and hand. Furthermore, there must be no visual field defect, no higher cortical dysfunction, no brain stem disturbance and no drowsiness in an LACS.

198
Q

An 81-year-old woman is unable to get out of bed one morning because she is weak in her right arm and leg. You notice that she has trouble understanding what you ask her, although she can speak comprehensively. On examination, she is blind in the lateral half of the visual field of the right eye.

A.	 Lacunar stroke
B.	 Partial anterior circulation stroke
C.	 Posterior circulation stroke
D.	 Total anterior circulation stroke
E.	 Transient ischaemic attack
A

D – Total anterior circulation stroke

The classification of acute stroke was first described by Bamford and the Oxfordshire Community Stroke Project in the 1990s as follows. A total anterior circulation stroke (TACS) results in a triad of hemiplegia, hemianopia and higher cortical dysfunction (e.g. aphasia and visuospatial problems). A partial anterior circulation stroke (PACS) displays two of the above three features. A posterior circulation stroke (POCS) affects the brain stem, resulting in vertigo, dysphagia, dysarthria and facial weakness.

199
Q

Common in South America, where maize
forms the staple diet. The main physical features are weakness, dermatitis and
diarrhoea.

A.	 Cushing’s syndrome
B.	 Delirium
C.	 Hyperthyroidism
D.	 Hypothyroidism
E.	 Multiple sclerosis
F.	 Neurosyphilis
G.	 Pellagra
H.	 Punch-drunk syndrome
I.	 Space-occupying lesion
J.	 Systemic lupus erythematosus
A

G - Pellagra

Pellagra is a disorder caused by niacin (vitamin B3) deficiency. The main physical features are weakness, dermatitis and diarrhoea. There can also be personality changes, progressing to dementia. Treatment is with niacin. Death can occur within a few years if the condition is untreated.

200
Q

A 70-year-old man with known angina, hypertension and hypercholesterolemia is experiencing severe episodic abdominal pains. These occur approximately 30 minutes after eating. A gastroscopy and colonoscopy show no abnormalities.

A.	 Acute angina
B.	 Acute mesenteric infarction
C.	 Acute myocardial infarction
D.	 Acute pulmonary oedema
E.	 Chronic left ventricular failure (NYHA class I)
F.	 Chronic left ventricular failure (NYHA class III)
G.	 Drug side-effect
H.	 Epilepsy
I.	 Hypertrophic cardiomyopathy
J.	 Infective endocarditis
K.	 Mesenteric ischaemia
L.	 Postural hypotension
M.	Rheumatic fever
N.	 Stokes–Adams collapse
O.	 Vasovagal syncope
A

K – Mesenteric ischaemia

Mesenteric ischaemia is caused by atherosclerotic disease of the coeliac trunk and mesenteric arteries. Stenosis of the mesenteric arteries leads to inadequate perfusion of the bowel at times of increased oxygen demand, such as when digesting food. The condition usually presents with severe, griping abdominal pain 15–45 minutes after eating. Patients often fear to eat, because of the anticipation of pain, and can lose significant amounts of weight as a result.

Mesenteric ischaemia must be differentiated from acute mesenteric infarction, which occurs when a local atherosclerotic plaque ruptures or when the artery becomes occluded by an embolus from a distant source, e.g. the left atrium in atrial fibrillation. Mesenteric infarction is extremely painful and typically lacks clinical signs. If it is suspected, an urgent surgical opinion is warranted.

201
Q

A 52-year-old woman presents to the GP with a 3-week history of worsening shortness of breath and cough productive of sputum. She has also developed fevers and night sweats. Auscultation of the lungs reveals crackles in the upper zones on both sides. She is known to have HIV infection.

A.	 Candidiasis
B.	 Cryptococcus infection
C.	 Cryptosporidiosis
D.	 Cytomegalovirus infection
E.	 HIV dementia
F.	 HIV-wasting syndrome
G.	 Kaposi’s sarcoma
H.	 Lichen planus
I.	 Mycobacterium avium complex
J.	 Pneumocystis infection
K.	 Seroconversion
L.	 Stevens–Johnson syndrome
M.	Toxoplasmosis
A

I – Mycobacterium avium complex

Mycobacterium avium complex (MAC) is a group of related airborne bacteria that includes Mycobacterium avium–intracellulare. Because of the low pathogenicity of these bacteria, clinical symptoms are usually found only in immunocompromised patients. Features are similar to those of tuberculosis: productive cough, shortness of breath, fever, lethargy, night sweats, weight loss, diarrhoea and abdominal pain. A chest X-ray may demonstrate the pulmonary MAC lesions. Sputum stain and culture can also help establish diagnosis. Treatment is with antituberculous drugs, including rifampicin, ethambutol and amikacin.

202
Q

A 67-year-old man presents to the emergency department with palpitations. He has known ischaemic heart disease and hypertension. On examination, he is pale and appears confused, and his hands are cool and clammy. His pulse is 160 beats/min and irregularly irregular, his blood pressure is 70/40 mmHg and his capillary refill time is 6s.

A.	 Adenosine
B.	 Atenolol
C.	 Digoxin
D.	 Non-steroidal anti-inflammatory drugs
E.	 Paracetamol
F.	 Streptokinase
G.	 Sublingual glyceryl trinitrate
H.	 Synchronized DC cardioversion
I.	 Tenecteplase alone
J.	 Tenecteplase followed by intravenous heparin infusion
K.	 Unsynchronized DC cardioversion
L.	 Verapamil	
M.	Warfarin
A

H – Synchronized DC cardioversion

This patient with palpitations is in fast atrial fibrillation causing haemodynamic compromise. This is an emergency, and requires prompt action. Once the airway and breathing have been assessed and secured, the patient requires emergency DC cardioversion. DC cardioversion can be either synchronized (performed at a specific point of the cardiac cycle) or unsynchronized (done at any point, regardless of the cardiac cycle). In acute, unstable atrial fibrillation, shocks are synchronized with an R-wave on the ECG in order to reduce the chance of ventricular fibrillation. The patient needs to be adequately anticoagulated with intravenous heparin before the procedure, as cardioversion may precipitate systemic embolization from an intracardiac thrombus.

203
Q

A 25-year-old asthmatic man presents to the emergency department with palpitations. Examination reveals a regular heart rate of 168 beats/min. The ECG shows a regular narrow complex tachycardia with no obvious P-waves. The Valsalva manoeuvre and carotid sinus massage have failed to slow the heart rate.

A.	 Adenosine
B.	 Atenolol
C.	 Digoxin
D.	 Non-steroidal anti-inflammatory drugs
E.	 Paracetamol
F.	 Streptokinase
G.	 Sublingual glyceryl trinitrate
H.	 Synchronized DC cardioversion
I.	 Tenecteplase alone
J.	 Tenecteplase followed by intravenous heparin infusion
K.	 Unsynchronized DC cardioversion
L.	 Verapamil	
M.	Warfarin
A

A - Adenosine

This patient is experiencing a supraventricular tachycardia that is most likely to be an atrioventricular nodal re-entrant tachycardia (AVNRT) or atrioventricular re-entrant tachycardia (AVRT). The typical ECG findings are of a regular narrow complex tachycardia with no obvious P-waves. Provided that the patient is haemodynamically stable, the initial treatment is with vagal manoeuvres such as the Valsalva manoeuvre, and carotid sinus massage. The Valsalva manoeuvre, where the patient forcefully exhales against a closed epiglottis by blowing hard on a syringe, is the most commonly employed and safest vagal manoeuvre available.

If such methods fail to slow the heart rate, or if the patient is haemodynamically compromised, they will require cardioversion. The first-line method in AVNRT and AVRT is chemical cardioversion with adenosine. Adenosine is given as an intravenous bolus of 6 mg, increasing to 12 mg if cardioversion does not occur. Before administrating adenosine, the patient should be placed on a cardiac monitor and warned that they will experience significant chest tightness and dyspnoea.

204
Q

A 66-year-old man presents with recurrent falls. He denies any other problems. On examination, he has multiple spider naevi on his face and gynaecomastia. There is no neurological weakness, but he does display past-pointing. You notice that he places his feet wide apart when he walks.

A.	 Antalgic gait
B.	 Cerebellar ataxia
C.	 Extrapyramidal gait
D.	 Gait apraxia
E.	 High-stepping gait
F.	 Marche à petit pas
G.	 Pyramidal gait
H.	 Sensory ataxia
I.	 Spastic gait
J.	 Trendelenburg gait
K.	 Waddling gait
A

B – Cerebellar ataxia

The examination features of spider naevi and gynaecomastia suggest significant alcohol use. This patient is also displaying features of cerebellar dysfunction (wide-based gait and past pointing), which can be a consequence of alcohol excess.

Cerebellar dysfunction results in a characteristic set of features summarized by the acronym DANISH:
Dysdiadochokinesis 
Ataxia (wide-based gait)
Nystagmus
Intention tremor
Slurred speech
Hypotonicity.

Less severe degrees of cerebellar ataxia can be elicited by asking the patient to walk heel to toe.

205
Q

A 66-year-old man presents with recurrent falls. He says that he has difficulty starting to walk and that, once he has started, he speeds up uncontrollably. On examination, he has a tremor in his arms.

A.	 Antalgic gait
B.	 Cerebellar ataxia
C.	 Extrapyramidal gait
D.	 Gait apraxia
E.	 High-stepping gait
F.	 Marche à petit pas
G.	 Pyramidal gait
H.	 Sensory ataxia
I.	 Spastic gait
J.	 Trendelenburg gait
K.	 Waddling gait
A

C – Extrapyramidal gait

This man has difficulty walking and controlling the speed of his gait. This is typical of an extrapyramidal gait seen in Parkinson’s disease. Patients find it difficult to initiate walking. They start with stuttering steps that quickly increase in frequency while decreasing in length (also known as a festinant gait). Patients with Parkinson’s disease may also get ‘stuck’ when walking though doors (‘freezing’).

206
Q

A 66-year-old man presents with recurrent falls. He was admitted to hospital only last month with weakness in his right arm and leg. On examination, his right arm is flexed at the elbow and his right leg is extended at the hip. When he walks, he swings his right leg out to stop it from dragging.

A.	 Antalgic gait
B.	 Cerebellar ataxia
C.	 Extrapyramidal gait
D.	 Gait apraxia
E.	 High-stepping gait
F.	 Marche à petit pas
G.	 Pyramidal gait
H.	 Sensory ataxia
I.	 Spastic gait
J.	 Trendelenburg gait
K.	 Waddling gait
A

I – Spastic gait

This man has recently had a stroke causing a right-sided spastic hemiparesis. This results in a gait where the upper limb is held in flexion, the hip in extension and the ankle plantarflexed. Patients with spastic hemiparesis find that they need to swing their affected leg out from the hip (circumduction) to prevent it from scuffing the ground on walking.

Cerebral palsy causes diplegic gait (scissoring) which is when this happens with both legs.

207
Q

A 47-year-old woman complains that the skin of her fingers is becoming tight and it is becoming difficult for her to type. She is also having some problems swallowing. On examination, you notice that she has tight skin on both arms distal to the elbow and a furrowed mouth.

A.	 Anti-centromere
B.	 Anti-dsDNA
C.	 Anti-Jo-1
D.	 Anti-Ro
E.	 Anti-topoisomerase II
F.	 Anti-U1-RNP
G.	 Anticardiolipin
H.	 cANCA
I.	 pANCA
J.	 Rheumatoid factor
A

A – Anti-centromere

This is the limited cutaneous form of systemic sclerosis (scleroderma) - previously known as CREST syndrome - with skin thickening confined to sites distal to the elbows and knees, but can also involve the face.

5 CREST features are: Calcinosis, Raynaud’s, oEsophageal dysmotility, Sclerodactyly (skin fibrosis on fingers), Telangiectasia

Associated with ANA, anti-centromere (limited form only) antibodies & Rhematoid factor.

DIFFUSE cutaneous systemic sclerosis: skin thickening on the proximal limbs or the trunk in addition to face and distal extremities.
Anti-Scl 70 (anti-topoisomerase) is more associated with the DIFFUSE form, with ANA

208
Q

A 56-year-old woman has a 2-month history of progressive weakness in her hips. She finds it especially difficult to walk up the stairs at home. On examination, both hips are tender. She also has a purple rash over her eyelids.

A.	 Anti-centromere
B.	 Anti-dsDNA
C.	 Anti-Jo-1
D.	 Anti-Ro
E.	 Anti-topoisomerase II
F.	 Anti-U1-RNP
G.	 Anticardiolipin
H.	 cANCA
I.	 pANCA
J.	 Rheumatoid factor
A

C – Anti-Jo-1

Dermatomyositis here, but anti-Jo-1 is more commonly found in polymyositis.

General ANA are found in both.

209
Q

A 28-year-old man presents with a 2-month history of pain in the small joints of his hand. He has also developed a rash on his cheeks and nose, which he feels gets worse in the sun.

A.	 Anti-centromere
B.	 Anti-dsDNA
C.	 Anti-Jo-1
D.	 Anti-Ro
E.	 Anti-topoisomerase II
F.	 Anti-U1-RNP
G.	 Anticardiolipin
H.	 cANCA
I.	 pANCA
J.	 Rheumatoid factor
A

B – Anti-dsDNA

SLE, also associated with anti-Smith and ANA (most sensitive) antibodies.

Anti-Smith and anti-dsDNA are both specific for SLE.

210
Q

A 41-year-old man has had frequent nosebleeds over the last month. He has never had one before this. In addition, he has recently started coughing up blood. Examination is unremarkable.

A.	 Anti-centromere
B.	 Anti-dsDNA
C.	 Anti-Jo-1
D.	 Anti-Ro
E.	 Anti-topoisomerase II
F.	 Anti-U1-RNP
G.	 Anticardiolipin
H.	 cANCA
I.	 pANCA
J.	 Rheumatoid factor
A

H – cANCA

(Wegener’s) granulomatosis with polyangiitis.

211
Q

A 26-year-old woman presents with sudden left-sided weakness, and a stroke is diagnosed. She has a past history of a left-sided deep vein thrombosis and a pulmonary embolus. Routine blood tests show a haemoglobin of 12.5 g/dL, a white cell count of 8 × 109/L and a platelet count of 68 × 109/L.

A.	 Anti-centromere
B.	 Anti-dsDNA
C.	 Anti-Jo-1
D.	 Anti-Ro
E.	 Anti-topoisomerase II
F.	 Anti-U1-RNP
G.	 Anticardiolipin
H.	 cANCA
I.	 pANCA
J.	 Rheumatoid factor
A

G – Anticardiolipin

Antiphospholipid syndrome, also associated with Anti-beta2-glycoprotein I antibody.

Hx of vascular thrombosis, pregnancy loss, SLE, arthritis, thrombocytopaenia.

212
Q

A 34-year-old man presents to the emergency department with epigastric pain. He admits having had brain surgery in the past for a tumour that caused him to produce milk. While you are taking a history, the patient suffers a large haematemesis, and an urgent endoscopy is arranged. The endoscopist notes multiple large ulcers throughout the stomach, duodenum and jejunum.

A.	 Acromegaly
B.	 Addison’s disease
C.	 Carcinoid tumour
D.	 Congenital adrenal hyperplasia
E.	 Conn’s syndrome
F.	 Cushing’s syndrome
G.	 Hyperparathyroidism
H.	 Hypoparathyroidism
I.	 Multiple endocrine neoplasia type 1
J.	 Multiple endocrine neoplasia type 2A
K.	 Multiple endocrine neoplasia type 2B
L.	 Nelson’s syndrome
M.	Phaeochromocytoma
A

I – Multiple endocrine neoplasia type 1

This young patient has multiple peptic ulcers, which are suggestive of a gastrinoma, a pancreatic tumour. The brain tumour that causes lactation is a prolactinoma, a tumour of the pituitary gland. The combination of pancreatic endocrine tumours with a gastrinoma suggests multiple endocrine neoplasia (MEN) type I.

There are three types of MEN, all of which are autosomal dominant conditions. MEN type 1 (Wermer’s syndrome) includes the presence of parathyroid adenomas, pancreatic islet cell tumours and pituitary adenomas. MEN type 2A (Sipple’s syndrome) comprises parathyroid adenomas, medullary carcinoma of the thyroid and phaeochromocytoma. Finally, MEN type 2B (which has in the past been termed ‘MEN type 3’) includes the presence of the tumours of MEN type 2A with the addition of multiple mucosal neuromas of the gastrointestinal tract and a marfanoid phenotype.

213
Q

A 48-year-old woman presents with frequency of urine and excessive thirst. She also complains of occasional muscle cramps. She has history of hypertension, but is not on any regular medication. Her capillary glucose is 4.2 mmol/L, and routine blood tests show a sodium of 148 mmol/L and a potassium of 3.2 mmol/L.

A.	 Acromegaly
B.	 Addison’s disease
C.	 Carcinoid tumour
D.	 Congenital adrenal hyperplasia
E.	 Conn’s syndrome
F.	 Cushing’s syndrome
G.	 Hyperparathyroidism
H.	 Hypoparathyroidism
I.	 Multiple endocrine neoplasia type 1
J.	 Multiple endocrine neoplasia type 2A
K.	 Multiple endocrine neoplasia type 2B
L.	 Nelson’s syndrome
M.	Phaeochromocytoma
A

E – Conn’s syndrome

This woman presents with polyuria, polydipsia and muscle cramps associated with a high sodium and a low potassium. This suggests a diagnosis of Conn’s syndrome. Conn’s syndrome results from an aldosterone-secreting adenoma of the adrenal gland. Aldosterone causes sodium reabsorption and potassium excretion in the kidneys. Excess aldosterone results in sodium and water retention, leading to high blood pressure and oedema. Excess potassium excretion results in hypokalaemia, the features of which include muscle cramps and weakness, polyuria (secondary to renal tubular damage, i.e. nephrogenic diabetes insipidus) and polydipsia.

The diagnosis of Conn’s syndrome can be made by measuring serum aldosterone and renin levels – aldosterone will be raised and renin levels will be reduced due to negative feedback. It should be noted that many antihypertensive medications interfere with these hormones, so it is important to stop these for at least 6 weeks before testing. Initial management is with spironolactone, an aldosterone antagonist. Once the adenoma has been localized (using CT), it can be surgically removed.

214
Q

A 24-year-old man presents to his GP with a lump in his neck that has been growing slowly over a few weeks. On examination, you note that he is tall and has long digits. He tells you that his father died of adrenal cancer.

A.	 Acromegaly
B.	 Addison’s disease
C.	 Carcinoid tumour
D.	 Congenital adrenal hyperplasia
E.	 Conn’s syndrome
F.	 Cushing’s syndrome
G.	 Hyperparathyroidism
H.	 Hypoparathyroidism
I.	 Multiple endocrine neoplasia type 1
J.	 Multiple endocrine neoplasia type 2A
K.	 Multiple endocrine neoplasia type 2B
L.	 Nelson’s syndrome
M.	Phaeochromocytoma
A

K – Multiple endocrine neoplasia type 2B

This patient is likely to have MEN type 2B, an autosomal dominant condition characterized by phaeochromocytoma, parathyroid adenomas, medullary thyroid cancer, multiple mucosal neuromas and a marfanoid habitus (e.g. tall, arachnodactylyl, high-arched palate).